1. Trang chủ
  2. » Trung học cơ sở - phổ thông

Toán 11 Chương 4 [VIETMATHS.COM] Chuyên ĐềSoHocVMF

150 65 0

Đang tải... (xem toàn văn)

Tài liệu hạn chế xem trước, để xem đầy đủ mời bạn chọn Tải xuống

THÔNG TIN TÀI LIỆU

Nội dung

giúp làm quen dễ dàng hơn với sự kì diệu của những con số cho đến những vấn đề đòi hỏi nhiều tư duy hơn như đồng dư, số nguyên tố, các phương trình Diophantine mà nổi tiếng nhất là định [r]

(1)

Chuyên đề

SỐ HỌC

(2)(3)

Chuyên đề

SỐ HỌC

Chế

Trần Quốc Nhật Hân [perfectstrong] Trần Trung Kiên [Ispectorgadget] Phạm Quang Toàn [Phạm Quang Toàn]

Lê Hữu Điền Khuê [Nesbit] Đinh Ngọc Thạch [T*genie*]

c

(4)(5)

Lời giới thiệu

Bạn đọc thân mến,

Số học phân mơn quan trọng tốn học gắn bó với xun suốt q trình học Tốn từ bậc tiểu học đến trung học phổ thông Chúng ta tiếp xúc với Số học bắt đầu khái niệm đơn giản tính chia hết, ước chung lớn nhất, bội chung nhỏ giúp làm quen dễ dàng với kì diệu số vấn đề đòi hỏi nhiều tư đồng dư, số nguyên tố, phương trình Diophantine mà tiếng định lý lớn Fermat , từ tầm vi mô đến vĩ mô, từ cậu bé lớp bi bô chia hết Giáo sư thiên tài Andrew Wiles (người giải tốn Fermat), thấy thở Số học

Số học quan trọng lạ thay số chun đề viết lại khơng nhiều đem so với kho tàng đồ sộ viết bất đẳng thức diễn đàn mạng Xuất phát từ thiếu hụt để kỉ niệm trịn năm Diễn đàn Tốn học khai trương trang chủ (16/01/2012 - 16/01/2013), nhóm biên tập chúng tơi với nhiều thành viên tích cực diễn đàn chung tay biên soạn chuyên đề gửi đến bạn đọc

Chuyên đề tập hợp viết riêng lẻ tác giảNguyễn Mạnh Trùng Dương (duongld),Nguyễn Trần Huy (yeutoan11), Nguyễn Trung Hiếu(nguyentrunghieua),Phạm Quang Toàn(Phạm Quang Toàn),Trần Nguyễn Thiết Quân(L Lawliet),Trần Trung Kiên( Is-pectorgadget), Nguyễn Đình Tùng (tungc3sp) góp sức

(6)

ii

gián tiếp nhiều thành viên tích cực trênDiễn đàn Tốn họcnhư

Nguyen Lam Thinh, nguyenta98, Karl Heinrich Marx, The Gunner,perfectstrong

Kiến thức đề cập chun đề khơng giúp bạn phần hiểu sâu số khái niệm Số học trao đổi bạn nhiều dạng tập hay khó từ cấp độ dễ đến tốn kì thi Học sinh giỏi quốc gia, quốc tế Chuyên đề gồm7chương Chương1đề cập đến khái niệm Ước Bội.Số nguyên tốvà số toán giới thiệu chương2 Chương 3nói sâu vềCác tốn chia hết.Phương trình nghiệm ngun,Phương trình đồng dưđược phác họa chương4và5.Hệ thặng dư định lý Thặng dư Trung Hoa gửi đến qua chương trước kết thúc chuyên đề bằngMột số toán số học hay VMF chương7

Do thời gian chuẩn bị gấp rút nội dung chuyên đề chưa đầu tư thật tỉ mỉ cịn nhiều sai sót viết, mong bạn đọc thông cảm Mọi ủng hộ, đóng góp, phê bình độc giả nguồn động viên tinh thần to lớn cho ban biên tập cho tác giả để phiên cập nhật sau chuyên đề tốt hơn, đóng góp nhiều cho kho tàng học thuật cộng đồng toán mạng Chúng hi vọng qua chuyên đề giúp bạn tìm thêm cảm hứng số học thêm yêu vẻ đẹp số Mọi trao đổi góy ý xin gửi địa email :

contact@diendantoanhoc.net

(7)

Mục lục

i

Lời giới thiệu

1

Chương 1

Ước Bội

1.1 Ước số, ước số chung, ước số chung lớn 1.2 Bội số, bội số chung, bội số chung nhỏ 1.3 Bài tập đề nghị

9

Chương 2

Số Nguyên Tố

2.1 Một số kiến thức số nguyên tố 2.2 Một số toán số nguyên tố 13 2.3 Bài tập 19

2.4 Phụ lục: Bạn nên biết 24

29

Chương 3

Bài toán chia hết

3.1 Lý thuyết 29

3.2 Phương pháp giải tốn chia hết 31

57

Chương 4

Phương trình nghiệm nguyên

(8)

iv Mục lục

4.1 Xét tính chia hết 57 4.2 Sử dụng bất đẳng thức 74

4.3 Nguyên tắc cực hạn, lùi vơ hạn 86

89

Chương 5

Phương trình đồng dư

5.1 Phương trình đồng dư tuyến tính 89 5.2 Phương trình đồng dư bậc cao 90

5.3 Hệ phương trình đồng dư bậc ẩn 90 5.4 Bậc phương trình đồng dư 95

5.5 Bài tập 95

5.6 Ứng dụng định lý Euler để giải phương trình đồng dư 96

5.7 Bài tập 101

103

Chương 6

Hệ thặng dư định lý Thặng dư Trung Hoa

6.1 Một số kí hiệu sử dụng viết 103 6.2 Hệ thặng dư 104

6.3 Định lí thặng dư Trung Hoa 117 6.4 Bài tập đề nghị & gợi ý – đáp số 125

129

Chương 7

Một số toán số học hay VMF

7.1 m3+ 17 3n 129

(9)

Chương

1

Ước Bội

1.1 Ước số, ước số chung, ước số chung lớn

1.2 Bội số, bội số chung, bội số chung nhỏ

1.3 Bài tập đề nghị

Nguyễn Mạnh Trùng Dương(duongld) Nguyễn Trần Huy(yeutoan11)

Ước bội là2khái niệm quan trọng chương trình số học THCS Chuyên đề giới thiệu khái niệm tính chất ước, ước số chung, ước chung lớn nhất, bội, bội số chung, bội chung nhỏ Một số tập đề nghị vấn đề đề cập đến cuối viết

1.1

Ước số, ước số chung, ước số chung lớn nhất

Trong phần này, chúng tơi trình bày số khái niệm ước số, ước số chung ước số chung lớn kèm theo vài tính chất chúng Một số tập ví dụ cho bạn đọc tham khảo đưa

1.1.1 Định nghĩa

Định nghĩa 1.1 Số tự nhiên d 6= gọi ước số số tự nhiênakhi khiachia hết chod Ta nóidchia hếta, kí hiệud|a Tập hợp ước củaalà: U(a) ={d∈N:d|a}

(10)

2 1.1 Ước số, ước số chung, ước số chung lớn

Tính chất 1.1– NếuU(a) ={1;a}thì a số nguyên tố

Định nghĩa 1.2 NếuU(a)và U(b) có phần tử chung phần tử gọi ước số chung avàb Ta kí hiệu:

U SC(a;b) = {d∈N: (d|a)∧(d|b)}

= {d∈N: (d∈U(a))∧(d∈U(b))}

Tính chất 1.2– NếuU SC(a;b) ={1}thìavàbngun tố

Định nghĩa 1.3 Sốd∈Nđược gọi ước số chung lớn củaavàb

(a;b∈Z) dlà phần tử lớn tậpU SC(a;b) Ký hiệu ước chung lớn củaa vàblà U CLN(a;b),(a;b) hay gcd(a;b) 1.1.2 Tính chất

Sau số tính chất ước chung lớn nhất:

• Nếu (a1;a2; .;an) = ta nói số a1;a2; .;an nguyên tố

• Nếu (am;ak) = 1,∀m =6 k,{m;k} ∈ {1; 2; .;n} ta nói

a1;a2; .;an đơi ngun tố • c∈U SC(a;b)

a c;

b c

= (a;b)

c

• d= (a;b)⇔

a d;

b d

= • (ca;cb) =c(a;b)

• (a;b) = vàb|acthìb|c

• (a;b) = và(a;c) = 1thì (a;bc) = • (a;b;c) = ((a;b);c)

• Choa > b >0

– Nếua=b.q (a;b) =b

(11)

1.1 Ước số, ước số chung, ước số chung lớn

1.1.3 Cách tìm ước chung lớn thuật tốn Euclide Để tìm (a;b) a khơng chia hết cho b ta dùng thuật toán Euclide sau:

a=b.q+r1 thì(a;b) = (b;r1) b=r1.q1+r2 thì(b;r1) = (r1;r2) · · ·

rn−2=rn−1.qn−1+rnthì (rn−2;rn−1) = (rn−1;rn) rn−1=rn.qn (rn−1;rn) =rn

(a;b) =rn

(a;b) số dư cuối khác0 thuật toán Euclide 1.1.4 Bài tập ví dụ

Ví dụ 1.1 Tìm (2k−1; 9k+ 4), k∈N∗ Lời giải Ta đặtd= (2k−1; 9k+ 4) Theo tính chất ước số chung ta cód|2k−1vàd|9k+ Tiếp tục áp dụng tính chất chia hết ta lại có d|9(2k−1)vàd|2(9k+ 4) Suy rad|2(9k+ 4)−9(2k−1)hay d|17

Vậy(2k−1; 9k+ 4) =

Ví dụ 1.2 Tìm (123456789; 987654321)

Lời giải Đặtb= 123456789;a= 987654321 Ta nhận thấy avàb chia hết cho

Ta lại có :

a+b = 1111111110 = 10

10−10

9

⇔9a+ 9b = 1010−10

(1.1)

Mặt khác :

10b+a = 9999999999

= 1010−1 (1.2)

(12)

4 1.2 Bội số, bội số chung, bội số chung nhỏ

Trừ (1.2) (1.1) vế theo vế ta đượcb−8a= Do đặtd= (a;b) thì9 d

Màa vàbđều chia hết cho 9, suy d=

Dựa vào thuật tốn Euclide, ta có lời giải khác cho Ví dụ1.2như sau : Lời giải 987654321 = 123456789.8+9thì(987654321; 123456789) =

(123456789; 9)

123456789 = 9.1371421

(123456789; 987654321) =

Ví dụ 1.3 Chứng minh dãy số An =

2n(n+ 1), n ∈ N

∗ chứa

những dãy số vô hạn số đôi nguyên tố Lời giải Giả sử dãy xét có k số đôi nguyên tố làt1 = 1;t2 = 3; .;tk =m(m ∈N∗) Đặt a=t1t2 tk Xét số hạngt2a+1 dãy An:

t2a+1 =

2(2a+ 1)(2a+ 2) = (a+ 1)(2a+ 1) ≥ tk

(13)

1.2 Bội số, bội số chung, bội số chung nhỏ

1.2.1 Định nghĩa

Định nghĩa 1.4 Số tự nhiên m gọi bội số a 6= khim chia hết choahay alà ước số m Nhận xét Tập hợp bội số củaa6= 0là:B(a) ={0;a; 2a; .;ka}, k∈

Z

Định nghĩa 1.5 Số tự nhiên m gọi bội số a 6= khim chia hết choahay alà ước số m Định nghĩa 1.6 Nếu tậpB(a)vàB(b)có phần tử chung phần tử chung gọi bội số chung a b Ta ký hiệu bội số chung củaavàb:BSC(a;b)

Định nghĩa 1.7 Số m 6= gọi bội chung nhỏ a

b m phần tử dương nhỏ tập BSC(a;b) Ký hiệu :

BCN N(a;b),[a;b]hay lcm(a;b) 1.2.2 Tính chất

Một số tính chất bội chung lớn nhất: • Nếu[a;b] =M

M a;

M b

= • [a;b;c] = [[a;b];c]

• [a;b].(a;b) =a.b 1.2.3 Bài tập ví dụ

Ví dụ 1.4 Tìm [n;n+ 1;n+ 2]

Lời giải Đặt A = [n;n+ 1] B = [A;n+ 2] Áp dụng tính chất [a;b;c] = [[a;b];c], ta có: B = [n;n+ 1;n+ 2]

Dễ thấy(n;n+ 1) = 1, suy [n;n+ 1] =n(n+ 1)

(14)

6 1.3 Bài tập đề nghị

Lại áp dụng tính chất[a;b] = a.b

(a;b)

[n;n+ 1;n+ 2] = n(n+ 1)(n+ 2) (n(n+ 1);n+ 2)

Gọid= (n(n+ 1);n+ 2) Do(n+ 1;n+ 2) = 1nên

d = (n;n+ 2) = (n; 2)

Xét hai trường hợp:

• Nếunchẵn thìd= 2, suy [n;n+ 1;n+ 2] = n(n+ 1)(n+ 2)

2

• Nếunlẻ d= 1, suy [n;n+ 1;n+ 2] =n(n+ 1)(n+ 2)

Ví dụ 1.5 Chứng minh [1; 2; .2n] = [n+ 1;n+ 2; .; 2n] Lời giải Ta thấy trongksố nguyên liên tiếp có số chia hết chok Do bất số{1; 2; .; 2n}đều ước số số{n+ 1;n+ 2; .; 2n} Do [1; 2; n; 2n] = [n+ 1;n+ 2; .; 2n]

1.3

Bài tập đề nghị

Thay cho lời kết, xin gửi đến bạn đọc số tập đề nghị để luyện tập nhằm giúp bạn quen với khái niệm tính chất trình bày chuyên đề

Bài1 a Cho A = 5a+ 3b;B = 13a+ 8b(a;b ∈ N∗) chứng minh

(A;B) = (a;b)

b Tổng quátA=ma+nb;B=pa+qbthỏa mãn|mq−np|= 1với a, b, m, n, p, q∈N∗ Chứng minh (A;B) = (a;b).

(15)

1.3 Bài tập đề nghị

Bài3 Từ chữ số 1; 2; 3; 4; 5; thành lập tất số có sáu chữ số (mỗi số viết lần) TìmU CLN tất số Bài4 ChoA= 2n+ 1;B = n(n+ 1)

2 (n∈N

∗) Tìm (A;B).

Bài5 a Chứng minh số tự nguyên liên tiếp chọn số nguyên tố với số lại

b Chứng minh trong16số nguyên liên tiếp chọn số nguyên tố với số lại Bài6 Cho1≤m≤n(m;n∈N)

a Chứng minh (22n−1; 22n+ 1) =

b Tìm(2m−1; 2n−1)

Bài7 Chom, n∈Nvới(m, n) = Tìm(m2+n2;m+n)

Bài8 ChoA= 2n+3;B = 2n+1+3n+1(n∈

N∗);C= 2n+2+3n+2(n∈ N∗) Tìm(A;B)và (A;C)

Bài9 Cho sáu số nguyên dươnga;b;a0;b0;d;d0sao cho(a;b) =d; (a0;b0) =

d0 Chứng minh (aa0;bb0;ab0;a0b) =dd0 Bài10 Chứng minh dãy sốBn=

1

6n(n+ 1)(n+ 2)(n∈N

∗)chứa

vô hạn số nguyên tố

Bài11 Chứng minh dãy số2n−3 với n∈N vàn≥2 chứa dãy số vô hạn số nguyên tố

Bài12 Chứng minh dãy MersenMn = 2n−1(n∈N∗) chứa dãy số vô

hạn số nguyên tố Bài13 Chứng minh dãy Fermat Fn = 22

n

+ 1(n ∈ N) dãy số nguyên tố

Bài14 Chon∈N;n >1và 2n−2 chia hết cho n Tìm(22

n

; 2n−1)

(16)

8 1.3 Bài tập đề nghị

Bài15 Chứng minh với mọin∈N, phân số 21n+

14n+ tối giản Bài16 Cho ba số tự nhiêna;b;cđôi nguyên tố Chứng

minh rằng(ab+bc+ca;abc) =

Bài17 Cho a;b ∈ N∗ Chứng minh tồn vô số n∈ N cho

(a+n;b+n) =

Bài18 Giả sửm;n∈N(m≥n)thỏa mãn(199k−1;m) = (1993−1;n) Chứng minh tồn tạit(t∈N) cho m= 1993t.n Bài19 Chứng minh a;m ∈N;a >1

am−1

a−1 ;a−1

= (m;a−1)

Bài20 Tìm số nguyên dươngnnhỏ để phân số sau tối giản:

a

n1996+ 1995n+ 2,

b

n1996+ 1995n+ 3,

c 1994

n1996+ 1995n+ 1995,

d 1995

n1996+ 1995n+ 1996

Bài21 Cho 20 số tự nhiên khác a1;a2; an có tổng S U CLN d Chứng minh U CLN S −a1;S−

(17)

Chương

2

Số Nguyên Tố

2.1 Một số kiến thức số nguyên tố

2.2 Một số toán số nguyên tố 13

2.3 Bài tập 19

2.4 Phụ lục: Bạn nên biết 24

Nguyễn Trung Hiếu (nguyentrunghieua) Phạm Quang Toàn(Phạm Quang Toàn)

2.1

Một số kiến thức số nguyên tố

2.1.1 Định nghĩa, định lý

Định nghĩa 2.1 Số nguyên tố số tự nhiên lớn 1, có

ước số

Định nghĩa 2.2 Hợp số số tự nhiên lớn có nhiều

ước

Nhận xét Các số số nguyên tố hợp số Bất kỳ số tự nhiên lớn có ước số ngun tố

Định lý 2.1– Dãy số nguyên tố dãy số vô hạn

(18)

10 2.1 Một số kiến thức số nguyên tố

Chứng minh Giả sử có hữu hạn số nguyên tố p1;p2;p3; ;pn; đópn số lớn nguyên tố

Xét sốN =p1p2 pn+ thìN chia cho số nguyên tốpi(i= 1, n) dư1 (*)

Mặt khácN hợp số (vì lớn số ngun tố lớn làpn) đóN phải có ước nguyên tố đó, tức N chia hết cho số pi (**)

Ta thấy (**) mâu thuẫn (*) Vậy khơng thể có hữu hạn số ngun tố

Định lý 2.2– Mọi số tự nhiên lớn phân tích thừa số nguyên tố cách (không kể thứ tự thừa số) Chứng minh * Mọi số tự nhiên lớn phân tích thừa số nguyên tố:

Thật vậy: giả sử điều khẳng định với số m thoả mãn: 1< m < nta chứng minh điều đến n

Nếunlà nguyên tố, ta có điều phải chứng minh

Nếunlà hợp số, theo định nghĩa hợp số, ta có: n=a.b (với a, b < n) Theo giả thiết quy nạp:a vàb tích thừa số nhỏ nnên n tích cuả thừa số nguyên tố

* Sự phân tích nhất:

Giả sử số m < n phân tích thừa số nguyên tố cách nhất, ta chứng minh điều đếnn:

Nếunlà số ngun tố ta điều phải chứng minh Nếu nlà hợp số: Giả sử có cách phân tích n thừa số nguyên tố khác nhau:

n=p.q.r n=p0.q0.r0

Trong p, q, r p0, q0, r0 số ngun tố khơng có số ngun tố có mặt hai phân tích (vì có số thoả mãn điều kiện trên, ta chia n cho số lúc thường nhỏ n, thương có hai cách phân tích thừa số nguyên tố khác nhau, trái với giả thiết quy nạp)

Khơng tính tổng quát, ta giả thiếtpvàp0 số nguyên tố nhỏ phân tích thứ thứ hai

(19)

2.1 Một số kiến thức số nguyên tố 11

Xét m = n−pp0 < n phân tích thừa số nguyên tố cách ta thấy:

p|n⇒p|n−pp0 hay p|m

Khi phân tích thừa số nguyên tố ta có:m=n−pp0 =p0p.P.Q với

P, Q∈P( P tập số nguyên tố)

⇒pp0|n⇒pp0|p.q.r ⇒p|q.r ⇒p ước nguyên tố củaq.r

Mà p không trùng với thừa số q, r (điều trái với gỉa thiết quy nạp số nhỏ n phân tích thừa số nguyên tố cách nhất)

Vậy, điều giả sử không Định lý chứng minh

2.1.2 Cách nhận biết số nguyên tố Cách

Chia số cho nguyên tố từ nhỏ đến lớn:2; 3; 5;

Nếu có phép chia hết số khơng ngun tố

Nếu thực phép chia lúc thương số nhỏ số chia mà phép chia có số dư số ngun tố

Cách

Một số có hai ước số lớn số khơng phải số nguyên tố Cho học sinh lớp học cách nhận biết số nguyên tố phương pháp thứ (nêu trên), dựa vào định lý bản:

Ước số nguyên tố nhỏ hợp số A số không vượt quá√A

Với quy tắc khoản thời gian ngắn, với dấu hiệu chia hết ta nhanh chóng trả lời số có hai chữ số

(20)

12 2.1 Một số kiến thức số nguyên tố

nguyên tố hay khơng

Hệ 2.1– Nếu có sốA >1 khơng có ước số ngun tố từ

2 đến √A thìA nguyên tố

2.1.3 Số ước số tổng ước số số Giả sử:A=px1

1 p x2

2 pnxn; đó: pi∈P;xi ∈N;i= 1, n Tính chất 2.1– Số ước số củaA tính cơng thức:

T(A) = (x1+ 1)(x2+ 1) (xn+ 1)

Ví dụ 2.1 30 = 2.3.5 T(A) = (1 + 1)(1 + 1)(1 + 1) = Kiểm tra: (30) ={1; 2; 3; 5; 6; 10; 15; 30} nên (30) có phân tử Tính chất 2.2– Tổng ước số A tính cơng thức:

σ(A) = n

Y

i=1

pxi+1

i −1

pi−1

2.1.4 Hai số nguyên tố

Định nghĩa 2.3 Hai số tự nhiên gọi nguyên tố chúng có ước chung lớn (ƯCLN) Tính chất 2.3– Hai số tự nhiên liên tiếp ln ngun tố Tính chất 2.4– Hai số nguyên tố khác nguyên tố

Tính chất 2.5– Các sốa, b, cnguyên tố khi(a, b, c)

=

(21)

2.2 Một số toán số nguyên tố 13

2.1.5 Một số định lý đặc biệt

Định lý 2.3 (Dirichlet)– Tồn vơ số số ngun tố p có dạng:

p=ax+b (x, a, b∈N, a, blà số nguyên tố nhau) Việc chứng minh định lý phức tạp, trừ số trường hợp đặc biệt, chẳng hạn có vơ số số ngun tố dạng:2x−1; 3x−1; 4x+ 3; 6x+ 5;

Định lý 2.4 (Tchebycheff-Betrand)– Trong khoảng từ số tự nhiên

nđến số tự nhiên 2ncó số ngun tố (n >2) Định lý 2.5 (Vinogradow)– Mọi số lẻ lớn 33 tổng số

nguyên tố

2.2

Một số toán số nguyên tố

2.2.1 Có số nguyên tố dạng ax+b

Ví dụ 2.2 Chứng minh rằng: có vơ số số nguyên tố có dạng 3x−1.4 Lời giải Mọi số tự nhiên khơng nhỏ có dạng:3x; 3x+ hoặc3x−1

• Những số có dạng3x (với x >1) hợp số

• Xét số có dạng3x+ 1: số3m+ 1và số3n+

Xét tích (3m+ 1)(3n+ 1) = 9mn+ 3m+ 3n+ Tích có dạng:3x+

• Lấy số nguyên tố p bất có dạng 3x−1, ta lập tích p

với tất số nguyên tố nhỏ p trừ ta có: M = 2.3.5.7 p−1 = 3(2.5.7 p)−1 M có dạng 3x−1

Có khả xảy ra:

1 Khả 1:M số nguyên tố, số ngun tố có dạng 3x−1> p, tốn chứng minh

(22)

14 2.2 Một số toán số nguyên tố

2 Khả 2:M hợp số: Ta chia M cho2,3,5, , pđều tồn số dư khác0nên ước nguyên tố M lớn hơnp, ước khơng có số có dạng3x+ 1(đã chứng minh trên) Do ước nguyên tố củaM phải có dạng3x (hợp số) 3x+

Vì tất có dạng3x+ 1thìM phải có dạng3x+ 1(đã chứng minh trên) Do đó, ước nguyên tố M

phải có dạng3x−1, ước ln lớn p

Vậy: Có vơ số số ngun tố dạng3x−1

Ví dụ 2.3 Chứng minh rằng: Có vơ số số ngun tố có dạng 4x+ 3.4 Lời giải Nhận xét Các số ngun tố lẻ khơng thể có dạng 4x 4x+ Vậy chúng tồn dạng 4x+ 4x+

Ta chứng minh có vơ số số ngun tố có dạng 4x+ • Xét tích số có dạng4x+ 1là: 4m+ 1và4n+

Ta có:(4m+1)(4n+1) = 16mn+4m+4n+1 = 4(4mn+m+n)+1 Vậy tích số có dạng4x+ 1là số có dạng 4x+ • Lấy số ngun tốp có dạng4x+ 3, ta lập tích của4p

với tất số nguyên tố nhỏ p trừ ta có:

N = 4(2.3.5.7 p)−1 Có khả xảy

1 N số nguyên tố⇒N = 4(2.3.5.7 p)−1 có dạng4x−1 Những số ngun tố có dạng4x−1cũng số có dạng 4x+ 3và tốn chứng minh

(23)

2.2 Một số toán số nguyên tố 15

Vậy: Có vơ số số ngun tố có dạng 4x−1 (hay có dạng4x+ 3)

Trên số tốn chứng minh đơn giản định lý Dirichlet: Có vơ số số ngun tố dạng ax+btrong đóa, b, x∈N,(a, b) = 2.2.2 Chứng minh số nguyên tố

Ví dụ 2.4 Chứng minh rằng: (p−1)! chia hết cho p nếup hợp số, không chia hết cho p nếup số nguyên tố Lời giải • Xét trường hợpplà hợp số: Nếuplà hợp số thìplà tích thừa số nguyên tố nhỏ hơnp số mũ luỹ thừa lớn số mũ luỹ thừa chứa (p−1)! Vậy: (p−1)! p (đpcm)

• Xét trường hợpp số nguyên tố: Vì p∈P⇒p nguyên tố với thừa số của(p−1)!(đpcm)

Ví dụ 2.5 Cho 2m−1 số nguyên tố Chứng minh m

số nguyên tố

Lời giải Giả sửmlà hợp số⇒m=p.q (p, q∈N;p, q >1)

Khi đó:2m−1 = 2pq−1 = (2p)q−1 = (2p−1)((2p)q−1+(2p)q−2+ +1) vìp >1⇒2p−1>1 và(2p)q−1+ (2p)q−2+ + 1>1

Dẫn đến2m−1 hợp số :trái với giả thiết2m˘1 số nguyên tố

Vậym phải số nguyên tố (đpcm)

Ví dụ 2.6 Chứng minh rằng: ước nguyên tố 1994!−1 lớn

hơn 1994

Lời giải Gọiplà ước số nguyên tố của1994!−1 Giả sửp≤1994⇒1994.1993 3.2.1 p⇒1994! p Mà1994!−1 p⇒1 p (vô lý)

Vậy:p >1994(đpcm)

Ví dụ 2.7 Chứng minh rằng: n >2 n n! có số nguyên tố (từ suy có vô số số nguyên tố)

(24)

16 2.2 Một số toán số nguyên tố

Lời giải Vìn >2 nên k=n!−1>1, k có ước số ngun tố p Tương tự tập 3, ta chứng minh ước nguyên tốpcủak lớn hơnk

Vậy:p > n⇒n < p < n!−1< n!(đpcm)

2.2.3 Tìm số nguyên tố thỏa mãn điều kiện cho trước Ví dụ 2.8 Tìm tất giá trị số nguyên tố p để: p+ 10

p+ 14cũng số nguyên tố

Lời giải Nếup= 3thì p+ 10 = + 10 = 13 vàp+ 14 = + 14 = 17 số nguyên tố nênp= giá trị cần tìm

Nếup >3⇒pcó dạng 3k+ 1hoặc dạng3k−1 • Nếup= 3k+ 1thìp+ 14 = 3k+ 15 = 3(k+ 5) • Nếup= 3k−1 thìp+ 10 = 3k+ = 3(k+ 3)

Vậy nếup >3thì p+ 10 hoặcp+ 14 hợp số : không thỏa mãn

bài Vậyp=

Ví dụ 2.9 Tìm k∈N để 10 số tự nhiên liên tiếp:

k+ 1;k+ 2;k+ 3; k+ 10

có nhiều số nguyên tố

Lời giải Nếuk= 0: từ đến 10 có số nguyên tố:2; 3; 5; Nếuk= 1: từ đến 11 có số nguyên tố: 2; 3; 5; 7; 11

Nếuk >1: từ trở khơng có số chẵn số nguyên tố Trong số lẻ liên tiếp, có số bội số đó, dãy có số nguyên tố

Vậy vớik= 1, dãy tương ứng:k+ 1;k+ 2, k+ 10có chứa nhiều số

nguyên tố (5 số nguyên tố)

Ví dụ 2.10 Tìm tất số nguyên tố p để:2p+p2 số nguyên

tố

(25)

2.2 Một số toán số nguyên tố 17

• p= 2⇒2p+p2 = 22+ 22 = 86∈P • p= 3⇒2p+p2 = 23+ 32 = 17∈P

• p >3⇒p6 Ta có2p+p2 = (p2−1) + (2p+ 1)

Vìp lẻ⇒2p+ vàp2−1 = (p+ 1)(p−1) 3⇒2p+p2 6∈P

Vậy có giá trị p= thoả mãn

Ví dụ 2.11 Tìm tất số nguyên tố p cho:p|2p+ Lời giải Vìp∈P:p|2p+ 1⇒p >2⇒(2;p) =

Theo định lý Fermat, ta có:p|2p−1−1 Mà

p|2p+ 1⇒p|2(2p−1−1) + 3⇒p|3⇒p=

Vậy:p=

2.2.4 Nhận biết số nguyên tố

Ví dụ 2.12 Nếu p số nguyên tố số8p+ 8p−1 số ngun tố số cịn lại số nguyên tố hay hợp số? Lời giải • Nếu p= 2⇒8p+ = 17∈P; 8p−1 = 156∈P

• Nếup= 3⇒8p−1 = 23∈P; 8p−1 = 256∈P

• Nếup >3, xét số tự nhiên liên tiếp:8p−1; 8pvà8p+ Trong số có số chia hết cho Nên hai số8p+ và8p−1 chia hết cho

Kết luận: Nếup∈Pvà số8p+ và8p−1 số nguyên tố

thì số cịn lại phải hợp số

Ví dụ 2.13 Nếu p ≥ 2p+ số nguyên tố 4p+

nguyên tố hay hợp số?

Lời giải Xét số tự nhiên liên tiếp: 4p; 4p+ 1; 4p+ Trong số có số bội

Màp≥5;p∈P nên pcó dạng 3k+ 1hoặc3k+

• Nếup= 3k+ 1thì2p+ = 6k+ 3: (trái với giả thiết)

(26)

18 2.2 Một số tốn số ngun tố

• Nếup= 3k+2 Khi đó4p+1 = 4(3k+2)+1 = 12k+9 3⇒4p+1

là hợp số

Ví dụ 2.14 Trong dãy số tự nhiên tìm 1997 số liên tiếp mà khơng có số ngun tố hay khơng ? Lời giải Chọn dãy số:(ai) : = 1998! +i+ (i= 1,1997)⇒ i+ 1∀i= 1,1997

Như vậy: Dãy số a1;a2;a3; a1997 gồm có 1997 số tự nhiên liên tiếp

khơng có số số nguyên tố

Ví dụ 2.15 (Tổng quát tập 2.14) Chứng minh tìm dãy số gồm n số tự nhiên liên tiếp (n > 1) khơng có số

là số ngun tố ?

Lời giải Ta chọn dãy số sau:(ai) :ai= (n+ 1)! +i+ 1⇒ai i+ ∀i= 1, n

Bạn đọc tự chứng minh dãy (ai) gồm có n số tự nhiên liên tiếp khơng có số số ngun tố

2.2.5 Các dạng khác

Ví dụ 2.16 Tìm số ngun tố cho tích chúng gấp lần tổng

của chúng

Lời giải Gọi số nguyên tố phải tìm làa, b, c Ta có:abc= 5(a+b+

c)⇒abc

Vì a, b, c có vai trị bình đẳng nên khơng tính tổng quát, giả sử:

a 5⇒a=

Khi đó:5bc= 5(5 +b+c)⇔5 +b+c=bc⇔(c−1)(b−1) = Do vậy:

  

b−1 =

c−1 = ⇔

b=

c= chọn

b−1 =

c−1 = ⇔

b=

c= loại

Vậy số(a;b;c) cần tìm hốn vị (2; 5; 7)

(27)

2.3 Bài tập 19

Lời giải Ta có:

p2= 8q+ 1⇒8q=p2−1 = (p+ 1)(p−1) (2.1) Dop2 = 8q+ :lẻ⇒p2 :lẻ⇒p:lẻ Đặtp= 2k+

Thay vào (2.1) ta có:

8q= 2k(2k+ 2)⇒2q=k(k+ 1) (2.2) Nếuq= 2⇒4 =k(k+ 1)⇒ khơng tìm đượck∈N

Vậyq >2 Vì q∈P⇒(2, q) = Từ (2.2) ta có:

a) k= vàq =k+ 1⇒ k= 2;q = Thay kết vào (2.2) ta có:p= 2.2 + =

b) q=k và2 =k+ 1⇒q= :loại

Vậy(q;p) = (5; 3)

2.3

Bài tập

2.3.1 Bài tập có hướng dẫn

Bài1 Ta biết có 25 số nguyên tố nhỏ 100 Tổng 25 số nguyên tố nhỏ 100 số chẵn hay số lẻ?

HD :Trong 25 số nguyên tố nhỏ 100 có chứa số nguyên tố chẵn 2, 24 số nguyên tố cịn lại số lẻ Do tổng 25 số nguyên tố số chẵn

Bài2 Tổng số nguyên tố bằng1012 Tìm số nguyên tố nhỏ ba số nguyên tố

HD: Vì tổng số nguyên tố 1012, nên số ngun tố tồn số nguyên tố chẵn Mà số nguyên tố chẵn số nguyên tố nhỏ Vậy số nguyên tố nhỏ số nguyên tố

(28)

20 2.3 Bài tập

Bài3 Tổng số nguyên tố 2003 hay khơng? Vì sao? HD: Vì tổng số nguyên tố 2003, nên số nguyên tố tồn số nguyên tố chẵn Mà số nguyên tố chẵn Do số ngun tố cịn lại 2001 Do 2001 chia hết cho 2001 > Suy 2001 số nguyên tố

Bài4 Tìm số nguyên tốp, cho p+ 2;p+ số nguyên tố

Bài5 Cho p vàp+ 4là số nguyên tố (p >3) Chứng minh

p+ 8là hợp số

HD: Vì p số nguyên tố p > 3, nên số nguyên tố p có dạng:

• Nếup= 3k+ 2thìp+ = 3k+ = 3(k+ 2)⇒p+

p+ 4>3 Do p+ 4là hợp số: trái đề

• Nếup= 3k+ 1thìp+ = 3k+ = 3(k+ 3)⇒p+

p+ 8>3 Do p+ 8là hợp số

Bài6 Chứng minh số nguyên tố lớn có dạng4n+1 hoặc4n−1

Bài7 Tìm số nguyên tố, biết số tổng hai số nguyên tố hiệu hai số nguyên tố

HD: Giả sử a, b, c, d, e số nguyên tố d > e Theo đề bài:

a=b+c=d−e (∗)

Từ (*)⇒a >2 nênalà số nguyên tố lẻ ⇒b+c;d−elà số lẻ Dob, dlà số nguyên tố ⇒b, dlà số lẻ ⇒c, elà số chẵn ⇒ c= e= (do c, elà số nguyên tố) ⇒ a=b+ = d−2 ⇒

d=b+

(29)

2.3 Bài tập 21

Bài8 Tìm tất số nguyên tố x, y cho:x2−6y2 = Bài9 Cho p vàp+ 2là số nguyên tố (p >3) Chứng minh

p+

2.3.2 Bài tập khơng có hướng dẫn

Bài1 Tìm số nguyên tố psao cho số sau số nguyên tố: a) p+ 2vàp+ 10

b) p+ 10vàp+ 20 c) p+ 10vàp+ 14 d) p+ 14vàp+ 20

e) p+ 2vàp+ f) p+ 2vàp+ 14 g) p+ 4vàp+ 10 h) p+ 8vàp+ 10

Bài2 Tìm số nguyên tố psao cho số sau số nguyên tố: a) p+ 2, p+ 8, p+ 12, p+ 14

b) p+ 2, p+ 6, p+ 8, p+ 14 c) p+ 6, p+ 8, p+ 12, p+ 14 d) p+ 2, p+ 6, p+ 8, p+ 12, p+ 14

e) p+ 6, p+ 12, p+ 18, p+ 24 f) p+ 18, p+ 24, p+ 26, p+ 32 g) p+ 4, p+ 6, p+ 10, p+ 12, p+ 16 Bài3 Cho trước số nguyên tốp >3 thỏa

a) p+ 4∈P Chứng minh rằng: p+ 8là hợp số b) 2p+ 1∈P Chứng minh rằng: 4p+ 1là hợp số

c) 10p+ 1∈P Chứng minh rằng:5p+ 1là hợp số

(30)

22 2.3 Bài tập

d) p+ 8∈P Chứng minh rằng: p+ 4là hợp số e) 4p+ 1∈P Chứng minh rằng: 2p+ 1là hợp số

f) 5p+ 1∈P Chứng minh rằng: 10p+ 1là hợp số g) 8p+ 1∈P Chứng minh rằng: 8p−1 hợp số h) 8p−1∈P Chứng minh rằng: 8p+ 1là hợp số

i) 8p2−1∈P Chứng minh rằng: 8p2+ 1là hợp số j) 8p2+ 1∈P Chứng minh rằng: 8p2−1 hợp số Bài4 Chứng minh rằng:

a) Nếu pvà q hai số nguyên tố lớn thìp2−q2 24 b) Nếua, a+k, a+ 2k(a, k∈N∗)là số nguyên tố lớn

3thì k

Bài5 a) Một số nguyên tố chia cho42có số dưr hợp số Tìm số dưr

b) Một số nguyên tố chia cho30có số dưr Tìm số dưr biết rằngr khơng số nguyên tố

Bài6 Tìm số nguyên tố có ba chữ số, biết viết số theo thứ tự ngược lại ta số lập phương số tự nhiên

Bài7 Tìm số tự nhiên có chữ số, chữ số hàng nghìn chữ số hàng đơn vị, chữ số hàng trăm chữ số hàng chục số viết dạng tích số nguyên tố liên tiếp

Bài8 Tìm số nguyên tố số lẻ liên tiếp

Bài9 Tìm số nguyên tố liên tiếpp, q, r cho p2+q2+r2∈P Bài10 Tìm tất ba số nguyên tố a, b, c cho abc < ab+

bc+ca

(31)

2.3 Bài tập 23

Bài12 Tìm số nguyên tốx, y, z thoả mãn xy+ =z

Bài13 Tìm số nguyên tốabcd thỏa ab, aclà số nguyên tố b2 =

cd+b−c

Bài14 Cho số p = bc+a, q = ab +c, r = ca+b(a, b, c ∈ N∗) là

các số nguyên tố Chứng minh sốp, q, r có hai số

Bài15 Tìm tất số nguyên tốx, y cho: a) x2−12y2 =

b) 3x2+ = 19y2

c) 5x2−11y2 = d) 7x2−3y2 =

e) 13x2−y2 = f) x2 = 8y+ 1

Bài16 Chứng minh điều kiện cần đủ đểpvà8p2+ 1là số nguyên tố làp=

Bài17 Chứng minh rằng: Nếua2−b2 số nguyên tố thìa2−b2 =

a+b

Bài18 Chứng minh số nguyên tố lớn hơn3đều có dạng6n+1 hoặc6n−1

Bài19 Chứng minh tổng bình phương của3số ngun tố lớn 3khơng thể số nguyên tố

Bài20 Cho số tự nhiênn≥2 Gọip1, p2, , pn số nguyên tố chopn≤n+ ĐặtA=p1.p2 pn Chứng minh dãy số số tự nhiên liên tiếp: A+ 2, A+ 3, , A+ (n+ 1), không chứa số nguyên tố

Bài21 Chứng minh rằng: Nếup số nguyên tố thì2.3.4 (p−3)(p− 2)−1 p

(32)

24 2.4 Phụ lục: Bạn nên biết

Bài22 Chứng minh rằng: Nếup số nguyên tố thì2.3.4 (p−2)(p− 1) + p

2.4

Phụ lục: Bạn nên biết

Mười số nguyên tố có 93 chữ số lập thành cấp số cộng Sau số nguyên tố gồm 93 chữ số:

100996972469714247637786655587969840329509324689190041 803603417758904341703348882159067229719

Kỷ lục 70 nhà toán học lập năm 1998 thật khó mà đánh bại Họ nhiều tháng tính tốn tìm mười số ngun tố tạo thành cấp số cộng

Từ mục trị chơi tạp chí khoa học, hai nhà nghiên cứu trường Đại học Lyonl (Pháp) đào sâu ý tưởng: Tìm số nguyên tố cho hiệu số liên tiếp luôn Điều dễ chuyên gia họ muốn xa Cũng khơng có vấn đề khó khăn dãy số Họ cần hỗ trợ chút để đạt số, hỗ trợ để đạt tới số Cuối tháng năm 1998 có 70 nhà toán học từ khắp giới với 200 máy điện tốn hoạt động liên tục tìm 10 số, số có 93 chữ số, mà hiệu số số liên tiếp luôn 210 Từ số nguyên tố cần thêm vào 210 số nguyên tố thứ

Kỷ lục có lẽ dừng đó: Theo ước tính nhà khoa học muốn tìm dãy 11 số nguyên tố phải 10 tỉ năm

“Sinh ba” ít, phải “sinh đơi” lại nhiều

(33)

2.4 Phụ lục: Bạn nên biết 25

Bài toán 2.1 Cho trước số nguyên dương n tuỳ ý Chứng minh tồn nsố tự nhiên liên tiếp mà số chúng hợp số.4 Vậy nhưng, số nguyên tố “có thể gần nhau” Cặp số(2,3) cặp số tự nhiên liên tiếp mà hai bên số nguyên tố Cặp số đ(p, q)ược gọi cặp số “sinh đôi”, số nguyên tố q =p+ Bộ số (p, q, r) gọi số nguyên tố “sinh ba” số p,q,r số nguyên tố vàq =p+ 2;r=q+

Bài toán 2.2 Tìm tất số nguyên tố “sinh ba”? Đây toán dễ, dùng phương pháp chứng minh ta tìm bộ(3,5,7)là ba số nguyên tố sinh ba nhất, số lẻ lớn ln có số hợp số chia hết cho

Từ tốn2.2thì tốn sau trở thành giả thuyết lớn chờ câu trả lời

Dự đoán 2.1– Tồn vô hạn cặp số sinh đôi

Số hoàn hảo (hoàn toàn) người Hy Lạp cổ đại Người Hy Lạp cổ đại có quan niệm thần bí số Họ thú vị phát số hoàn hảo, nghĩa số tự nhiên mà tổng ước số tự nhiên thực (các ước số nhỏ số đó) Chẳng hạn:

6 = + + 28 = + + + + 14

Người Hy Lạp cổ đại biết tìm tất số hồn hảo chẵn nghĩa họ làm toán sau đây:

Bài toán 2.3 Một số tự nhiên chẵn n6= số hoàn hảo nếu: n = 2m+1(2m−1) Trong m số tự nhiên khác cho

2m−1 số nguyên tố

Từ ta có giả thuyết

(34)

26 2.4 Phụ lục: Bạn nên biết

Dự đốn 2.2– Khơng tồn số hồn hảo lẻ

Ở toán 2.3 trên, số nguyên tố dạng 2m −1 gọi số nguyên tố Merseme Các số ngun tố Merseme có vai trị quan trọng Cho đến người ta chưa biết có hữu hạn hay vô hạn số nguyên tố Merseme

Dự đốn 2.3– Tồn vơ hạn số ngun tố Merseme Năm 1985 số nguyên tố lớn mà người ta biết số2132049−1gồm 39751chữ số ghi hệ thập phân Gần sinh viên Mỹ tìm số nguyên tố lớn số2216091−1gồm65050chữ số Ta biết với học sinh lớp để thử xem số A có 20 chữ số có số ngun tố khơng cách thử xem Acó chia hết cho số nhỏ A hay khơng, để tìm hết số ngun tố với máy siêu điện toán cần hàng kỷ !!!

David SlowinSky soạn phần mềm, làm việc máy siêu điện toán Gray-2 , sau 19 ông tìm số nguyên tố2756839−1 Số viết hệ thập phân có 227832 chữ số- viết hết số cần 110 trang văn bình thường Hoặc viết hàng ngang số phông chữ VnTime Size 14 ta cần khoảng 570 m

Lời Kết

Thông qua đề tài này, khẳng định rằng: Tốn học có mặt công việc, lĩnh vực sống quanh ta, khơng thể tách rời lãng qn được, nên phải hiểu biết nắm bắt cách tự giác hiệu

(35)

2.4 Phụ lục: Bạn nên biết 27

những năm học

Với điều kiện có nhiều hạn chế thời gian, lực trình độ nên khuôn khổ đề tài phân chia dạng tốn, loại tốn có tính tương đối Đồng thời đưa lời giải chưa có phương pháp, thuật làm rõ ràng Tuy có cố gắng nhiều chnsg tự thấy đề tài cịn nhiều hạn chế Chúng tơi mong nhận ý kiến đóng góp thầy giáo bạn đọc để tốn học thật có ý nghĩa cao đẹp câu ngạn ngữ Pháp viết:

“Toán học Vua khoa học” “Số học Nữ hoàng”

(36)(37)

Chương

3

Bài toán chia hết

3.1 Lý thuyết 29

3.2 Phương pháp giải toán chia hết 31

Phạm Quang Toàn (Phạm Quang Toàn)

Chia hết đề tài quan trọng chương trình Số học bậc THCS Đi kèm theo tốn khó hay Bài viết xin giới thiệu với bạn đọc phương pháp giải toán chia hết: phương pháp xét số dư, phương pháp quy nạp, phương pháp đồng dư, v.v

3.1

Lý thuyết bản

3.1.1 Định nghĩa chia hết

Định nghĩa 3.1 Cho hai số nguyêna vàb b6= 0, ta ln tìm hai số ngunq r cho

a=bq+r

với0≤r < b

Trong đó, ta nóialà số bị chia, blà số chia, q thương,r số dư.4 Như vậy, khiachia chobthì đưa số dưr ∈ {0; 1; 2;· · ·;|b|} Đặc biệt, vớir= 0thì a=bq, ta nóiachia hết chob(hoặcalà bội b, hoặcb ước củaa) Ta kí hiệu b|a Cịn akhơng chia

(38)

30 3.1 Lý thuyết

hết chob, ta kí hiệub-a

Sau số tính chất thường dùng, chứng minh suy trực tiếp từ định nghĩa

3.1.2 Tính chất

Sau xin giới thiệu số tính chất chia hết, việc chứng minh dễ dàng nên dành cho bạn đọc Ta có với a, b, c, d số ngun thì:

Tính chất 3.1– Nếua6= a|a,0|a

Tính chất 3.2– Nếub|athì b|ac

Tính chất 3.3– Nếub|avà c|bthì c|a Tính chất 3.4– Nếuc|avà c|bthì c|(ax±by)với x, yngun Tính chất 3.5– Nếub|avà a|b thìa=bhoặc a=−b

Tính chất 3.6– Nếuc|avà d|b thìcd|ab Tính chất 3.7– Nếub|a, c|athì BCNN(b;c)|a Tính chất 3.8– Nếuc|abvà UCLN(b, c) = thìc|a

Tính chất 3.9– Nếup|ab,p số nguyên tố thìp|ahoặcp|b Từ tính chất ta suy hệ

Hệ 3.1– Nếu p | an với p số nguyên tố, n nguyên dương

(39)

3.2 Phương pháp giải toán chia hết 31

3.1.3 Một số dấu hiệu chia hết Ta đặtN =anan−1 a1a0

Dấu hiệu chia hết cho2; 5; 4; 25; 8; 125

2|N ⇔ 2|a0 ⇔a0 ∈ {0; 2; 4; 6; 8} 5|N ⇔ 5|a0 ⇔a0 ∈ {0; 5} 4; 25|N ⇔ 4; 25|a1a0

8; 125|N ⇔ 8; 125|a2a1a0 Dấu hiệu chia hết cho3 và9

3; 9|N ⇔3; 9|(a0+a1+· · ·+an−1+an) Một số dấu hiệu chia hết khác

11|N ⇔ 11|[(a0+a2+· · ·)−(a1+a3+· · ·)]

101|N ⇔ 101|[(a1a0+a5a4+· · ·)−(a3a2+a7a6+· · ·)]

7; 13|N ⇔ 7; 37|[(a2a1a0+a8a7a6+· · ·)−(a5a4a3+a11a10a9+· · ·)] 37|N ⇔ 37|(a2a1a0+a5a4a3+· · ·+anan−1an−2)

19|N ⇔ 19| an+ 2an−1+ 22an−2+· · ·+ 2na0

3.2

Phương pháp giải toán chia hết

3.2.1 Áp dụng định lý Fermat nhỏ tính chất chia hết

Định lý Fermat nhỏ

Định lý 3.1 (Định lý Fermat nhỏ)– Với số nguyên a số

nguyên tốp thìap ≡p (mod p)

Chứng minh Nếup|athìp|(a5−a)

2 Nếup -a 2a,3a,4a,· · · ,(p−1)acũng không chia hết cho p

Gọir1, r2,· · ·, rp−1lần lượt số dư chiaa,2a,3a,· · · ,(p−1)a chop chúng thuộc tập{1; 2; 3;· · · ;p−1}và đơi khác (vì chẳng hạn r1 = r3 p | (3a−a) hay p | 2a,

(40)

32 3.2 Phương pháp giải toán chia hết

chỉ p = 2, màp = tốn khơng đúng) Do

r1r2·rp−1 = 1·2·3· · ·(p−1) Ta có

a≡r1 (modp) 2a≡r2 (modp)

· · ·

(p−1)a≡rp−1 (modp) Nhân vế theo vế ta suy

1·2·3· · ·(p−1)·ap−1 ≡r1r2· · ·rp−1 (modp)⇒ap−1 ≡1 (mod p) VìU CLN(a, p) = nên ap≡a (modp).

Như với số nguyênavà số nguyên tố pthì ap ≡a (mod p)

Nhận xét Ta chứng minh định lý quy nạp Ngồi ra, định lý cịn phát biểu dạng sau:

Định lý 3.2– Với số nguyên a,p số nguyên tố,U CLN(a, p) =

1 thìap−1 ≡1 (mod p)

Phương pháp sử dụng tính chất chia hết áp dụng định lý Fermat nhỏ

Cơ sở:Sử dụng tính chất chia hết định lý Fermat nhỏ để giải tốn

Ví dụ 3.1 Choavàblà hai số tự nhiên Chứng minh rằng5a2+15ab−

b2 chia hết cho49 3a+b chia hết cho7 Lời giải ⇒) Giả sử 49 |5a2+ 15ab−b2 ⇒ 7|5a2+ 15ab−b2 ⇒ 7| (14a2+ 21ab)−(5a2+ 15ab−b2) ⇒ | (9a2+ 6ab+b2) ⇒ | (3a+b)2 ⇒7|3a+b

⇐) Giả sử7|3a+b Đặt3a+b= 7c(c∈Z Khi b= 7c−3a Như

(41)

3.2 Phương pháp giải toán chia hết 33

chia hết cho49

Vậy 5a2+ 15ab−b2 chia hết cho 49khi khi3a+b chia hết cho

7

Ví dụ 3.2 Cho 11 |(16a+ 17b)(17a+ 16b) với a, b hai số nguyên Chứng minh 121|(16a+ 17b)(17a+ 16b) Lời giải Ta có theo đầu bài, 11 ngun tố nên hai số16a+ 17bvà 17a+ 16bchia hết cho11 Ta lại có(16a+ 17b) + (17a+ 16b) = 33(a+b) chia hết cho11 Do hai số 16a+ 17b và17a+ 16b chia hết cho11thì số cịn lại chia hết cho 11 Cho nên 121|(16a+ 17b)(17a+ 16b)

Ví dụ 3.3 Chứng minh A = 130+ 230+·+ 1130 không chia hết

cho 11

Lời giải Với a= 1,2,· · ·,10 (a,10) = Do theo định lý Fermat bé a10 ≡ 1 (mod 11) ⇒ a30 ≡ 1 (mod 11) với mọi a = 1,2,· · ·,10và 1130≡0 (mod 11) Như

A≡1 + +· · ·+

| {z }

10 số1

+0 (mod 11) ≡10 (mod 11)⇒11-A

Ví dụ 3.4 Cho p vàq hai số nguyên tố phân biệt Chứng minh

pq−1+qp−1−1 chia hết cho pq Lời giải Vìq nguyên tố nên theo định lý Fermat nhỏ

pq−1 ≡1 (modq) Do

pq−1+qp−1 ≡1 (mod q)

Vìq pcó vai trị bình đẳng nên ta dễ dàng suy

qp−1+pq−1 ≡1 (modp)

Cuối U CLN(q, p) = nên pq−1 +qp−1 ≡ (mod pq) hay

pq−1+qp−1−1 chia hết cho pq

(42)

34 3.2 Phương pháp giải toán chia hết

Bài tập đề nghị

Bài1 Chứng minh rằng11a+2bchia hết cho19khi khi18a+5b

chia hết cho 19với a, blà số nguyên

Bài2 Chứng minh 2a+ chia hết cho 3a2 + 10ab−8b2

Bài3 Chop số nguyên tố lớn hơn5 Chứng minh nlà số tự nhiên cóp−1chữ số chữ số bằng1thìnchia hết chop

Bài4 Giả sửn∈N, n≥2 Xét số tự nhiênan= 11·1 viết n chữ số Chứng minh nếuan số nguyên tố thìnlà ước an−1

Bài5 Giả sửavàb số nguyên dương cho2a−1,2b−1

a+b số nguyên tố Chứng minh rằngab+ba vàaa+bb

đều không chia hết choa+b

Bài6 Chứng minh với số nguyên tốpthì tồn số nguyên

nsao cho 2n+ 3n+ 6n−1chia hết cho p 3.2.2 Xét số dư

Cơ sở: Để chứng minh A(n) chia hết cho p, ta xét số n dạng

n=kp+r vớir ∈ {0; 1; 2;· · ·;p−1}

Chẳng hạn, với p = số nguyên n viết lại thành 5k; 5k+ 1; 5k+ 2; 5k+ 3; 5k+ Ta dạng vào vị trí nrồi lý luận đáp số Sau số ví dụ

Ví dụ 3.5 Tìm k∈N để tồn n∈N cho 4|n2−k

với k∈ {0; 1; 2; 3}

(43)

3.2 Phương pháp giải tốn chia hết 35

1 Nếun= 4mthì n2−k= 16m2−kchia hết cho 4khi 4|knên k=

2 Nếun= 4m±1thì n2−k= 16m2±8m+ 1−kchia hết cho4 khi4|1−k nên k=

3 Nếun= 4m±2thì n2−k= 16m2±16m+ 4−kchia hết cho4 khi4|k nên k=

Vậyk= hoặck=

Ví dụ 3.6 Chứng minh với mọin∈Nthì6|n(2n+ 7)(7n+ 1).4 Lời giải Ta thấy hai sốnvà 7n+ 1là số chẵn∀n∈N Do đó2|n(2n+ 7)(7n+ 1) Ta chứng minh3|n(2n+ 7)(7n+ 1) Thật vậy, xét

1 Vớin= 3kthì 3|n(2n+ 7)(7n+ 1)

2 Với n = 3k+ 2n+ = 6k+ chia hết |

n(2n+ 7)(7n+ 1)

3 Với n = 3k+ 7n+ = 21k+ 15 chia hết |

n(2n+ 7)(7n+ 1)

Do đó3|n(2n+ 7)(7n+ 1)mà(2,3) = 1nên6|n(2n+ 7)(7n+ 1)∀n∈

N

Ví dụ 3.7 (HSG 9, Tp Hồ Chí Minh, vịng 2, 1995) Cho x, y, z số nguyên thỏa mãn

(x−y)(y−z)(z−x) =x+y+z (3.1) Chứng minh 27|(x+y+z) Lời giải Xét hai trường hợp sau

(44)

36 3.2 Phương pháp giải toán chia hết

1 Nếu ba số x, y, z chia hết cho có số dư khác hiệux−y, y−z, z−xcùng khơng chia hết cho3 Mà3|(x+y+z) nên từ (3.1) suy vơ lí

2 Nếu ba sốx, y, zchỉ có hai số chia cho3có số dư ba hiệux−y, y−z, z−xcó hiệu chia hết cho3 Mà3-(x+y+z)

nên từ (3.1) suy vơ lí

Vậyx, y, z chia cho 3có số dư, đóx−y, y−z, z−xđều chia hết cho3 Từ (3.1) ta suy 27|(x+y+z), ta có đpcm

Bài tập đề nghị

Bài1 i) Tìm số tự nhiênnđể 7|(2n−1) ii) Chứng minh rằng7-(2n+ 1) ∀n∈N

Bài2 Chứng minh với số nguyên a a(a6−1) chia hết cho7

Bài3 Tìmnđể 13|32n+ 3n+

Bài4 Chứng minh với mọia, b∈Nthìab(a2−b2)(4a2−b2)ln

ln chia hết cho5

Bài5 Chứng minh 24|(p−1)(p+ 1) với p số nguyên tố lớn

Bài6 Chứng minh không tồn số nguyênađểa2+ 1chia hết cho12

Bài7 Chứng minh với số nguyênx, y, znếu 6|x+y+z 6|x3+y3+z3

Bài8 Choab= 20112012, vớia, b∈

N Hỏi tổnga+bcó chia hết cho

2012hay không ?

(45)

3.2 Phương pháp giải toán chia hết 37

Bài10 Cho số nguyên dươngx, y, z thỏa mãnx2+y2 =z2 Chứng minh rằngxyz chia hết cho60

Bài11 Cho số nguyên dươngx, y, zthỏa mãnx2+y2 = 2z2 Chứng minh rằngx2−y2 chia hết cho 84

Bài12 Chon >3, (n∈N) Chứng minh nếu2n= 10a+b, (0<

b <9)thì6|ab 3.2.3 Phân tích Phân tích thành tích

Cơ sở:Để chứng minhA(n)chia hết chop, ta phân tíchA(n) =D(n)p, cịn ta khơng thể đưa cách phân tích vậy, ta viếtp=kq

• Nếu(k, q) = ta chứng minhA(n)cùng chia hết cho k vàq • Nếu(k, q)6= 1thì ta viếtA(n) =B(n)C(n) chứng minhB(n)

chia hết chok,C(n)chia hết cho q

Ví dụ 3.8 Cho nlà số nguyên dương Chứng minh 2n|(n+ 1) (n+ 2)· · ·(2n)

Lời giải Ta có

(n+ 1) (n+ 2)· · ·(2n) =(2n)!

n! =

(1.3.5 (2n−1)) (2.4.6 2n)

n! = 1.3.5 (2n−1).2n.n!

n! = 1.3.5 (2n−1).2n

Do 2n|(n+ 1) (n+ 2)· · ·(2n)

(46)

38 3.2 Phương pháp giải tốn chia hết

Ví dụ 3.9 Chứng minh với số nguyên n thì6|n3−n Lời giải Phân tích

n3−n=n(n2−1) =n(n−1)(n+ 1)

Biểu thức tích ba số nguyên liên tiếp nên tồn ba số số chia hết cho2và số chia hết cho3 Mà(2,3) = 1nên

6|n3−n.

Ví dụ 3.10 Chứng minh n6−n4−n2+ chia hết cho 128với n

lẻ

Lời giải Ta có

n6−n4−n2+ = (n2−1)2(n+ 1) = (n−1)2(n+ 1)2 Vìnlẻ nên đặt n= 2k, k ∈N, suy

(n2−1)2 =(2k+ 1)2−1= (4k2+ 4k)2 = [4k(k+ 1)]2

Vậy64|(n2−1)2 Vì nlẻ nên2|n+ 1, suy đpcm

Ví dụ 3.11 Cho ba số nguyên dương khác x, y, z Chứng minh rằng(x−y)5+ (y−z)5+ (x−z)5 chia hết cho 5(x−y)(y−z)(x−z).4 Lời giải Ta có

(x−y)5+ (y−z)5+ (x−z)5

= (x−z+z−y)5+ (y−z)5+ (z−x)5

= (x−z)5+ 5(x−z)4(z−y) + 10(x−z)3(z−y)2 +10(x−z)4(z−y) + 10(x−z)3(z−y)2

+10(x−z)2(z−y)3+ 5(x−z)(z−y)4 = 5(x−z)(z−y)×

×

(47)

3.2 Phương pháp giải toán chia hết 39

Nhưng ta có:

(x−z)3+ 2(x−z)2(z−y) + 2(x−z)(z−y)2+ (z−y)3 = (x−y+y−z)3+ 2(x−y+y−z)2(z−y)

+2(x−y+y−z)(z−y)2+ (z−y)3

= (x−y)3+ 2(x−y)2(y−z) + 3(x−y)(y−z)2 +(y−z)3+ 2(x−y)2(z−y)

+4(x−y)(y−z)(z−y) + 2(y−z)2(z−y) +2(x−y)(z−y)2+ 2(y−z)(z−y)2+ (z−y)3 = (x−y)3+ 3(x−y)2(y−z) + 3(x−y)(y−z)2

+2(x−y)2(z−y) + 4(x−y)(y−z)(z−y) + 2(x−y)(z−y)2,

Biểu thức cuối có nhân tử chung (x−y) Ta suy điều phải

chứng minh

Bài tập đề nghị

Bài1 Chứng minh a, k số nguyên, a lẻ 2k+1 | (a2k−1)

Bài2 Chứng minh n5−n chia hết cho 30với n∈Z

Bài3 Chứng minh rằng3n4−14n3+ 21n2−10nchia hết cho 24 với n∈Z

Bài4 Chứng minh rằngn5−5n3+ 4nchia hết cho120với mọin∈Z

Bài5 Chứng minh rằngn3−3n2−n+ chia hết cho 48 với n

lẻ, n∈Z

Bài6 Chứng minh n8−n6−n4+n2 chia hết cho 1152với số nguyênnlẻ

Bài7 Chứng minh rằngn4−4n3−4n2+ 16nchia hết cho348với mọi

nlà số nguyên chẵn

Bài8 Chứng minh rằngn4−14n3+ 71n2−154n+ 120 chia hết cho 24 với số tự nhiênn

(48)

40 3.2 Phương pháp giải toán chia hết

Bài9 Cho x, y, z số nguyên khác Chứng minh

x2−yz =a, y2−zx =b, z2−xy =c thì tổng (ax+by+cz) chia hết cho tổng(a+b+c)

Bài10 Chom, n hai số phương lẻ liên tiếp Chứng minh

mn−m−n+ 1chia hết cho 192

Bài11 (HSG TQ 1970)Chứng minh rằngn12−n8−n4+ 1chia hết cho512với số tự nhiên nlẻ

Bài12 (HSG TQ 1975)Chứng minh rằngn4+ 6n3+ 11n2+ 6nchia hết cho24 với số nguyên dương n

Tách tổng

Cơ sở: Để chứng minh A(n) chia hết cho p, ta biến đổi A(n) thành tổng nhiều hạng tử chứng minh hạng tử chia hết chop Ta sử dụng số đẳng thức áp dụng vào chia hết, ví dụ như:

Choa, b số thực vàn số nguyên dương Khi ta có

an−bn= (a−b)(an−1+an−2b+· · ·+abn−2+bn−1) Ta có hệ là:

Hệ 3.2– Nếu a−b6= an−bn chia hết choa−b Hệ 3.3– Nếu a+b6= n lẻ an+bn chia hết choa+b Hệ 3.4– Nếua+b6= vànchẵn thìan−bn chia hết choa+b

Ví dụ 3.12 Chứng minh ax2+bx+c∈Z, ∀x∈Z

(49)

3.2 Phương pháp giải toán chia hết 41

Lời giải Phân tích

ax2+bx+c=ax2−ax+ (a+b)x+c

= 2a.x(x−1)

2 + (a+b)x+c∈Z, ∀x∈Z

Ví dụ 3.13 Chứng minh 6|(a3+ 5a)∀a∈N

Lời giải Phân tícha3+5a= (a3−a)+6a Hiển nhiên vì6|n3−n

(chứng minh ví dụ Equation 4.27)

Nhận xét Từ ví dụ Equation 4.27ta đưa tốn sau, chứng minh cách vận dụng phương pháp tách tổng: Bài toán 3.1 Cho m, n∈Z Chứng minh 6|m2n2(m−n)

Bài toán 3.2 Cho a, b, c∈ Z Chứng minh |(a3+b3+c3)

và 6|(a+b+c)

Bài toán 3.3 Cho a∈Z Chứng minh a +

a2

2 +

a3

6 ∈Z

Bài toán 3.4 Viết số 20112012 thành tổng số nguyên dương Đem tổng lập phương tất số hạng chia cho dư bao

nhiêu ?

Ví dụ 3.14 Cho m, n số nguyên thỏa mãn:

m n = 1−

1 +

1 3−

1

4 +· · · − 1334+

1 1335

Chứng minh 2003|m

(50)

42 3.2 Phương pháp giải toán chia hết

Lời giải Để ý rằng2003là số nguyên tố Ta có

m

n = 1−

1 +

1 −

1

4 +· · · − 1334+ 1335 =

1 +1 +

1

3 +· · ·+ 1335 −2 + +

6 +· · ·+ 1334

=

1 +1 +

1

3 +· · ·+ 1335

1 +1 2+

1

3 +· · ·+ 667 = 668+

669+· · ·+ 1335 = 668+ 1335 + 669+ 1334 +· · ·+ 1001 + 1002 = 2003 668.1335+

1

669.1334+· · ·+ 1001.1002

= 2003.p q

Ở đâyplà số ngun cịn q= 668·669· · ·1335 Vì 2003nguyên tố nên (q,2003) =

Do từ (∗) suy 2003pn=mq

Vìp, n nguyên nên suy 2003|mq mà(q,2003) = 1nên 2003|m

Ví dụ 3.15 Chứng minh với số tự nhiên nthì A= 2005n+ 60n−1897n−168n chia hết cho2004 Lời giải Ta có 2004 = 12×167 Vì (12,167) = 1nên để chứng minh

A chia hết cho2004ta chứng minh A chia hết cho 12và 167

Áp dụng tính chất an−bn chia hết cho a−b với n tự nhiên

a−b6= 0suy ra2005n−1897nchia hết cho2005−1897 = 108 = 12×9, hay 2005n−1897n chia hết cho 12 Tương tự thì 168n−60n chia hết cho12 VậyA chia hết cho 12

Tiếp tục phân tích

A= (2005n−168n)−(1897n−60n)

(51)

3.2 Phương pháp giải toán chia hết 43

Ví dụ 3.16 (Đề thi tuyển sinh ĐHKHTN-ĐHQG Hà Nội, vòng 1, năm 2007-2008) Choa, b hai số nguyên dương vàa+ 1, b+ 2007đều chia hết cho Chứng minh 4a+a+b chia hết cho6 Lời giải Phân tích

4a+a+b= (4a+ 2) + (a+ 1) + (b+ 2007)−2010 4a+ = 4a−1 + = (4−1)(4a−1+· · ·1) +

Như vậy3|4a+ Do đó4a+a+blà tổng số nguyên dương chia hết cho 6nên 4a+a+b chia hết cho 6. Bài tập đề nghị

Bài1 Đưa mở rộng từ tập đề nghị phương pháp phân tích thành tích thành toán vận dụng phương pháp tách tổng (giống cách mở rộng ví dụ 1.9)

Bài2 (Hungary MO 1947) Chứng minh 46n+ 296.13n chia hết cho1947với số tự nhiên nlẻ

Bài3 Chứng minh rằng20n+ 16n−3n−1chia hết cho 323với số tự nhiênn chẵn

Bài4 Chứng minh rằng2903n−803n−464n+ 261nchia hết cho1897 với số tự nhiênn

Bài5 Chứng minh với số nguyên n > ta có nn+ 5n2 − 11n+ 5chia hết cho(n−1)2

Bài6 (HSG Tp Hà Nội, vòng 2, 1998) Chứng minh rằng1997|m

vớim, n∈Nthỏa mãn

m n = 1−

1 +

1 −

1

4 +· · ·+ 1329−

1 1330+

1 1331 Bài7 Chứng minh 32n+1+ 2n+2 chia hết cho với mọin∈N

(52)

44 3.2 Phương pháp giải toán chia hết

Bài8 Chứng minh 20032005+ 20172015 chia hết cho 12

Bài9 Cho p số tự nhiên lẻ số nguyên a, b, c, d, e thỏa mãn

a+b+c+d+evà a2+b2+c2+d2+e2 đều chia hết cho p. Chứng minh sốa5+b5+c5+d5+e5−5abcde chia hết chop

Bài10 (Canada Training for IMO 1987) Kí hiệu:

1·3·5· · ·(2n−1) = (2n−1)!! 2·4·6· · ·(2n) = (2n)!!

Chứng minh (1985)!! + (1986)!!chia hết cho 1987

Bài11 Chứng minh số22225555+ 55552222 chia hết cho Bài12 Cho k số nguyên dương cho sốp = 3k+ 1là số nguyên

tố

1 1·2+

1

3·4+· · ·+

(2k−1)2k = m

n

với hai số nguyên dương nguyên tố m n.Chứng minh mchia hết cho p

(Tạp chí Mathematics Reflections, đăng T.Andreescu)

3.2.4 Xét đồng dư

Định nghĩa số tính chất

Định nghĩa 3.2 Choa, blà số nguyên vànlà số nguyên dương Ta nói, ađồng dư vớibtheo modun nvà kí hiệu a≡b (modn) nếua

bcó số dư chia chon Như vậya≡n (modn) ⇐⇒ n|(a−b) Ví dụ: 2012≡2 (mod 5) Tính chất(bạn đọc tự chứng minh)

Choa, b, c, d, n số nguyên Tính chất 3.10–

a≡a (mod n),

a≡b (modn)⇔b≡a (mod n),

a≡b (modn), b≡c (modn)⇒a≡c (mod n)

(53)

3.2 Phương pháp giải toán chia hết 45

Tính chất 3.11–

(

a≡b (modn)

c≡d (mod n) ⇒

(

a±c≡b±d (modn)

ac≡bd (modn) Tính chất 3.12– a≡b (mod n)⇒ak≡bk (modn), ∀k≥1

Tính chất 3.13– a≡b (mod n)⇒ac≡bc (modmc), c >0 Tính chất 3.14– (a+b)n≡bn (moda), (a >0)

Tính chất 3.15– Nếu d ước chung dương a, b m a ≡ b

(modm) a

d ≡ b d (mod

m d)

Tính chất 3.16– a≡b (mod m),clà ước chung củaavàb,(c, m) = a

c ≡ b

c (mod m)

Phương pháp đồng dư thức để giải tốn chia hết Cơ sở:Sử dụng tính chất định nghĩa để giải toán chia hết

Ví dụ 3.17 Chứng minh với số tự nhiên n 7|8n+ Lời giải Ta có8n≡1 (mod 7) =⇒ 8n+ 6≡7≡0 (mod 7)

Ví dụ 3.18 Chứng minh 19|7·52n+ 12·6n với số nguyên

dương n

Lời giải Ta có52 = 25≡6 (mod 19) =⇒ (52)n≡6n (mod 19) =⇒ 7·52n≡7·6n (mod 19) =⇒ 7·52n+ 12·6n≡19·6n≡0 (mod 19)

Ví dụ 3.19 Viết liên tiếp số 111,112,· · ·,888 để số A = 111112· · ·888 Chứng minh 1998|A

(54)

46 3.2 Phương pháp giải toán chia hết

Lời giải Ta thấyA chẵn nên2|A Mặt khác

A= 111·1000777+ 112·1000776+· · ·+ 888

Do1000k≡1 (mod 999), ∀k∈Nnên

A≡111 + 112 +· · ·+ 888≡0 (mod 999)

Suy ra999|A, và(999,2) = 1nên 1998|A

Ví dụ 3.20 Chứng minh 7|55552222+ 22225555. 4 Lời giải Ta có

2222≡ −4 (mod 7) =⇒ 22225555 ≡(−4)5555 (mod 7) 5555≡4 (mod 7) =⇒ 55552222 ≡4 (mod 7)

=⇒ 55552222+ 22225555≡ −45555+ 42222 (mod 7) Lại có

−45555+ 42222 =−42222 43333−1

=−42222 641111−1 Và64≡1 (mod 7) =⇒ 641111−1≡0 (mod 7)

Do 7|55552222+ 22225555

Bài tập đề nghị

Bài1 Một số tập phương pháp phân tích giải phương pháp đồng dư thức

Bài2 Chứng minh 333555777 + 777555333 chia hết cho 10 Bài3 Chứng minh số 11101967−1 chia hết cho 101968

Bài4 Cho9|a3+b3+c3, ∀a, b, c∈Z Chứng minh rằng3|a·b·c

(55)

3.2 Phương pháp giải toán chia hết 47

Bài6 Chứng minh 9n+ 1không chia hết cho100, ∀n∈N

Bài7 Chứng minh với số nguyên không âm n 25n+3+ 5n·3n+1 chia hết cho17.

Bài8 Tìmn∈Nsao cho 2n3+ 3n= 19851986

Bài9 Viết liên tiếp 2000 số 1999 ta sốX = 19991999· · ·1999

Tìm số dư phép chia X cho10001 Bài10 Chứng minh 100|7777

7

−777

Bài11 Chob2−4acvàb2+ 4aclà hai số phương vớia, b, c∈N Chứng minh 30|abc

3.2.5 Quy nạp

Cơ sở :Để chứng minh mệnh đề với số tự nhiên n≥p, ta làm sau:

• Kiểm tra mệnh đề vớin=p

• Giả sử mệnh đề vớin=k Ta chứng minh mệnh đề vớin=k+

Ví dụ 3.21 Chứng minh A= 4n+ 15−1 chia hết cho với

n∈N∗

Lời giải Vớin= =⇒ A= 18chia hết cho

Giả sử tốn vớin=k Khi đó9|4k+15k−1, hay4k+15k−1 = 9q với q∈N∗ Suy 4k= 9q−15k+

Ta chứng minh toán vớin=k+1, tức9|4k+1+15(k+1)−1 Thật vậy:

4k+1+ 15(k+ 1)−1 = 4·4k+ 15k+ 14

= (9q−15k+ 1) + 15k+ 14 = 36q−45k+ 18

chia hết cho Ta có đpcm

(56)

48 3.2 Phương pháp giải tốn chia hết

Ví dụ 3.22 (HSG TQ 1978)Chứng minh số tạo bởi3nchữ số giống chia hết cho 3n với 1≤n, n∈N Lời giải Vớin= 1, toán hiển nhiên

Giả sử toán với n=k, tức3k|aa· · ·a

| {z }

3nsốa

Vớin=k+ 1ta có:

aa· · ·a

| {z }

3k+1

=aa· · ·a

| {z }

3k

aa· · ·a

| {z }

3k

aa· · ·a

| {z }

3k

=aa· · ·a

| {z }

3k

×1 00· · ·0

| {z }

3k−1

00· · ·0

| {z }

3k−1

1

chia hết cho 3k+1 Ta có đpcm.

Ví dụ 3.23 Chứng minh với n∈N∗, k là số tự nhiên lẻ thì

2n+2 |k2n−1

Lời giải Vớin= 1thì k2n−1 =k2−1 = (k+ 1)(k−1) Dok lẻ,nên đặtk= 2m+ 1vớim∈N∗, đó(k+ 1)(k−1) = 4k(k+ 1)chia

hết cho23=

Giả sử toán vớin=p, tức2p+2 |k2p−1hay k2p =q·2p+2+ vớiq ∈N∗.

Ta chứng minh toán vớin=p+ Thật

A=k2p+1−1 =k2·2p−1 = k2p2−1 = k2p−1 k2p+

=q·2p+2· +q·2p+2

=q·2p+3· +q·2p+1

(57)

3.2 Phương pháp giải toán chia hết 49

Bài tập đề nghị

Bài1 Một số toán phương pháp nêu giải phương pháp quy nạp

Bài2 Chứng minh 255|16n−15n−1với n∈N Bài3 Chứng minh 64|32n+3+ 40n−27 vớin∈N Bài4 Chứng minh 16|32n+2+ 8n−9với n∈N Bài5 Chứng minh 676|33n+3−16n−27 vớin∈

N, n≥1

Bài6 Chứng minh 700|292n−140n−1 vớin∈N

Bài7 Chứng minh 270|2002n−138n−1 vớin∈N

Bài8 Chứng minh 22|324n+1+ 234n+1+ 5vớin∈N

Bài9 Chứng minh số23n+ chia hết cho 3n không chia hết cho3n+1 vớin∈N

Bài10 Chứng minh số20012n−1chia hết cho2n+4 không chia hết cho 2n+5 vớin∈N

Bài11 Chứng minh với số tự nhiênn≥2, tồn số tự nhiênm cho 3n|(m3+ 17), nhưng3n+1-(m3+ 17)

Bài12 Có tồn hay không số nguyên dương bội của2007 có bốn chữ số tận là2008

Bài13 Chứng minh tồn số có 2011chữ số gồm tồn chữ số1và 2sao cho số chia hết cho 22011

Bài14 Tìm phần dư chia 32n cho 2n+3, đó n là số nguyên dương

Bài15 Chon∈N, n≥2 Đặt A= 77

(lũy thừa nlần) Chứng minh rằngAn+ 17 chia hết cho20

(58)

50 3.2 Phương pháp giải tốn chia hết

3.2.6 Sử dụng ngun lí Dirichlet

Nội dung:Nhốt5con thỏ vào3chuồng tồn chuồng chứa 2con

Định lý 3.3– Nhốt m=nk+ thỏ vào kchuồng (k < n) tồn chuồng chứa nhấtn+ 1con thỏ Chứng minh Giả sử khơng có chuồng chứa nhấtn+ 1con thỏ, chuồng chứa nhiều nhấtncon thỏ, nênkchuồng chứa nhiều nhấtkncon thỏ, mâu thuẫn với số thỏ nk+

Định lý 3.4 (Áp dụng vào số học)– Trong m = nk + số có n+ số chia cho k có số dư Tuy nguyên lý phát biểu đơn giản lại có ứng dụng bất ngờ, thú vị Bài viết xin nêu số ứng dụng nguyên lí việc giải tốn chia hết

Ví dụ 3.24 Chứng minh ln tồn số có dạng 20112011· · ·201100· · ·0

chia hết cho2012

Lời giải Lấy2013số có dạng

2011; 20112011, · · · , 20112011· · ·2011

| {z }

2012số 2011

Lấy2013số chia cho2012 Theo nguyên lí Dirichlet tồn hai số có số dư chia cho2012

Giả sử hai số 20112011· · ·2011

| {z }

m số 2011

và 20112011· · ·2011

| {z }

nsố 2011

(m > n >

0)

=⇒ 2012|20112011· · ·2011

| {z }

m số2011

−20112011· · ·2011

| {z }

(59)

3.2 Phương pháp giải toán chia hết 51

=⇒ 2012|20112011· · ·2011

| {z }

m−nsố 2011

00· · ·00

| {z }

nsố 2011

Vậy tồn số thỏa mãn đề

Ví dụ 3.25 Chứng minh 101 số ngun tìm hai số có2 chữ số tận giống Lời giải Lấy 101 số nguyên cho chia cho 100 theo ngun lí Dirichlet tồn hai số có số dư chia cho 100 Suy 101số nguyên cho tồn hai số có chữ số tận giống

Ví dụ 3.26 (Tuyển sinh 10 chuyên ĐHSPHN, 1993) Cho5số nguyên phân biệt tùy ý a1, a2, a3, a4, a5 Chứng minh tích

P = (a1−a2)(a1−a3)(a1−a4)(a1−a5)(a2−a3)×

×(a2−a4)(a2−a5)(a3−a4)(a3−a5)(a4−a5)

chia hết cho288

Lời giải Phân tích288 = 25·32

1 Chứng minh | P: Theo ngun lí Dirichlet số

a1, a2, a3 có hai số có hiệu chia hết cho Khơng tính tổng qt, giả sử: 3|a1−a2 Xét 4số a2, a3, a4, a5 có hai số có hiệu chia hết cho Như P có hai hiệu khác chia hết cho3, tức9|p

2 Chứng minh32|P: Theo ngun lí Dirichlet tỏng5số cho tồn nhất3 số có tính chẵn lẻ Chỉ có hai khả sau xảy ra:

• Nếu có nhất4số có tính chẵn lẻ, từ bốn số lập thành sáu hiệu khác chia hết cho2 Do 32|P

(60)

52 3.2 Phương pháp giải tốn chia hết

• Nếu có3số có tính chẵn lẻ Khơng tính tổng qt, giả sử ba số a1, a2, a3 Khi a4, a5 tính chẵn lẻ lại khác tính chẵn lẻ a1, a2, a3 Khi hiệu sau chia hết cho2:a1−a2, a1−a3, a2−a3, a4−a5 Mặt khác, trong5số cho có hai hiệu chia hết cho 4, hiệua1−a2, a1−a3, a2−a3, a4−a5 có hiệu chia hết cho4 Vậy 32|P

Ta có đpcm

Ví dụ 3.27 Cho 2012số tự nhiên a1, a2,· · ·, a2012 Chứng minh tồn số chia hết cho2012hoặc tổng số số chia hết cho

2012

Lời giải Xét2012số

S1 =a2

S2 =a1+a2 · · ·

S2012 =a1+a2+· · ·+a2012

Trường hợp 1: Nếu tồn số Si (i = 1,2,· · · ,2012) chia hết cho 2012thì tốn chứng minh xong

Trường hợp 2: Nếu 2012 -Si với i = 1,2,· · · ,2012 Đem 2012 số chia cho2012 nhận được2012 số dư Các số dư nhận giá trị thuộc tập {1; 2;· · ·; 2011} Vì có 2012 số dư mà có 2011 giá trị nên theo ngun lí Dirichlet chắn có hai số dư Gỉa sử gọi hai số Sm Sn có số dư chia cho2012 (m, n∈N, 1≤n < m≤2012)thì hiệu

Sm−Sn=an+1+an+2+· · ·+am

(61)

3.2 Phương pháp giải toán chia hết 53

Nhận xét Ta rút toán tổng quát toán mở rộng sau:

Bài toán 3.5 (Bài toán tổng quát) Cho n số a1, a2,· · ·, an Chứng minh trongnsố tồn số chia hết chonhoặc tổng

số số chia hết cho n

Bài tốn 3.6 (Bài tốn mở rộng) (Tạp chí Tốn Tuổi Thơ số 115) Cho nlà số chuyên dương nsố nguyên dương a1, a2,· · ·, an có tổng 2n−1 Chứng minh tồn số số n số

cho có tổng n

Bài tập đề nghị

Bài1 Chứng minh có vơ số số chia hết cho 201311356 mà biểu diễn thập phân số khơng có chữ số0,1,2,3 Bài2 (HSG Hà Nội, 2006) Chứng minh tồn số tự nhiên

n6= thỏa mãn 313579|(13579n−1)

Bài3 Chứng minh 52 số ngun dương ln ln tìm hai số có tổng hiệu chia hết cho100

Bài4 Cho10 số nguyên dương a1, a2,· · ·, a10 Chứng minh tồn số ci ∈ {0,−1,1}, (i= 1,· · ·10)không đồng thời 0sao cho

A=c1a1+c2a2+· · ·+c10a10 chia hết cho 1032

Bài5 Chứng minh tồn số tự nhiên ksao cho 2002k−1 chia hết cho200310

Bài6 Biết ba sốa, a+k, a+ 2kđều số nguyên tố lớn Chứng minh đók chia hết cho6

(62)

54 3.2 Phương pháp giải toán chia hết

3.2.7 Phản chứng

Cơ sở: Để chứng minhp-A(n), ta làm sau:

• Giả sử ngược lạip|A(n) • Chứng minh điều ngược lại sai

Ví dụ 3.28 Chứng minh với số ngunnthìn2+n+ không

chia hết cho9

Lời giải Giả sử9|(n2+n+ 1) Khi đón2+n+ = (n+ 2)(n−1) + 3 chia hết cho Suy |n+ 3| n−1 Như (n+ 2)(n−1) chia hết cho 9, tứcn2+n+ 1chia9 dư 3, mâu thuẫn Ta có đpcm

Nhận xét Bài tốn giải theo phương pháp xét số dư

Ví dụ 3.29 Giả sử p = k.2t+ số nguyên tố lẻ, t số nguyên dương k số tự nhiên lẻ Giả thiết x y số tự nhiên mà

p|x2t +y2t

Chứng minh x vày đồng thời chia hết cho

p

Lời giải Giả sử trái lạip-x, suy p-y

Dop số nguyên tố nên theo định lý Fermat nhỏ ta có

xp−1 ≡1 (modp)

yp−1≡1 (modp) Theo giả thiết thìp−1 =k.2t, đó

xk.2t ≡1 (modp)

yk.2t ≡1 (modp) Từ ta có

xk.2t +yk.2t ≡2 (modp) (i) Theo giả thiết

(63)

3.2 Phương pháp giải toán chia hết 55

Doklẻ nên

xk.2t +yk.2t =

x2t

k

+

y2t

k

x2t+y2t

⇒xk.2t+yk.2t≡0 (modp) (ii)

Từ(i)và(ii)suy điều mâu thuẫn Vậy giả thiết phản chứng sai Do

đó x, yđồng thời chia hết cho p

Bài tập đề nghị

Bài1 Chứng minhn2+n+ 2không chia hết cho15 với mọin∈Z

Bài2 Chứng minhn2+ 3n+ 5 không chia hết cho121với mọin∈

N

Bài3 Chứng minh 9n3 + 9n2+ 3n−16 không chia hết cho 343 với n∈N

Bài4 Chứng minh 4n3 −6n2+ 3n+ 37 không chia hết cho 125 với n∈N

Bài5 Chứng minhn3+ 3n−38không chia hết cho49với mọin∈N

(64)(65)

Chương

4

Phương trình nghiệm

ngun

4.1 Xét tính chia hết 57 4.2 Sử dụng bất đẳng thức 74

4.3 Nguyên tắc cực hạn, lùi vô hạn 86

Trần Nguyễn Thiết Quân(L Lawliet) Phạm Quang Toàn(Phạm Quang Toàn)

Trong chương trình THCS THPT phương trình nghiệm ngun ln đề tài hay khó học sinh Các toán nghiệm nguyên thường xuyên xuất kì thi lớn, nhỏ, ngồi nước Trong viết tơi muốn đề cập đến vấn đề nghiệm nguyên (các dạng, phương pháp giải) không nghiên cứu sâu sắc Tơi khơng đề cập tới phương trình Pell, phương trình Pythagore, phương trình Fermat có nhiều sách, chun đề khác

4.1

Xét tính chia hết

4.1.1 Phát tính chia hết ẩn Ví dụ 4.1 Giải phương trình nghiệm nguyên

13x+ 5y= 175 (4.1)

(66)

58 4.1 Xét tính chia hết

Lời giải Giả sửx, ylà số nguyên thỏa mãn phương trình (4.1) Ta thấy175và5yđều chia hết cho5nên13x 5⇒x 5(do GCD(13; 5) = 1) Đặtx= 5t(t∈Z) Thay vào phương trình (4.1), ta

13.5t+ 5y= 175⇔13t+y= 35⇔y= 35−13t

Do đó, phương trình (4.1) có vơ số nghiệm nguyên biểu diễn dạng

(x;y) = (5t; 35−13t),(t∈Z) Bài tập đề nghị

Bài1 Giải phương trình nghiệm nguyên12x−19y= 285 Bài2 Giải phương trình nghiệm nguyên7x+ 13y= 65 Bài3 Giải phương trình nghiệm nguyên5x+ 7y= 112 4.1.2 Đưa phương trình ước số

Ví dụ 4.2 Tìm nghiệm nguyên phương trình

3xy+ 6x+y−52 = (4.2) Lời giải Nhận xét Đối với phương trình này, ta áp dụng phương pháp phát tính chia hết, ta phải giải nào?

Ta giải sau:

(4.2)⇔3xy+y+ 6x+ 2−54 = ⇔y(3x+ 1) + (3x+ 1)−54 = ⇔(3x+ 1) (y+ 2) = 54

Như vậy, đến ta cóx vày nguyên nên3x+ vày+ phải ước của54 Nhưng ta phải xét đến 10trường hợp sao? Vì:

4 = 1.54 = 2.27 = 3.18 = 6.9

(67)

4.1 Xét tính chia hết 59

Có cách khác khơng? Câu trả lời có! Nếu ta để ý chút đến thừa số 3x+ 1, biểu thức chia cho dư1 với x nguyên Với lập luận trên, ta được:

  

3x+ =

y+ = 54 ⇔

x=

y = 52

3x+ =−2

y+ =−54 ⇔

x=−1

y=−56 Ví dụ 4.3 Giải phương trình nghiệm nguyên sau:

2x+ 5y+ 3xy= (4.3)

Lời giải Ta có

(4.3)⇔x(2 + 3y) + 5y= ⇔3x(2 + 3y) + 15y= 24 ⇔3x(2 + 3y) + 5(2 + 3y) = 34 ⇔(3x+ 5)(3y+ 3) = 34

Đến phân tích 34 = 1·34 = 2·17 xét trường hợp Chú ý rằng3x+ 5,3y+ 2là hai số nguyên chia 3dư 2, vận dụng điều ta

có thể giảm bớt số trường hợp cần xét

Ví dụ 4.4 Giải phương trình nghiệm ngun

x2−y2 = 2011 (4.4)

Lời giải (4.4) ⇔(x−y)(x+y) = 2011 Vì 2011là số nguyên tố nên ước nguyên 2011 ±1,±2011 Từ suy nghiệm (x;y) là(1006; 1005); (1006;−1005); (−1006;−1005); (−1006; 1005)

Ví dụ 4.5 Tìm số ngun x, y thoả mãn điều kiện

x2+y2= (x−y)(xy+ 2) + (4.5)

(68)

60 4.1 Xét tính chia hết

Lời giải Đặta=x−y, b=xy Khi (4.5) trở thành

a2+ 2b=a(b+ 2) + 9⇔(a−2)(a−b) = (4.6) Vìx, y∈Znêna, , a−2, a−bđều số nguyên Từ (4.6) ta có trường hợp sau:

(

a−2 =

a−b= ⇔

(

a= 11

b= 10 ⇔

(

x−y= 11

xy= 10 (4.7)

(

a−2 =

a−b= ⇔

(

a=

b= ⇔

(

x−y=

xy= (4.8)

(

a−2 =

a−b= ⇔

(

a=

b=−6 ⇔

(

x−y=

xy=−6 (4.9)

(

a−2 =−1

a−b=−9 ⇔

(

a=

b= 10 ⇔

(

x−y=

xy= 10 (4.10)

(

a−2 =−3

a−b=−3 ⇔

(

a=−1

b= ⇔

(

x−y=−1

xy= (4.11)

(

a−2 =−3

a−b=−3 ⇔

(

a=−1

b= ⇔

(

x−y=−1

xy= (4.12)

Dễ thấy hệ (4.7),(4.8),(4.10) khơng có nghiệm ngun, hệ (4.9) vơ nghiệm, hệ (4.11) có hai nghiệm nguyên (1; 2) (−2;−1), hệ (4.12) có hai nghiệm nguyên (−1; 6)và(−6; 1)

Tóm lại phương trình (4.5) có cặp nghiệm ngun (x;y) (1; 2); (−2;−1);(−1; 6);(−6; 1)

Ví dụ 4.6 Tìm nghiệm nguyên phương trình:

x2+

y2+

(69)

4.1 Xét tính chia hết 61

Lời giải Phương trình (4.13) tương đương với:

x2y2+x2+y2+ + 2x−2x2y−2y+ 2xy2 = + 4xy

⇔ (x2+ 2x+ 1)y2−2(x2+ 2x+ 1)y+ (x2+ 2x+ 1) = ⇔ (x+ 1)2(y−1)2 =

(x+ 1)(y−1) = (x+ 1)(y−1) =−2

Với(x+ 1)(y−1) = 2màx, y∈Znên ta có trường hợp sau: •

x+ =

y−1 = ⇔

x=

y= •

x+ =

y−1 = ⇔

x=

y= •

x+ =−2

y−1 =−1 ⇔

x=−3

y= •

x+ =−1

y−1 =−2 ⇔

x=−2

y=−1 Với(x+ 1)(y−1) =−2 , tương tự ta suy được:

x+ =−1

y−1 = ⇔

x=−2

y = •

x+ =

y−1 =−2 ⇔

x=

y=−1 •

x+ =

y−1 =−1 ⇔

x=

y= •

x+ =−2

y−1 = ⇔

x=−3

y = Vậy phương trình cho có cặp nghiệm ngun:

(x;y) ={(0; 3); (1; 2); (−3; 0); (−2;−1); (−2; 3); (0;−1); (1; 0); (−3; 2)} Ví dụ 4.7 Tìm nghiệm nguyên phương trình

x6+ 3x3+ =y4 (4.14)

(70)

62 4.1 Xét tính chia hết

Lời giải Nhân hai vế phương trình (4.14) cho 4, ta được: 4x6+ 12x3+ = 4y4

⇔ (4x6+ 12x3+ 9)−4y4 = 5 ⇔ (2x3+ 3)2−4y4 =

⇔ (2x3−2y2+ 3)(2x3+ 2y2+ 3) =

Với lưu ý rằng5 = 1.5 = 5.1 = (−1).(−5) = (−5).(−1)vàx, y∈Znên ta suy trường hợp sau:

2x3−2y2+ = 2x3+ 2y2+ = ⇔

x3−y2=−1

x3+y2= ⇔

x3 =

y2=

⇔       

x=

y=

x=

y=−1 •

2x3−2y2+ =−1 2x3+ 2y2+ =−5 ⇔

x3−y2 =−2

x3+y2 =−4 ⇔

x3 =−3

y2 =−1 (loại) •

2x3−2y2+ = 2x3+ 2y2+ = ⇔

x3−y2=

x3+y2=−1 ⇔

x3 =

y2=−1 (loại) •

2x3−2y2+ =−5 2x3+ 2y2+ =−1 ⇔

x3−y2 =−4

x3+y2 =−2 ⇔

x3 =−3

y2 = (loại) Vậy phương trình cho có cặp nghiệm nguyên:

(x;y) ={(0; 1); (0;−1)}

Nhận xét Bài tốn giải phương pháp kẹp

Ví dụ 4.8 Giải phương trình nghiệm nguyên dương: x + y = p (4.15)

(71)

4.1 Xét tính chia hết 63

Lời giải

(4.15)⇔xy =px+py⇒(x−y)(y−p) =p2

Vìplà số nguyên tố nên ước số nguyên củap2chỉ là±1,±p,±p2. Thử với ước ta dễ tìm kết Phần trình bày

xin dành cho bạn đọc

Nhận xét Phương pháp cần hai bước chính: Phân tích thành ước số xét trường hợp để tìm kết Hai bước nói khơng q khó bạn đọc, xin nói số lưu ý thêm bước xét trường hợp Trong số toán, số nguyên vế phải sau phân tích số có nhiều ước, địi hỏi xét trường hợp tính tốn nhiều Một câu hỏi đặt là: Làm để giảm số trường hợp bị xét đây? Và để trả lời câu hỏi đó, ta tham khảo ví dụ

Ví dụ 4.9 Tìm nghiệm ngun phương trình:

x2+ 12x=y2 (4.16)

Lời giải (thơng thường) Phương trình (4.16) cho tương đương với: (x+ 6)2−y2 = 36⇔(x+ +y)(x+ 6−y) = 36

Suy x+y+ 6, x+ 6−y ước 36 Mà số 36 có tất 18 ước nên ta phải xét18 trường hợp tương ứng với

x+ +y ∈ {±1;±2;±3;±4;±6;±9;±12;±18;±36} Kết ta tìm cặp nghiệm nguyên(x;y)

(0; 0); (−12; 0); (−16; 8); (−16;−8); (4; 8); (4;−8)

Nhận xét Đúng vấn đề mà ta nêu trên, số ước nhiều để xét Cho nên ta có nhận xét sau đề thực thao tác "siêu phàm" chuyển từ số18 xuống còn2!

(72)

64 4.1 Xét tính chia hết

Vì y có số mũ chẵn phương trình nên giả sửy ≥0 Khi x+ 6−y≤x+ +y, ta loại tám trường hợp lại trường hợp sau:

(

x+ +y=

x+ 6−y= ,

(

x+ +y =−9

x+ 6−y =−4 ,

(

x+y+ =−1

x+y−6 =−36 ,

(

x+y+ = 36

x−y+ = ,

(

x+y+ =−2

x−y+ =−18 ,

(

x+y+ = 18

x−y+ = ,

(

x+y+ =−3

x−y+ =−12 ,

(

x+y+ = 12

x−y+ = ,

(

x+y+ =−6

x−y+ =−6 ,

(

x+y+ =

x+y−6 =

Bây ta có 10 trường hợp, ta tiếp tục lược bỏ Nhận thấy (x+y+ 6)−(x+ 6−y) = 2y nên x+ 6−y vàx+ +y có tính chẵn lẻ, ta loại thêm6 trường hợp,

(

x+y+ = 18

x+y−6 = ,

(

x+y+ =−2

x+y−6 =−18 ,

(

x+y+ =−6

x−y+ =−6 ,

(

x+y+ =

x+y−6 =

Tiếp tục xét hai phương trình

(

x+y+ =−6

x−y+ =−6

(

x+y+ =

x+y−6 = , hai phương trình tìm đượcy = Vậy không để đơn giản hơn, ta xét y= từ đầu Phương trình có dạngx(x+ 12) =y2, xét hai khả năng:

• Nếuy= thìx= hoặcx=−12

• Nếuy6= 0thìx+6+y > x+6−y, áp dụng hai nhận xét ta có hai trường hợp:

(

x+y+ =−2

x−y+ =−18

(

x+y+ = 18

x−y+ =

(73)

4.1 Xét tính chia hết 65

Phương trình cho có nghiệm nguyên

(x;y) = (−16; 8),(0; 0),(−12; 0),(−16; 8),(4; 8),(4;−8)

Nhận xét Như toán ngắn gọn, xác nhờ linh hoạt việc xét tính chẵn lẻ, giới hạn hai số để giảm số trường hợp cần xét Ngoài cách đánh giá ta cịn áp dụng xét số dư vế để đánh giá (đây phương pháp giải phương trình nghiệm nguyên)

Bài tập đề nghị

Bài1 Thử biến đổi tốn giải phương trình nghiệm nguyên phương pháp Biểu thị ẩn theo ẩn lại phương pháp đưa ước số

Bài2 Tìm độ dài cạnh tam giác vng cho tích hai cạnh huyền gấp ba lần chu vi tam giác

Bài3 Giải phương trình nghiệm nguyênx−y+ 2xy = Bài4 Giải phương trình nghiệm nguyên2x+ 5y+ 2xy=

Bài5 (Thi HSG lớp tỉnh Quảng Ngãi năm 2011-2012) Giải phương trình nghiệm nguyên6x+ 5y+ 18 = 2xy

Bài6 Tìm nghiệm nguyên(xy−7)2=x2+y2

Bài7 Tìmx, y∈Z thỏa mãn2x2−2xy = 5x−y−19

Bài8 Tìm nghiệm nguyên phương trìnhx2+6xy+8y2+3x+6y=

Bài9 Tìm nghiệm nguyên dương phương trìnhx3−y3 =xy+ 61 Bài10 Tìm nghiệm nguyên phương trình4x2y2 = 22 +x(1 +x) +

y(1 +y)

Bài11 Giải phương trình nghiệm nguyênx(x+ 1)(x+ 7)(x+ 8) =y2

(74)

66 4.1 Xét tính chia hết

Bài12 Tìm nghiệm nguyên dương phương trình 6x3−xy(11x+ 3y) + 2y3= 6 (Tạp chí TTT2 số 106).

Bài13 Tìm nghiệm nguyên dương phương trìnhx(x+ 2y)3−y(y+ 2x)3= 27(tạp chí THTT số 398)

Bài14 Tìm nghiệm nguyên phương trình√9x2+ 16x+ 96 = 3x− 16y−24

Bài15 Tìm nghiệm nguyên dương phương trình

2 +

s

x+ 2+

r

x+1 =y

Bài16 Tìm số nguyênxđể x2−4x−52 số phương

Bài17 Giải phương trình nghiệm nguyênx2+ 2y2+ 3xy−2x−y = Bài18 Giải phương trình nghiệm nguyênx2+ 3xy−y2+ 2x−3y = Bài19 Giải phương trình nghiệm nguyên2x2+ 3y2+xy−3x−3 =y Bài20 (Tuyển sinh vào lớp 10 THPT chuyên trường KHTN Hà Nội năm học 2012-2013) Tìm tất cặp số nguyên x, y thỏa mãn đẳng thức(x+y+ 1)(xy+x+y) = + 2(x+y)

Bài21 Giải phương trình nghiệm nguyênx4−2y4−x2y2−4x2−7y2− =

(Thi HSG lớp tỉnh Hưng Yên năm 2011-2012)

Bài22 (Romanian 1999)Chứng minh phương trình sau khơng có nghiệm ngun

(75)

4.1 Xét tính chia hết 67

4.1.3 Biểu thị ẩn theo ẩn lại sử dụng tính chia hết

Ví dụ 4.10 Tìm nghiệm nguyên phương trình

2x−xy+ = (4.17) Lời giải Nhận xét Ở phương trình ta áp dụng cách biết, ta phải làm sao? Chú ý xíu ta thấy biểu diễn y theo x vận dụng kiến thức tìm giá trị nguyên lớp tìm nghiệm nguyên phương trình, thử làm theo ý tưởng xem

(4.17)⇔xy= 2x+

Nếux= phương trình (4.17) cho vơ nghiệm nguyêny Nếux6=

(4.17)⇔y= 2x+

x = +

3

x

Như muốn y nguyên ta cần

x nguyên hay nói cách khác x

ước của3 Với giá trị nguyên x ta tìm giá trịy nguyên Từ đó, ta có nghiệm (4.17)

(x;y) = (−3; 1); (−1;−1); (1; 5); (3; 3)

Ví dụ 4.11 (Thi HSG lớp Quảng Ngãi năm 2011-2012) Tìm số nguyên dương x, y cho

6x+ 5y+ 18 = 2xy (4.18) Nhận xét Hướng phân tích định hướng lời giải Đã xác định phương pháp dạng ta biểu diễn ẩn x theo y Khơng khó để viết thànhx= −5y−18

6−2y Ta dường nhân thấy biểu

thức khó phân tích biểu thức ví dụ đầu Tuy nhiên, để ý kĩ thấy bên mẫu là2y tử 5y, ta mạnh dạn nhân vào tử để xuất hiện2y giống mẫu

(76)

68 4.1 Xét tính chia hết

Lời giải Ta có

(4.18)⇔x= −5y−18 6−2y

⇔2x= −10y−36 6−2y

⇔2x= −66 + 5(6−2y) 6−2y =

−66 6−2y +

⇔2x= −33 3−y +

Như muốnx số nguyên dương thì3−y phải ước −33 Hay3−y∈ {±1;±3;±11,±33} Lại để ý vìy≥1nên 3−y ≤2 Do 3−y∈ {±1;−3;−11;−33} Ta có bảng sau:

3−y −1 −3 −11 −33

y 14 36

x −14 19

Thử lại thấy cặp(x;y) nguyên dương thỏa mãn (4.18) (x;y) = (19; 4),(8; 6),(4; 14),(3; 36)

Nhận xét Bài ta sử dụng phương pháp đưa phương trình ước số Cũng xin ý với bạn lời giải ta nhân x để biến đổi, phải có bước thử lại coi giá trị x, y tìm có thỏa mãn (4.18) hay khơng kết luận

Bài tập đề nghị

Bài1 Giải phương trình nghiệm nguyênx2−xy = 6x−5y−8 Bài2 Giải phương trình nghiệm nguyênx2+x+ = 2xy+y Bài3 Giải phương trình nghiệm nguyênx3−x2y+ 3x−2y−5 =

(77)

4.1 Xét tính chia hết 69

Bài5 (Vào 10 chuyên THPT ĐHKHTN Hà Nội năm 2000-2001)Tìm cặp số nguyên(x, y) thỏa mãn đẳng thứcy(x−1) =x2+ 2. Bài6 Tìm số nhỏ số nguyên dương bội của2007và

có 4chữ số cuối 2008

Bài7 Tìm nghiệm nguyên phương trình5x−3y= 2xy−11 4.1.4 Xét số dư vế

Cơ sở phương pháp.Đọc tiêu đề phương pháp bạn hiểu phương pháp nói đến việc xét số dư vế cho số Vậy, lại phải xét xét có lợi ích "cơng cuộc" giải tốn? Hãy tìm hiểu qua ví dụ đầu sau:

Ví dụ 4.12 Tìm nghiệm ngun phương trình

x2+y2 = 2011 (4.19) Lời giải Ta cóx2;y2 chia4có thể dư hoặc1 nên tổng chúng chia4 dư0; 1hoặc2 Mặt khác2011chia4dưa 3nên phương trình

(4.19) vơ nghiệm ngun

Nhận xét Qua ví dụ đầu ta thấy rõ số dư chia cho4của hai số khác phương trình vơ nghiệm Do ta lại hiểu thêm mục đích phương pháp Bật mí thêm tí phương pháp chủ yếu dùng cho phương trình khơng có nghiệm ngun Cho nên, bạn bắt gặp phương trình mà bạn khơng thể tìm nghiệm cho phương trình đó, nghĩ đến phương pháp Còn ta tiếp tục đến với ví dụ sau:

Ví dụ 4.13 (Balkan MO 1998) Tìm nghiệm nguyên phương trình

x2 =y5−4 (4.20)

Lời giải Ta có: x2 ≡ 0; 1; 3; 4; 5; (mod 11) Trong y5−4 ≡ 6; 7; (mod 11): vơ lý Vậy phương trình (4.20) vơ nghiệm ngun

(78)

70 4.1 Xét tính chia hết

Nhận xét Một câu hỏi lại lóe lên đầu ta: Làm lại tìm số 11 xét đồng dư nhỉ? Đáp án câu hỏi cốt lõi để bạn vận dụng phương pháp này, kinh nghiệm sau:

1 Đối với phương trình nghiệm ngun có tham gia bình phương ta thường xét đồng dư với3,4,5,8 Cụ thể là:

a2 ≡0,1 (mod 3)

a2 ≡0,1 (mod 4)

a2 ≡0,1,4 (mod 5)

a2 ≡0,1,4 (mod 8)

2 Đối với phương trình nghiệm nguyên có tham gia số lập phương ta thường xét đồng dư với 9, x3 ≡ 0; 1; (mod 9)và đồng dư với7, x3 ≡0,1,6 (mod 7)

3 Đối với phương trình nghiệm ngun có tham gia lũy thừa bậc ta thường xét đồng dư với 8, như: z4 ≡ 0,1 (mod 8)

4 Một vấn đề cuối định lí Fermat: Đối với phương trình nghiệm ngun có tham gia lũy thừa có số mũ số nguyên tố số mà cộng1vào số ta số nguyên tố ta thường sử dụng định lí nhỏ Fermat để xét đồng dư

Trên số kinh nghiệm thân, bạn muốn vận dụng phương pháp xét số dư này, yêu cầu ghi nhớ kinh nghiệm tìm cách chứng minh Ngồi ra, bạn muốn mở rộng tầm hiểu biết nữa, bạn tìm đồng dư với lũy thừa khác (chẳng hạn qua ví dụ2 ta rút mođun 11 cho lũy thừa bậc hai, bậc năm) Còn bây giờ, thử xem kinh nghiệm có hiệu khơng nhé!

Ví dụ 4.14 (Bài tốn tuần - diendantoanhoc.net) Chứng minh phương trình sau khơng có nghiệm ngun

(79)

4.1 Xét tính chia hết 71

Nhận xét Thường thường tốn đặt câu hỏi phương trình có nghiệm hay khơng thường có câu trả lời khơng Do để chứng minh phương trình khơng có nghiệm, ta tìm số cho chia VT VP cho số hai số dư khác

Như vậy, công việc ta tìm số Để ý đến số mũ10 khiến ta liên tưởng số 11 số nguyên tố Như lời giải ta áp dụng định lý Fermat nhỏ cho số11 để chứng minh hai vế phương trình chia cho 11không số dư

Lời giải Áp dụng định lý Fermat nhỏ

  

 

x10≡0,1 (mod 11)

y10≡0,1 (mod 11)

z10≡0,1 (mod 11)

Do đóx10+y10−z10≡0,1,2,10 (mod 11)mà199≡8 (mod 11) nên

phương trình vơ nghiệm ngun

Ví dụ 4.15 (Đề thi chọn HSG tốn quốc gia năm 2003 - Bảng B) Hệ phương trình sau có tồn nghiệm nguyên hay không:

x2+y2 = (x+ 1)2+u2 = (x+ 2)2+v2= (x+ 3)2+t2 (4.21)

Nhận xét Ta dự đốn phương trình vơ nghiệm Do cần tìm số chia cả5 vế số dư khác Để ý tốn có bình phương nên ta nghĩ tới việc sử dụng tính chất như:a2 ≡0,1 (mod 3), a2 ≡0,1 (mod 4), a2 ≡0,1,4 (mod 5), a2 ≡ 0,1,4 (mod 8) Ở toán này, ta chọn Bây cần xét tính dư chia cho8

Lời giải Giả sử phương trình (4.21) có nghiệm nguyên(x0, y0, u0, v0, t0), tức là:

x20+y02= (x0+ 1)2+u20= (x0+ 2)2+v20 = (x0+ 3)2+t20 (4.22) Vớia∈Zthìa2 ≡0,1,4 (mod 8) Ta xét khả sau:

(80)

72 4.1 Xét tính chia hết

1 Nếux0 ≡0 (mod 4) thìx20+y20 ≡0,1,4 (mod 8) Và

x0+ 1≡1 (mod 8)⇒(x0+ 1)2 ≡1 (mod 8)

⇒(x0+ 1)2+u20 ≡1,2,5 (mod 8)

x0+ 2≡2 (mod 4)⇒(x0+ 2)2 ≡4 (mod 8)

⇒(x0+ 2)2+v02≡0,4,5 (mod 8)

x0+ 3≡3 (mod 4)⇒(x0+ 3)2 ≡1 (mod 8)

⇒(x0+ 3)2+t20 ≡1,2,5 (mod 8) Nhận thấy{0,1,4} ∩ {1,2,5} ∩ {0,4,5} ∩ {1,2,5}= nên phương trình khơng có nghiệm ngun vớix≡0 (mod 4) Tương tự vớix0 ≡1 (mod 4), x0≡2 (mod 4)vàx0 ≡3 (mod 4)

ta thực tương tự cho kết phương trình khơng có nghiệm ngun

Vậy phương trình (4.21) cho khơng có nghiệm ngun

Nhận xét Ví dụ ta tổng qt lên:

Ví dụ 4.16 Tìm số nguyên dương nlớn cho hệ phương trình (x+ 1)2+y12= (x+ 2)2+y22 = .= (x+n)2+yn2

có nghiệm ngun

Đây đề thi chọn đội tuyển HSG quốc gia toán năm 2003 - Bảng A Lời giải xin giành cho bạn đọc Cũng xin nói thêm thừa nhận rằng, phương pháp xét số dư vế này, tưởng chừng đơn giản, thực chất Dẫn chứng ví dụ trên, tốn hay khó lấy từ khác thi nước nước

Bài tập đề nghị

(81)

4.1 Xét tính chia hết 73

Bài2 Tồn hay không nghiệm nguyên phương trìnhx12+y12+

z12= 372012+ 20141995

Bài3 Giải phương trình nghiệm nguyên312x+ 122x+ 19972x =y2 Bài4 Giải phương trình nghiệm nguyên dương7z = 2x·3y−1 Bài5 Giải phương trình nghiệm nguyên dương2x·3y = + 5z Bài6 Giải phương trình nghiệm tự nhiên19x+ 5y+ 1890 = 1975430+

1993

Bài7 Giải phương trình nghiệm nguyênx3+y3+z3 = 1012

Bài8 (Tuyển sinh vào lớp 10 chuyên Trần Phú, Hải Phòng năm học 2012-2013)x4+y4+z4 = 2012

Bài9 |x−y|+|y−z|+|z−x|= 10 n−1

9 với n∈N

Bài10 Tìm nghiệm nguyên phương trình (2x + 1)(2x+ 2)(2x + 3)(2x+ 4)−5y = 11879

Bài11 Tìm nghiệm nguyên phương trìnhx2+ (x+ 1)2+ (x+ 2)2 =

y2

Bài12 (Tuyển sinh vào THPT chuyên ĐHKHTN Hà Nội năm 2011-2012)Chứng minh không tồn ba số nguyên(x;y;z) thỏa mãnx4+y4 = 7z4+ 5.

Bài13 Giải phương trình nghiệm nguyênx41+x24+· · ·=x413+20122015 Bài14 Chop số nguyên tố lẻ Chứng minh phương trình xp+

yp =p[(p−1)!]p khơng có nghiệm ngun

Bài15 Tìm nghiệm nguyên phương trìnhx2012−y2010 = Bài16 Chứng minh không tồn số nguyênx, y thỏa mãn x5+

y5+ = (x+ 2)5+ (y−3)5.

(82)

74 4.2 Sử dụng bất đẳng thức

4.2

Sử dụng bất đẳng thức

4.2.1 Sắp thứ tự ẩn

Ví dụ 4.17 Giải phương trình nghiệm ngun dương sau

x +

1

y +

1

z = (4.23)

Lời giải Khơng tính tổng qt, ta giả sử 1≤x≤y≤z⇒

x +

1

y +

1

z = 1≤

3

x ⇒x≤3

• Vớix= (4.23) khơng có nghiệm ngun dương • Vớix =

2 +

y +

1

z = ⇔

1 y + z = ≤

y ⇒ y ≤ Mặt

khác,y≥x= 2⇒y∈ {2; 3; 4} Ta thử giá trị củay

∗ Vớiy= (4.23) vơ nghiệm ngun ∗ Vớiy= thìz=

∗ Vớiy= thìz= • Vớix= 3, ta có

3+

y +

1

z = 1⇔

1 y + z = ≤

y ⇒y≤3 Mặt

khác, y≥x= 3⇒y= 3⇒z=

Vậy nghiệm nguyên(x;y;z) (4.23) hoán vị (2; 3; 6);

(2; 4; 4);(3; 3; 3)

Nhận xét Phương pháp sử dụng chỗ thứ tự ẩn 1≤x≤y≤z giới hạn nghiệm để giải

Ta sử dụng phương pháp thứ tự ẩn vai trị ẩn bình đẳng với Dó vận dụng phương pháp bạn cần ý để tránh nhầm lẫn Cụ thể, ta đến với ví dụ sau:

Ví dụ 4.18 Giải phương trình nghiệm nguyên dương

(83)

4.2 Sử dụng bất đẳng thức 75

Lời giải (Lời giải sai) Khơng tính tổng qt, giả sử1≤x≤y≤

z Khi đóx+y+1≤3zhayxyz≤3z, suy raxy≤3 Màz≥y≥x≥1 nên x=y=z=

Nhận xét Cái lỗi sai lời giải x, y, z khơng bình đẳng, nên thứ tự ẩn Sau lời giải đúng: Lời giải Khơng tính tổng quát, giả sử1 ≤x≤y Ta xét trường hợp:

• Nếux=y

(4.24)⇔2y+ =y2z

⇔y(z−2) = ⇔

(

y=

yz−2 = ⇔

(

y=

z=

• Nếu x < y từ (4.24) suy 2y + > xyz ⇒ 2y ≥ xyz ⇒

xz≤2⇒xz∈ {1; 2}

∗ Vớixz = ⇒ x=z = 1, thay vào (4.24) suy y+ =y

(vô nghiệm) ∗ Với xz = ⇒

(

x=

z=

(

x=

z= Từ ta tìm nghiệmx= 1, y= 2, z= hoặcx= 1, y = 3, z= Vậy phương trình có nghiệm nguyên dương là(1; 1; 3),(1; 2; 2),(2; 1; 2),

(2; 3; 1),(3; 2; 1)

Nhận xét Bây bạn hiểu cách xếp ẩn Nhưng lại xétx=yvàx < ymà lại không vào phân

(84)

76 4.2 Sử dụng bất đẳng thức

tích ln trước Nếu bạn để ý không phân chia thành hai trường hợp nhưu phương trình (4.24) thành2y+1≥y2z, khó để tiếp tục phân tích nghiệm Do việc xét nhưu hợp lí

Bài tập đề nghị

Bài1 Giải phương trình nghiệm nguyên dương2(x+y+z)+9 = 3xyz Bài2 Giải phương trình nghiệm nguyên dươngxyz= 3(x+y+z) Bài3 Giải phương trình nghiệm nguyên dương5(x+y+z+t) + 10 =

2xyzt

Bài4 Giải phương trình nghiệm nguyên dươngx! +y! = (x+y)! (Kí hiệux!là tích số tự nhiên liên tiếp từ1 đến x)

Bài5 Tìm nghiệm nguyên dương phương trìnhx3+ 7y=y3+ 7x Bài6 Tìm nghiệm nguyên dương phương trìnhx1+x2+· · ·+x12=

x1x2· · ·x12

Bài7 Tìm tất nghiệm nguyên dương phương trình x

y2z2 +

y z2x2 +

z x2y2 =t

Bài8 Tìm nghiệm nguyên dương phương trìnhx! +y! +z! =u! 4.2.2 Sử dụng bất đẳng thức

Nhận xét Để giải phương trình này, ta thường sử dụng bất đẳng thức quen thuộc để đánh giá vế phương trình khơng nhỏ (hoặc khơng lớn hơn) vế cịn lại Muốn cho hai vế bất đẳng thức phải trở thành đẳng thức

Cụ thể, ta có số bất đẳng thức thường dùng:

1 Bất đẳng thức Cauchy (hay gọi bất đẳng thức AM-GM): Nếua1, a2,· · ·, an số thực không âm

a1+a2+· · ·+an

n ≥

n

(85)

4.2 Sử dụng bất đẳng thức 77

Dấu đẳng thức xảy khia1 =a2=· · ·=an

2 Bất đẳng thức Bunhiacopxki (hay gọi bất đẳng thức Cauchy - Bunyakovsky - Schwarz): Với hai số thực (a1, a2,· · · , an) và(b1, b2,· · · , bn), ta có

a21+a22+· · ·+a2n

b21+b22+· · ·+b2n

≥(a1b1+a2b2+· · ·+anbn)2 Đẳng thức xảy tồn số thựcksao choai =kbi với mọii= 1,2,· · · , n

3 Bất đẳng thức Trebusep (hay viết bất đẳng thức Chebyshev): Cho dãy hữu hạn số thực theo thứ tự a1 ≤ a2 ≤ · · · ≤an b1≤b2 ≤ · · · ≤bn Khi ta có:

n(a1b1+a2b2+· · ·+anbn)≥(a1+a2+· · ·+an)(b1+b2+· · ·+bn) Dấu đẳng thức xảy

a1 =a2=· · ·=an

b1 =b2 =· · ·=bn

Bây ta xem xét số ví dụ sau:

Ví dụ 4.19 Giải phương trình nghiệm ngun dương sau:

x6+z3−15x2z= 3x2y2z−(y2+ 5)3 (4.25) Lời giải Nhận xét Ở phương trình nhìn vào hẳn đa số bạn có phần rối, khơng xác định phương pháp làm, không vận dụng phương pháp học Tuy nhiên để ý kĩ xí ta thấy x6 = (x2)3 điều có đặc biệt? Ta thấy (x2)3, z3 (y2+ 5)3 có bậc ba đề cho nguyên dương nên ta nghĩ đến Bất đẳng thức kinh điển: Bất đẳng thức Cauchy hay gọi bất đẳng thức AM-GM

Ta giải sau

(4.25)⇔(x2)3+ (y2+ 5)3+z3 = 3x2z(y2+ 5)

(86)

78 4.2 Sử dụng bất đẳng thức

Áp dụng Bất đẳng thức AM-GM cho ba số dương(x2)3, z3 và(y2+ 5)3 ta được:

(x2)3+(y2+5)3+z3 ≥3p3

(x2)3.(y2+ 5)3.z3 = 3x2z(y2+5) =V P(4.25) Dấu xảy khix2 =y2+ =

Mặt khác ta có:

x2=y2+ 5⇔(x−y)(x+y) =

Đây dạng phương trình nghiệm nguyên quen thuộc ta học, tơi tin bạn dễ dàng giải phương trình trên, từ

x;y ta tìm zmột cách dễ dàng

Đáp số: Nghiệm nguyên phương trình (4.25) là(x;y;z) = (3; 2; 9)

Ví dụ 4.20 Tìm nghiệm ngun phương trình (x+y+z)2 = 3(x2+y2+ 1)

Lời giải Áp dụng bất đẳng thức Bunyakovsky cho hai số (x, y,1) và(1,1,1)ta có

(x+y+ 1)2 ≤(12+ 12+ 12)(x2+y2+ 1) = 3(x2+y2+ 1) Đẳng thức xảy x=y=

Vậy phương trình có nghiệm ngun là(x, y) = (1,1)

Nhận xét Các Toán phương trình nghiệm nguyên mà giải cách sử dụng Bất đẳng thức dung dễ bị lộ dụng ý người đề léo Tuy nhiên, ta phải thành thạo phương pháp khơng xem thường để tránh sai lầm đáng tiếc sửa

Bài tập đề nghị

Bài1 Tìm nghiệm nguyên dương x, y thỏa mãn phương trình (x2+ 1)(x2+y2) = 4x2y

Bài2 Tìm nghiệm nguyên phương trình xy

z + yz

x + zx

(87)

4.2 Sử dụng bất đẳng thức 79

Bài3 (Đề thi tuyển sinh vào đại học Vinh) Tìm nghiệm nguyên phương trình

(x2+ 1)(y2+ 4)(z2+ 9) = 48xyz

Bài4 Giải phương trình nghiệm nguyên

x−2+ √

y−1+ 25 √

z−5 = 16− √

x−2−py−1−√z−5

Bài5 Tìm nghiệm nguyên hệ phương trình

  

 

x2+z2 =

y2+t2 = 16

xt+yz= 12 Bài6 Tìm nghiệm nguyên dương phương trìnhx3+y3−6xy+8 =

0

Bài7 Tìm nghiệm nguyên hệ phương trình

(

xy+yz+zx= 12

x4+y4+z4= 48 Bài8 Cho phương trìnhx3+y3+z3 =nxyz

a, Chứng minh m = m = phương trình khơng có nghiệm ngun dương

b, Giải phương trình nghiệm nguyên dương khim= Bài9 Giải phương trình nghiệm nguyên dương(x3+y3)+4(x2+y2)+

4(x+y) = 16xy

Bài10 Giải phương trình nghiệm nguyên dương

3(x4+y4+x2+y2+ 2) = 2(x2−x+ 1)(y2−y+ 1) Bài11 Giải phương trình nghiệm nguyên dương vớix, y, zlà số đôi

một khác

x3+y3+z3 = (x+y+z)2

(88)

80 4.2 Sử dụng bất đẳng thức

4.2.3 Chỉ nghiệm

Nhận xét Phương pháp dành cho toán giải phương trình nghiệm nguyên mà ta tìm xác nghiệm nguyên muốn chứng minh phương trình có nghiệm ngun mà thơi

Ví dụ 4.21 Tìm nghiệm ngun dương phương trình

2x+ 3x= 5x (4.26)

Lời giải Chia vế phương trình (4.26) cho số dương5x, ta được: (4.26)⇔

x + x = Với x = ta

5 +

5 = 1:đúng nên x = nghiệm (4.26)

Vớix >1

x + x > + =

Do giá trịx > khơng nghiệm (4.26) Vậy nghiệm

nguyên dương (4.26) làx=

Nhận xét Ở ví dụ trên, ta dễ nhận thấy x = nghiệm phương trình nên cần chứng minh với x > phương trình vơ nghiệm Ngồi ra, từ tốn ta mở rộng thành hai tốn mới:

Bài tốn 4.1 Tìm nghiệm nguyên dương phương trình (√3)x+ (√4)x = (√5)x

(89)

4.2 Sử dụng bất đẳng thức 81

Bài tốn 4.2 Tìm nghiệm ngun dương phương trình

3x+ 4y = 5z

Bài tốn4.2rõ ràng nâng cao lên rõ rệt, lời giải toán sử dụng phương pháp xét số dư học Sau lời giải đẹp củakhanh3570883 Điều hành viên THPT VMF: Lời giải Xét theo module ta có:

5z ≡(−1)z (mod 3)⇒4y ≡(−1)z (mod 3)⇒z= 2h(h∈N)

⇒(5h−2y)(5h+ 2y) = 3x

Do5h−2y và5h+ 2y không đồng thời chia hết cho nên5h+ 2y = 3x và5h−2y =

Ta có5h+2y ≡(−1)h+(−1)y = (mod 3)và5h−2y ≡(−1)h−(−1)y = (mod 3)⇒h lẻ y chẵn

Nếu y > 5h + 2y ≡ (mod 4) ⇒ 3x ≡ (mod 4) ⇒ 3x ≡ (mod 8)

Mặt khác 5≡5h+ 2y (mod 8) ⇒5 ≡3x (mod 8)⇒ ≡1 (mod 8): vô lý

Do y= Suy x=y=z=

Phương pháp thường hay sử dụng cho phương trình có ẩn số mũ phương trình có nghiệm nhỏ

4.2.4 Sử dụng ∆ phương trình bậc

Nhận xét Viết phương trình dạng phương trình bậc hai ẩn, dùng điều kiện.∆≥0 ∆là số phương Ta tùy trường hợp để chọn hai cách xét∆vào việc giải tốn Ví dụ 4.22 Giải phương trình nghiệm ngun

3x2+ (3y−1)x+ 3y2−8y= (4.27)

(90)

82 4.2 Sử dụng bất đẳng thức

Lời giải Coi (4.27) phương trình bậc ẩn x Xét ∆x = −27y2 + 9y+

Đề (4.27) có nghiệmx

∆x ≥0⇔ −27y2+ 9y+ 1≥0⇔ −0,01≤y ≤3,3⇒y∈ {0; 1; 2; 3} Nếuy= 0⇒3x2−x= 0⇒x= x∈Z

Nếuy= 1⇒3x2+ 2x−5 = 0⇒x= vìx∈Z

Nếuy= hoặcy= khơng tìm x ngun nên loại

Vậy (4.27) có nghiệm nguyên(x;y) = (0; 0); (1; 1)

Ví dụ 4.23 Giải phương trình nghiệm ngun

3x2−y2−2xy−2x−2y+ = (4.28) Lời giải Ta có

(4.28)⇔y2+ 2(x+ 1)y−(3x2−2x+ 8) = 0 ∆0y = (x+ 1)2+ 3x2−2x+ = 4x2+

Để (4.28) có nghiệm thì∆0y = 4x2+9là số phương Đặt4x2+9 =

k2vớik∈N, ta đưa phương trình ước số tìm đượcx∈ {2; 0;−2}

• Vớix= ta đượcy2+ 6y−16 = 0nên y∈ {−8; 2} • Vớix= thìy2+ 2y−8 = 0nên y∈ {−4; 2}. • Vớix=−2thì y2−2y−24 = nên y∈ {−6; 4}

Kết luân Vậy phương trình (4.28) có nghiệm (x;y) (2;−8), (2; 2), (0;−4),(0; 2),(−2; 6),(−2;−4)

Nhận xét Hai tốn sử dụng phương pháp đưa phương trình ước số để giải

Bài tập đề nghị

(91)

4.2 Sử dụng bất đẳng thức 83

Bài2 Tìm nghiệm nguyên phương trìnhx+xy+y=x2+y2 Bài3 Giải phương trình nghiệm nguyên 10x2 + 5y2 + 38−12xy +

16y−36x=

Bài4 Tìm nghiệm nguyên phương trình9x2+x2+ 4y2+ 34−12xy+ 20y−36x=

Bài5 Tìm nghiệm nguyên dương củax+ 2y2+ 3xy+ 3x+ 5y= 14 Bài6 Tìm nghiệm nguyên phương trìnhx2−xy−6y2+2x−6y−10 = Bài7 Tìm nghiệm nguyên phương trìnhx2+2y62+3xy+3x+5y=

15

Bài8 Tìm nghiệm nguyên phương trình2x2+ 6y2+ 7xy−x−y= 25

Bài9 Tìm nghiệm nguyên phương trình9x2−10y2−9xy+ 3x− 5y=

Bài10 Tìm nghiệm nguyên phương trình12x2+6xy+3y2 = 28(x+

y)

(Thi vào lớp 10 chuyên, ĐHKHTN-ĐHQGHN năm 1994) Bài11 Tìm nghiệm nguyên phương trình3(x2+xy+y2) =x+ 8y Bài12 Tìm nghiệm nguyên phương trình7(x2+xy+y2) = 39(x+

y)

Bài13 Tìm nghiệm nguyên phương trình 2x2+y2 + 3xy+ 3x+ 2y+ =

Bài14 Tìm nghiệm nguyên phương trìnhx2+2y2+3xy−x−y+3 =

Bài15 Tìm nghiệm nguyên phương trình3x2+4y2+12x+3y+5 =

(92)

84 4.2 Sử dụng bất đẳng thức

4.2.5 Phương pháp kẹp

Nhận xét Sử dụng tính chất lũy thừa bậc số nguyên liên tiếp tích số nguyên liên tiếp để đưa phương trình nghiệm nguyên cần giải dạng phương trình khác ẩn quen thuộc Phương pháp cịn có cách gọi khác phương pháp khử ẩn Ta thường vận dụng nhận xét sau:

1 Xn ≤ Yn ≤ (X+a)n (a ∈

N∗) Yn = (X +a−i)n với

i= 0; 1; 2;· · ·;a Ví dụ vớin= thì:

• Khơng tồn tạix∈Z để a2 < x2 <(a+ 1)2 với a∈Z

• Nếua2 < x2<(a+ 2)2 thìx2= (a+ 1)2

2 X(X+ 1)· · ·(X+n)≤Y(Y + 1)· · ·(Y +n)≤(X+a)(X+a+ 1)· · ·(X+a+n) Y(Y + 1)· · ·(Y +n) = (X+i)(X+ +

i)· · ·(X+a+i) vớii= 0; 1; 2;· · ·;a Ví dụ:

• Khơng tồn tạib∈Zđể a(a+ 1)< b(b+ 1)<(a+ 1)(a+ 2)

vớia∈Z

• Với a(a+ 1) < b(b+ 1) < (b+ 2)(b+ 3) b(b+ 1) = (b+ 2)(b+ 3)

Ví dụ 4.24 Tìm số ngun dương x để biểu thức sau số phương

A=x4+ 2x3+ 2x2+x+ (4.29) Lời giải VìA số phương nên ta đặt

A=x4+ 2x3+ 2x2+x+ =y2(y∈N) Ta thấy

y2 = (x4+ 2x3+x2) +x2+x+ = (x2+x)2+

x+1

2

+11

(93)

4.2 Sử dụng bất đẳng thức 85

Nếux= 1⇒A= 9: số phương nên thỏa đề Nếux >1 xét hiệu

(x2+x+1)2−y2 =x2+x−2 = (x+2)(x−1)>0⇒y2<(x2+x+1)2,(ii) Từ (i) (ii), ta có

(x2+x)2< y2 <(x2+x+ 1)2 Suy ra, không tồn tạiy∈N để y2 =A x >1

Vậyx= giá trị cần tìm

Ví dụ 4.25 Giải phương trình nghiệm nguyên

x4+x2+ =y2−y (4.30) Lời giải Ta có đánh giá sau

x2(x2+ 1)< x4+x2+ 4<(x2+ 2)(x2+ 3) (4.31) Từ (4.30) (4.31) suy

x2(x2+ 1)< y(y−1)<(x2+ 2)(x2+ 3) (4.32) Vìx, y, z nguyên nên từ (4.32) suy

y(y−1) = (x2+ 1)(x2+ 2) (4.33) Từ (4.30) (4.33)

x4+x2+ = (x2+ 1)(x2+ 2)⇔x2= 1⇔x=±1 Từ dễ tìm đượcy =−1hoặcy=

Vậy pt cho có bốn cặp nghiệm

(x, y) ={(1,−2),(1,3),(−1,−2),(−1,3)} Bài tập đề nghị

Tìm nghiệm nguyên phương trình sau:

(94)

86 4.3 Nguyên tắc cực hạn, lùi vô hạn

Bài1 x4+x2+ =y2

Bài2 3(x4+y4+x2+y2+ 2) = 2(x2−x+ 1)(y2−y+ 1) Bài3 2x4+ 3x2+ 1−y2=

Bài4 x2+ (x+y)2 = (x+ 9)2 Bài5 y3−x3= 2x+

Bài6 x4−y4+z4+ 2x2z2+ 3x2+ 4z2+ = Bài7 x3−y3−2y2−3y−1 =

Bài8 x4+ (x+ 1)4 =y2+ (y+ 1)2 Bài9 9x−3x=y4+ 2y3+y2+ 2y

Bài10 x4+x2−y2+y+ 10 = Bài11 x6−4y3−4y4 = + 3y+ 6y2

Bài12 (x−2)4−x4 =y3

Bài13 x3+ 8x2−6x+ =y3

4.3

Nguyên tắc cực hạn, lùi vô hạn

4.3.1 Lùi vơ hạn

Ví dụ 4.26 (Korea 1996) Giải phương trình nghiệm nguyên sau:

x2+y2+z2 = 2xyz (4.34) Lời giải Giả sử(x0;y0;z0) nghiệm nguyên (4.34) ta có

x02+y02+z02 = 2x0y0z0

(95)

4.3 Nguyên tắc cực hạn, lùi vô hạn 87

• Trường hợp 1.Trong x0;y0;z0, có số lẻ, số chẵn Khơng tính tổng qt, giả sử x0;y0 lẻ cịnz0 chẵn Xét theo module

V T(4.34)≡2 (mod 4), V P(4.34)≡0 (mod 4) :vô lý! Vậy trường hợp khơng xảy

•Trường hợp 2.x0; y0; z0 chẵn Đặtx0= 2x1; y0 = 2y1; z0= 2z1 vớix1;y1;z1 ∈Z Thay vào (4.34) rút gọn, ta thu

x12+y12+z12 = 4x1y1z1 Lập luận trên, ta lại đượcx1; y1; z1 chẵn

Q trình diễn tiếp tục nênx0; y0; z0 2k với ktự nhiên tùy ý Điều xảy khix0=y0=z0 = 4.3.2 Nguyên tắc cực hạn

Định nghĩa 4.1 Nguyên tắc cực hạn hay cịn gọi ngun lí khởi đầu cực trị Về mặt hình thức phương pháp khác với phương pháp lùi vô hạn cách sử dụng chứng minh phương trình có nghiệm tầm thường (nghiệm tầm thường nghiệm 0) Phương pháp giải sau:

Giả sử (x0;y0;z0; )là nghiệm củaf(x;y;z; ) với điều kiện ràng buộc bộ(x0;y0;z0; ) Chẳng hạn x0 nhỏ hoặcx0+y0+

z0 + nhỏ sau phép biến đổi số học ta lại tìm nghiệm (x1;y1;z1; ) trái với điều kiện ràng buộc Ví dụ ta chọn (x0;y0;z0; )với điều kiện x0 nhỏ sau ta lại tìm (x1;y1;z1; ) với x1 < x0 dẫn đến phương trình

có nghiệm tầm thường

Ví dụ 4.27 Giải phương trình nghiệm nguyên sau

8x4+ 4y4+ 2z4 =t4 (4.35) Lời giải Giả sử(x0;y0;z0;t0)là nghiệm nguyên không tầm thường (4.35) vớix0 nhỏ

(96)

88 4.3 Nguyên tắc cực hạn, lùi vô hạn

Từ (4.35) suy rat0 chẵn Đặtt= 2t1(t1∈Z)thế vào (4.35) rút gọn,

ta

4x4o+ 2yo4+zo4 = 8t41

Do vậyz0 chẵn Đặtz0 = 2z1(z1∈Z), vào rút gọn ta

2x4o+y4o+ 8z14 = 4t41

Do vậyy0 chẵn Đặt y0= 2y1(y1 ∈Z), vào rút gọn ta

x4o+ 8y41+ 4z14 = 2t41

Do vậyx0 chẵn Đặt x0 = 2x1(x1∈Z), vào phương trình ta

8x41+ 4y14+ 2z14 =t41

Suy ra(x1;y1;z1;t1) nghiệm (4.35) Dễ thấy x1 < x0 (vơ lí với điều giả sử) Do phương trình có nghiệm ngun

(x;y;z;t) = (0; 0; 0; 0)

Bài tập đề nghị

Bài1 Giải phương trình nghiệm nguyênx2+y2 = 3z2

Bài2 Giải phương trình nghiệm nguyênx3+ 2y3 = 4z3

Bài3 Giải phương trình nghiệm nguyên3x2+ 6y2+ 12z2 =t2

(97)

Chương

5

Phương trình đồng dư

5.1 Phương trình đồng dư tuyến tính 89 5.2 Phương trình đồng dư bậc cao 90 5.3 Hệ phương trình đồng dư bậc

một ẩn 90

5.4 Bậc phương trình đồng dư 95 5.5 Bài tập 95

5.6 Ứng dụng định lý Euler để giải phương trình đồng dư 96

5.7 Bài tập 101

Trần Trung Kiên (Ispectorgadget) Nguyễn Đình Tùng (tungc3sp)

5.1

Phương trình đồng dư tuyến tính

Định nghĩa 5.1 Phương trình đồng dư dạngax≡b (mod m)được gọi phương trình đồng dư tuyến tính vớia, b, mlà số biết

x0 nghiệm phương trình ax0≡b (modm) Nếux0 nghiệm phương trình phần tử thuộc lớp x0

cũng nghiệm

Ví dụ 5.1 Giải phương trình đồng dư sau: 12x≡7 (mod 23)

Lời giải Do(12; 23) = nên phương trình ln có nghiệm Ta tìm số nguyên cho + 23k chia hết cho 12 Chọn k = suy ra12x≡7.24 (mod 23)⇒x≡14 (mod 23)

(98)

90 5.2 Phương trình đồng dư bậc cao

Ví dụ 5.2 Giải phương trình 5x≡2 (mod 7) Lời giải Vì(5; 2) = 1nên tồn số k= cho2 + 7k chia hết cho Khi 5x ≡2 + 6.7 (mod 7) ta nghiệm x ≡ 30

5 ≡6 (mod 7)

hay x= + 7k

Ví dụ 5.3 Giải phương trình: 5x≡4 (mod 11) Lời giải Ta có:

5x≡4 (mod 11) 4≡4 (mod 11)

Áp dụng tính chất bắc cầu ta có:5x≡4 (mod 11)⇒5x= 11t+ Ta lấyt= 1;x= Từ phương trình có nghiệm

x≡3 (mod 11)

Nhận xét Cách xác định nghiệm đơn giản dùng trường hợpalà số nhỏ dễ thấy sốk

5.2

Phương trình đồng dư bậc cao

Ví dụ 5.4 Giải phương trình 2x3+ 4≡0 (mod 5) Lời giải Ta thấyx= suy ra2x3 ≡ −4 (mod 5)

Nênx= nghiệm phương trình cho

5.3

Hệ phương trình đồng dư bậc ẩn

Định nghĩa 5.2 Hệ phương trình có dạng sau gọi hệ phương trình đồng dư bậc ẩn

   

  

x≡b1 (modm1)

x≡b2 (modm2)

x≡bk (mod mk)

Vớim1;m2; mk số nguyên lớn vàb1;b2; ;bklà

(99)

5.3 Hệ phương trình đồng dư bậc ẩn 91

Nhận xét • Trong trường hợp tổng qt, chứng minh rằng: Điều kiện cần đủ để hệ phương trình (5.2) có nghiệm làU CLN(mi;mj)chia hếtbi−bj vớii6=j(1≤i, j≤k) • Giả sử m = pα11pαa2

2 pαkk phân tích tiêu chuẩn m Khi phương trình đồng dưf(x)≡0 (modm)tương đương với hệ phương trình đồng dư f(x) ≡0 (mod piα1), i = 1,2, , k.Từ suy x ≡ b1 (modpα11) nghiệm phương trình f(x)≡0 (mod pi), i= 1,2, , k nghiệm hệ phương trình hệ phương trình đồng dư

   

  

x≡b1(modpα11)

x≡b2(modpα22)

x≡bk modpαkk

cho ta nghiệm phương trình f(x)≡0(modm)

Vậy • Trường hợp tổng quát giải phương trình đồng dư dẫn đến giải hệ Với module m1, m2, , mk đôi nguyên tố

Phương pháp chung để giải:

• Trường hợp 1: hệ phương trình

x≡b1 (modm1)

x≡b2 (modm2)

Với giả thiếtd= (m1, m2)chia hết chob1−b2 Trước tiên ta nhận xét rằng, sốx=b1+m1t, t∈Zlà nghiệm phương trình

thứ Sau ta tìm cách xác định t cho x nghiệm phương trình thứ hai, nghĩa hệ hai phương trình tương đương với hệ phương trình

x=b1+m1t

b1+m1t≡b2 (modm2)

(100)

92 5.3 Hệ phương trình đồng dư bậc ẩn

Vì giả thiết d= (m1, m2) ước b1−b2 nên phương trình: b1+

m1t≡b2 (modm2) tương đương với phương trình:

m1

d t≡

b2−b1

d (mod m2

d )

Nhưng (m1

d , m2

d ) = nên phương trình đồng dư cho ta

nghiệmt≡t0 (mod

m2

d ), tập hợp tất số nguyên t=t0+

m2

d u, u∈Z

Thay biểu thức t vào biểu thức tính x ta tập hợp giá trị x nghiệm hai phương trình đồng dư xét là:

x=b1+m1(t0+

m2

d u) =b1+m1t0+

m1m2

d u, hayx =x0+mu vớix0 =b1+m1t0, m=BCN N(m1, m2)

Vậyx≡x0 (modm)là nghiệm hệ hai phương trình đồng dư xét

• Trường hợp 2: Hệ gồm n phương trình Đầu tiên giải hệ hai phương trình hệ cho, thay hệ hai phương trình giải nghiệm tìm thấy, ta hệ gồmn−1 phương trình tương đương với với hệ cho Tiếp tục saun−1bước ta nghiệm cần tìm

Ví dụ 5.5 Giải hệ phương trình:

      

x≡26 (mod 36)

x≡62 (mod 60)

x≡92 (mod 150)

x≡11 (mod 231)

4

Lời giải Hệ hai phương trình:

x≡26 (mod 36)

x≡62 (mod 60) ⇔

x= 26 + 36t

26 + 36t≡62, t∈Z 26 + 36t ≡ 62 (mod 60)

(101)

5.3 Hệ phương trình đồng dư bậc ẩn 93

Vậy nghiệm hệ là:x≡26 + 36.1 (mod 180)hay x≡62 (mod 180) Do hệ phương trình cho tương đương với hệ:

 

x≡62 (mod 180)

x≡92 (mod 150)

x≡11 (mod 231) Ví dụ 5.6 Giải hệ phương trình

x≡62 (mod 180)

x≡92 (mod 150) ⇔

x= 62 + 180t

62 + 180t≡92 (mod 150), t∈Z Lời giải Ta có:

62 + 180t≡92 (mod 1)50) ⇔180t≡30 (mod 150)

⇔6t≡1 (mod 5)⇔ t≡1 (mod 5) Vậy nghiệm hệ là:

x≡62 + 180.(1+) (mod 900)⇔x≡242 (mod 900) Hệ cho tương đương với:

x≡242 (mod900)

x≡11 (mod231)

Hệ có nghiệm x ≡242 (mod 69300) , nghiệm

hệ cho cần tìm

Ví dụ 5.7 Tìm số ngun dương nhỏ thỏa tính chất: chia7 dư5,

chia 11 dư chia 13 dư

Lời giải Ta có: n1 = 7;N1 = 11.13 = 143;n2 = 11;N2 = 7.13 = 91;n3 = 13;N3 = 7.11 = 77

Ta cóN1b1 ≡3b1 ≡1 (mod 7)→b1=−2 Tương tựb2 = 4;b3=−1 Vậya= 143(−2)5 + (91)(4)(7) + (77)(−1)(3) =−1430 + 2548−231 = 887vậy số cần tìm có dạngb= 877 + 1001k

Vậy877là số cần tìm

(102)

94 5.3 Hệ phương trình đồng dư bậc ẩn

Ví dụ 5.8 (Chọn đội tuyển KHTN) Xét hệ đồng dư gồm phương trình:

xy ≡ −1 (modz) (5.1)

yz ≡ (modx) (5.2)

xz ≡ (mody) (5.3)

Hãy tìm số (x, y, z)nguyên dương phân biệt với1 trong3 số 19.4 Lời giải Từ ba phương trình, theo tính chất đồng dư ta có

xy+ z yz−1 x vàzx−1 y

Suy

(xy+ 1)(yz−1)(zx−1) xyz

⇒x2y2z2−x2yz−xy2z+xyz2+xy−yz−zx+ xyz

⇒xy−yz−zx+ xyz

Nhận thấy dox, y, z nguyên dương cho nênxyz≥1 Suy xy−yz−

zx+ 1≤2xyz

Mặt khácyz+zx−xy−1≤2xyz⇒ −(yz+zx−xy−1)≥ −2xyz

Do ta có bất phương trình kép −2xyz≤xy−yz−zx+ 1≤2xyz

Màxy−yz−zx+1 xyz⇒xy−yz−zx+1 = 2xyz,1xyz,0,−1xyz,−2xyz

• Trường hợp 1:xy−yz−zx+ = 2xyz⇒xy ≡ −1 (mod z), yz ≡1 (modx), zx≡1 (mod y)

Cho nên ta cần tìm nghiệm củaxy−yz−zx+ = 2xyz xong Vìx, y, z có số 19 nên ta thay vào

Nếu x = 19 ⇒ 19y−yz−19z+ = 38yz ⇒ 39yz −19y+ 19z = ⇒(39y+ 19)(39z−19) =−322Vớiy= 19hoặc z= 19 tương tự •Trường hợp 2,3,4,5:xy−yz−zx+1 = 1xyz,0,−1xyz,−2xyzlàm hồn tồn tương tự, ta đẩy phương trình có dạngau+bv=ab+uv+x

vớix số

Đưa (a−v)(b−u) = x giải kiểu phương trình ước số Bài toán

(103)

5.4 Bậc phương trình đồng dư 95

Nhận xét Bài tốn mà khơng cho điều kiện số 19 khơng đưa dạngau+bv=ab+uv+x↔(a−v)(b−u) =x lúc suy vơ hạn nghiệm

5.4

Bậc phương trình đồng dư

Định nghĩa 5.3 Xét phương trình đồng dư f(x) = (modm) với

f(x) =a0xn+a1xn−1+ +an, ∈N, i= 0,1, , n

Nếua0 khơng đồng dư0 (modm)thì ta nóinlà bậc phương trình

đồng dư

Ví dụ 5.9 Xác định bậc phương trình 15x6−8x4+x2+ 6x+ 8≡0

(mod 3)

Lời giải Ta thấy 15 ≡ (mod 3) nên bậc phương trình khơng phải bậc Phương trình tương đương với −8x4+x2 + 2≡0 (mod 3)

Vì−86≡0 (mod 3)nên bậc phương trình n=

5.5

Bài tập

Bài1 Giải phương trình sau: a) 7x ≡ (mod 13) b) (a+b)x ≡

a2+b2 (modab) với (a, b) = c)17x≡13 (mod 11) d) x2+

x−2≡1 (mod 3)

Bài2 Giải hệ phương trình: a)

   

  

x≡1 (mod 3)

x≡4 (mod 4)

x≡2 (mod 7)

x≡9 (mod 11) b)

 

5x≡1 (mod 12) 5x≡2 (mod 8) 7x≡3 (mod 11)

Bài3 Tìm a nguyên để hệ phương trình sau có nghiệm

(104)

96 5.6 Ứng dụng định lý Euler để giải phương trình đồng dư

a)

   

  

x≡3 (mod 3)

x≡1 (mod 4)

x≡11 (mod 7)

x≡a (mod 11) b)

2x≡a (mod 3) 3x≡4 (mod 10)

Bài4 Một lớp gồm 40 học sinh đứng thành vòng tròn quay mặt vòng tròn để chơi bóng Mỗi học sinh nhận bóng phải ném qua mặt bạn bên tay trái Chứng minh tất học sinh lớp nhận bóng ném tới sau 40 lần ném bóng liên tiếp

5.6

Ứng dụng định lý Euler để giải phương trình

đồng dư

Qua viết xin giới thiệu phương pháp để giải phương trình đồng dư cách khai thác định lý Euler

Trước hết, xin nhắc lại vài kiến thức quen thuộc

Định nghĩa 5.4 Hàm Euler ϕ(m) với số nguyên dương m số tự nhiên nhỏ hơnm số nguyên tố vớim 5.6.1 Định lý Euler

Định lý 5.1 (Euler)– Cho m số nguyên dương (a, m) =

aϕ(m)≡1 (mod m) Hàmϕcó tính chất sau:

• ϕ(mn) =ϕ(m)ϕ(n) với(m;n) =

(105)

5.6 Ứng dụng định lý Euler để giải phương trình đồng dư 97

• Nếum=pα1

1 p α2

2 pα

k

k , pi số nguyên tố

φ(m) =m

1−

p1

1−

p2

1−

pk

Bây ta xétm=a.b (a;b) = có kết sau Định lý 5.2–

aϕ(b)+bϕ(a)≡1 (modab) (5.4) Chứng minh Theo định lý Euler ta có:aϕ(b)≡1 (mod b)màbϕ(a)≡0 (modb)

Nênaϕ(b)+bϕ(a)≡1 (mod b).

Tương tự ta có:aϕ(b)+bϕ(a) ≡1 (mod a)

Theo tính chất đồng dư :aϕ(b)+bϕ(a)≡1 (modab)

Định lý 5.3– Giả sử có k(k≥ 2)số nguyên dương m1;m2; mk chúng nguyên tố với đôi ĐặtM =m1.m2 mk=miti với i= 1,2,3 , k ta có

tϕ(m1)

1 +t ϕ(m2)

2 + +t ϕ(mk)

k ≡1 (modM) (5.5)

Chứng minh Từ giả thiết ta có(mi, ti) = 1với mỗii= 1,2, , knên theo định lý Euler

tϕ(m1)

1 ≡1 (mod mi) (5.6)

Mặt khác với i;j thuộc tập 1;2; ;k i6= j tj chia hết cho mj nên (tj;mi) =mi hay

tϕ(mi)

j ≡0 (modmi) (5.7)

ĐặtS =tϕ(m1)

1 +t ϕ(m2)

2 + +t ϕ(mk)

k Từ (5.6) (5.7) có S≡tϕmi

i ≡1 (mod mi)

Vìm1;m2; mk nguyên tố với đôi một, nên theo tính chất đồng dư thức có

S−1≡0 (modm1.m2 mk)⇔S ≡1 (mod M), tức có (5.5)

(106)

98 5.6 Ứng dụng định lý Euler để giải phương trình đồng dư

Khi mở rộng (5.4) theo hướng nâng lên lũy thừa số hạng ta có kết sau

Định lý 5.4– Với (a, b) = vàn, v hai số nguyên dương

anϕ(b)+bvϕ(a)≡1 (modab) (5.8) Chứng minh Để tiện lập luận đặtx=aϕ(b)

Theo định lý Euler thìx=aϕ(b) ≡1 (modb)⇔x−1≡0 (modb) Đồng thờix=aϕ(b) ≡0 (moda)

Từ cóx(x−1)≡0 (mod a)vàx(x−1)≡0 (modb)nênx(x−1)≡0 (modab)

Từ x3 ≡x2.x≡x.x≡x2 ≡x (modab) lập luận có

xn≡x (modab) hay anϕ(b)≡aϕ(b) (mod ab)

Tương tự ta có: bvϕ(a) ≡ bϕ(a) (modab) nên theo (5.4) có anϕ(b) +

bvϕ(a) ≡bϕ(a)+aϕ(b)≡1 (mod ab)

(5.8) chứng minh

Hệ 5.1– Với (a;b) = anϕ(b)+bnϕ(a) ≡1 (modab)

Hệ chứng minh trực tiếp nâng hai vế hệ thức (5.4) lên lũy thừa bậcn (sử dụng triển nhị thức Newton) ý ab≡ (mod ab) Nên lưu ý đồng dư thức a 6≡0 (modab)!

Với kí hiệu định lý5.3ta cóti.tj ≡0 (modM)với i khác j

i;j thuộc tập 1,2, ,k (nhưngt6≡0 (mod M) với mọii= 1,2,3, k) Từ nâng hai vế (5.5) lên lũy thừa bậcnta có kết sau Định lý 5.5– Với giả thiết định lý 5.3 ta có:

tnϕ(m1)

1 +t nϕ(m2)

2 + +t nϕ(mk)

k ≡1 (mod M) (5.9)

Với kí hiệu ta đặta=mi vàb=ti theo (5.4) có

mnϕ(ti)

i +t

nϕ(mi)

i ≡1 (modM) (5.10)

(107)

5.6 Ứng dụng định lý Euler để giải phương trình đồng dư 99

Định lý 5.6– Với giả thiết định lý 5.3 ta có:

mϕ(t1)

1 +m ϕ(t2)

2 + +m nϕ(tk)

k ≡k−1 (modM) (5.11) Khi nhân vế (??) vớimi ta

m1+ϕ(ti)

1 +mi.tiϕ(mi)+≡mi (modM) (5.12) Domi.tϕi(mi)=mi.ti.tϕ(mi)

−1

i =M.t

(mi)−1

i nên

m1+ϕ(t1)

i ≡mi (mod M), i= 1, k (5.13) Cộng vế kđồng thức dạng (5.13) ta kết sau:

Định lý 5.7– Với giả thiết định lý 5.3 ta có:

m1+ϕ(t1)

1 +m 2+ϕ(t2)

2 + +m 1+ϕ(tk)

k ≡m1+m2+ +mk (modM) (5.14) Khi nhân vế (5.10) với ti ta

m1+ϕ(t1)

1 +m 2+ϕ(t2)

2 + +m 1+ϕ(tk)

k ≡m1+m2+ +mk (modM) (5.15) ⇒t1+ϕ(mi)

i ≡ti (modM), i= 1, k (5.16) Cộng vế kđồng dư dạng (5.16) ta kết sau

Định lý 5.8– Với giả thiết định lý 5.3 ta có:

t1+ϕ(m1)

1 +t

1+ϕ(m2)

2 + +t

1+ϕ(mk)

k ≡t1+t2+ +tk (modM) (5.17) Chú ý ti.tj ≡ (mod M) nên nâng lên lũy thừa bậc n tổngt1+t2+ +tk ta có kết sau

Định lý 5.9– Với giả thiết định lý 5.3 ta có:

tn1 +tn2 + +tnk ≡(t1+t2+ +tk)n (modM) (5.18)

(108)

100 5.6 Ứng dụng định lý Euler để giải phương trình đồng dư

Khả tìm hệ thức đồng dư chưa phải hết mời bạn đọc nghiên cứu thêm Để nắm rõ phần ta tìm hiểu qua số ví dụ sau

Ví dụ 5.10 Tìm bốn nghiệm phương trình đồng dư:

x3+y7 ≡1 (mod 30) (5.19) Lời giải Do30 = 5.6 (6; 5) = nên theo (5.4) có5ϕ(6)+ 6ϕ(5) ≡1 (mod 30)

vìϕ(6) =ϕ(2).ϕ(3) = 2vàϕ(5) = 4; 62 ≡6 (mod 30)

Tương tự ta có:257 ≡25 (mod 30)và63 ≡6 (mod 30)nên63+ 257 ≡ 26 + 6≡1 (mod 30)

Nếu phân tích30 = 3.10với(3; 10) = 1thì theo (5.4) có3ϕ(10)+10ϕ(3) ≡ (mod 30) Tính tốn tương tự ta có34+ 102≡1 (mod 30) Vì 34 = 81 ≡ 21 (mod 30) 102 ≡ 10 (mod 30) nên theo (5.8) có (34)3+ (102)7 ≡1 (mod 30) và(34)7+ (102)3 ≡1 (mod 30)

Suy phương trình có bốn nghiệm(x;y)là(25; 6);(6; 25);

(21; 10);(10; 21)

Ví dụ 5.11 Chứng minh phương trình đồng dư sau có nghiệm (x;y;z;t) khác (0; 0; 0; 0):

x4+y4+z4+t4 ≡t3 (mod 60)

Lời giải 60 = 3.4.5 (5; 3) = 1; (5; 4) = 1; (3; 4) = nên đặt m1 = 3;m2= 4;m3 = 5;t1= 15;t2 = 1;t3 = 20theo (5.18)

154+ 124+ 204 ≡(15 + 20 + 12)4 ≡1 (mod 60)

Ví dụ 5.12 Tìm nghiệm phương trình đồng dư x17+

y19≡1 (mod 35) 4

Lời giải Ta có:35 = 5.7 mà (5; 7) = nên theo (5.4): 5ϕ7 + 7ϕ5 ≡1 (mod 35))

Vìϕ(5) = 4;ϕ(7) = 6nên 54+ 76 ≡1 (mod 35) Theo (5.8):1417+ 3019≡14 + 30≡1 (mod 35)

(109)

5.7 Bài tập 101

5.7

Bài tập

Bài1 Chứng minh phương trình đồng dư sau có nghiệm(x;y;z;t) khác(0; 0; 0; 0):

a)x3+y3+z3 ≡t3 (mod 210) b) x5+y5+z5 ≡t5 (mod 1155)

Bài2 Tìm nghiệm phương trình đồng dư sau:

x11+y13≡1 (mod 45)

Bài3 Chứng tỏ phương trình sau có nghiệm nguyên dương a)2x+ 3y+ 5z+ 7t≡3 (mod 210)

b) 3x+ 5y+ 7z≡2 (mod 105)

(110)(111)

Chương

6

Hệ thặng dư định lý

Thặng dư Trung Hoa

6.1 Một số kí hiệu sử dụng viết 103

6.2 Hệ thặng dư 104

6.3 Định lí thặng dư Trung Hoa 117 6.4 Bài tập đề nghị & gợi ý – đáp số 125

Nguyễn Đình Tùng (tungc3sp)

Bài viết trình bày Hệ thặng dư định lý Thặng dư Trung Hoa Một số kí hiệu sử dụng phác họa Phần 6.1 Phần 6.2 giới thiệu đến bạn đọc số kiến thức Hệ thặng dư đầy đủ Hệ thặng dư thu gọn kèm theo tập ứng dụng Định lý Thặng dư Trung Hoa kèm ứng dụng giúp giải số dạng tốn trình bày Phần 6.3 Phần 6.4 kết thúc viết bao gồm số tập đề nghị kèm gợi ý đáp số

6.1

Một số kí hiệu sử dụng viết

• [x, y] : bội chung nhỏ hai số nguyên dương x, y (nếu khơng nói thêm)

• (x, y) : ước chung lớn hai số nguyên x, y • x,y (modp):xkhơng đồng dư với y theo modulep • HĐĐ: hệ thặng dư đầy đủ

(112)

104 6.2 Hệ thặng dư

• HTG: hệ thặng dư thu gọn • P: tập số nguyên tố

• Φ(n): hàm Ơle củan • |A|: số phần tử tậpA

• {x}: phần lẻ số thựcx, xác định sau:{x}=x−[x], đó[x]là phần nguyên số thực x(là số nguyên lớn không vượt quáx)

• n

Q

i=1

pi =p1p2 pn

6.2

Hệ thặng dư

6.2.1 Kiến thức Hệ thặng dư đầy đủ

Định nghĩa 6.1 Cho tậpA={a1;a2; ;an} Giả sử ri,0≤ri ≤n−1 số dư chia aicho n Nếu tập số dư {r1;r2; ;rn} trùng với tập {0; 1; 2; ;n−1} ta nói A hệ thặng dư đầy đủ (gọi tắt HĐĐ) mod n

Nhận xét Từ định nghĩa, dễ thấy:

NếuA={a1;a2; ;an}lập thành HĐĐ (modn)nếu nếu:

i6=j⇒ai 6=aj (modn)

Nếu A ={a1;a2; ;an} HĐĐ (mod n) từ định nghĩa dễ dàng suy ra:

– Với m ∈ Z, tồn ∈ A cho ≡ m (modn)

(113)

6.2 Hệ thặng dư 105

– Với c ∈Z (c;n) = 1; tập cA = {ca1;ca2; ;can} HĐĐ (modn)

Chú ý: tậpA∗ ={0; 1; 2; 3; ;n−1}là HĐĐ (modn)không âm nhỏ Số phần tử tập A là|A|=n

Ví dụ 6.1 Cho hai HĐĐ (mod n): A={a1;a2; ;an}

B ={b1;b2; ;bn}

a Chứng minh rằng: Nếu n chẵn tập A+B = {a1 +b1;a2 +

b2; ;an+bn} không hợp thành HĐĐ (modn)

b Kết luận câua nlà số lẻ Lời giải a Ta có điều kiện cần sau HĐĐ (modn), khinchẵn Giả sửC={c1;c2; ;cn}là HĐĐ (modn) Khi theo định nghĩa ta có:

c1+c2+ +cn≡(1 + + + (n−1))≡

n(n+ 1)

2 (modn) Donchẵn nên n= 2k, suy ra:

n(n+ 1)

2 =k(2k+ 1)6

.n⇒k(2k+ 1),0 (modn) ⇒c1+c2+ +cn,0 (modn) (6.1) Ta có:

A+B = {a1+b1;a2+b2; ;an+bn}

≡ {(a1+a2+ +an) + (b1+b2+ +bn)} (modn) ≡

n(n+ 1)

2 +

n(n+ 1)

(modn) ≡ [n(n+ 1)] (modn)

⇒A+B ≡0 (modn) (6.2)

(Ở ta sử dụng giả thiết Avà B hai HĐĐ modn) Từ (6.1) (6.2) ta suy đpcm

(114)

106 6.2 Hệ thặng dư

b Xét n lẻ: Lúc chưa thể kết luận tính chất hệ

A+B

Thật vậy, ta xétn= 3;A={1; 2; 3};B ={4; 5; 6} Khi đóA+B ={5; 7; 9}là HĐĐ mod Nhưng, xét hệA={1; 2; 3}, B={5; 4; 6}

Khi đóA+B ={6; 6; 9}khơng phải HĐĐ mod

Hệ thặng dư thu gọn

Định nghĩa 6.2 Cho tậpB ={b1;b2; ;bk} tập hợp gồm k số nguyên và(bi;n) = với mọii= 1; 2; ;k

Giả sử:bi =qin+ri với 1≤ri < n Khi dễ thấy(ri;n) = Nếu tập {r1;r2; ;rn} tập K gồm tất số nguyên dương nhỏ n nguyên tố với n B gọi hệ thặng dư thu gọn mod n, gọi tắt HTG (modn) Nhận xét Ta rút hai nhận xét:

Dễ thấy tậpB ={b1;b2; ;bk} gồm k số nguyên lập thành HTG

i (bi;n) =

ii bi6=bj (mod n) với 1≤i6=j≤k

iii |B|= Φ(n)

Điều kiện (iii) tương đương với (iii0): với x ∈ Z; (x;n) =

tồn nhấtbi∈B chox≡bi (modn)

Từ định nghĩa ta suy ra: cho tậpB ={b1;b2; ;bk}là HTG mod

(115)

6.2 Hệ thặng dư 107

Ví dụ 6.2 Cho hai số nguyên dương m, n với (m;n) = Giả sử A= {a1, a2, , ah};B ={b1, b2, , bk} tương ứng hệ thu gọn mod m modn Xét tập hợp C={ain+bjm}; 1≤i≤h; 1≤j ≤k Chứng minh C hệ thu gọn HTG mod mn Lời giải + Ta chứng minh (ain+bjm, mn) = ∀i= 1, h; j= 1, k

(điều kiện(i))

Giả sử tồn tạii, j số nguyên tố p ước chung ain+bjm vàmn

Ta cóain+bjm pvà mn p

Domn pmà (m, n) = nên giả sửn p, suy

ain p⇒bjm p⇒bj p

Vậy p ước nguyên tố chung nvà bj Điều mâu thuẫn với giả thiết Nên điều giả sử sai Vậy(ain+bjm, mn) = 1∀i= 1, h; j = 1, k

+ Chứng minh điều kiện(ii)

Giả sử tồn a ∈ A; b ∈ B cho an+bm ≡ a0n+b0m

(modmn)

⇒an≡a0n (modm)⇒a≡a0 (modm) (do(m, n) = 1) (điều mâu thuẫn)

Vậyan+bm,a0n+b0m (modmn) + Chứng minh điều kiện(iii0)

Giả sử(x, mn) = 1⇒(x, m) = 1; (x, n) =

Vì(m, n) = 1nên tậpB ={mb1, mb2, , mbk}là HTG mod

n

Vậy tồn nhấtb∈B để x≡mb (modn)

(116)

108 6.2 Hệ thặng dư

Tương tự, tồn nhấta∈Ađể x≡na (modm)

Từ suy x ≡ na+mb (modn) x ≡na+mb (modm) Từ kết hợp với(m, n) = suy x≡na+mb (modmn)

Nhận xét Từ đây, ta suy cơng thức tính hàm Ơle Φ(n) 6.2.2 Ứng dụng

Trong toán đa thức, dãy số

Ví dụ 6.3 [THTT, số 340] Choplà số nguyên tố lẻ đa thứcQ(x) = (p−1)xp−x−1 Chứng minh tồn vô hạn số nguyên dương a

sao choQ(a) chia hết cho pp

Lời giải Thay cho việc chứng minh tồn vô hạn số nguyên dươnga

sao cho Q(a)chia hết cho pp, ta chứng minh tập

H={Q(1);Q(2); ;Q(pp)} HĐĐ modpp

Ta có nhận xét sau: tập số {1; 2; ;pp} gồmpp số, giả sử có hai sốu, vkhác thìQ(u),Q(v) (mod pp)

Ta chứng minh điều phản chứng Giả sử có Q(u) ≡ Q(v) (modpp)

⇔(p−1)up−u−1≡(p−1)vp−v−1 (modpp)

⇔(p−1)(up−vp)−(u−v)≡0 (modp) (6.3) Theo định lí Ferma nhỏ up ≡u (modp) vp ≡vp (mod p) vớip

là số nguyên tố nênup−vp ≡u−v (mod p). Từ (6.3) suy

(p−2)(u−v)≡0 (modp)⇒u≡v (mod p) (6.4) Cũng từ (6.3) ta có:

(117)

6.2 Hệ thặng dư 109

Kết hợp với (6.4) suy

(u−v)((p−1).p.up−1−1)≡0 (mod pp)⇒u−v≡0 (modpp) Điều mâu thuẫn với giả sử u , v (modpp) Vậy nhận xét chứng minh

• Từ nhận xét suy raH ={Q(1);Q(2); ;Q(pp)}là HĐĐ modpp Từ suy tập số {1; 2; ;pp}gồmpp số tồn số a choQ(a)≡0 (mod pp) hay Q(a) pp • Ta xét dãy số hạngak=a+k.pp vớik= 0,1,2 , dễ thấy rằng:

Q(ap)≡Q(a)≡0 (mod pp)

Nghĩa tồn vô hạn sốak (k= 0,1,2, )thỏa mãnQ(ak) pp

Ví dụ 6.4 Cho đa thức P(x) =x3−11x2−87x+m Chứng minh với số nguyên m, tồn số nguyên n cho P(n) chia hết cho

191

Lời giải Ý tưởng tương tự Ví dụ6.3, ta sử dụng HĐĐ Trước hết ta đưa bổ đề sau:

Bổ đề 6.1– Cho p số nguyên tố, p ≡2 (mod 3) Khi đó,với số nguyên x, y màx3 ≡y3 (modp)⇒x≡y (modp)

Chứng minh Thật vậy:

• Nếux≡0 (mod p)⇒y3 ≡0 (modp) ⇒y≡0 (mod p)⇔ x≡

y(modp)

• Nếu x, y khơng chia hết cho p, p ≡ 2(mod3) ⇒ p = 3k+ 2(k∈Z)

(118)

110 6.2 Hệ thặng dư

Theo định lí Ferma:

xp−1 =x3k+1≡1 (modp)

yp−1=y3k+1 ≡1 (modp)

⇒x3k+1 ≡y3k+1 (modp) (6.5) Mà theo giả thiết,x3 ≡y3modp⇒x3k ≡y3k (modp)

Từ suy rax≡y (mod p) Vậy bổ đề chứng minh

Trở lại toán, ta chứng minh P(n1) ≡ P(n2) (mod 191) với

n1;n2∈Zthì n1≡n2 (mod 191) Thật vậy,

27P(n1) = (3n1−11)3−11.191.n1+ 113+ 27m 27P(n2) = (3n2−11)3−11.191.n2+ 113+ 27m nên

P(n1)≡P(n2) (mod 191) ⇔27P(n1) ≡27P(n2) (mod 191) ⇔(3n1−11)3 ≡(3n2−11)3 (mod 191)

⇔3n1−11 ≡3n2−11 (mod 191)(suy từ bổ đề)

⇔n1 ≡n2 (mod 191)

Với n1, n2 ∈ A = {1; 2; 3; ; 1991} (A HĐĐ mod 191),

n16=n2 ta có P(n1),P(n2) (mod 191)

⇒A∗ ={P(1);P(2); ;P(191)} HĐĐ mod191 Từ suy ∃n∈A={1; 2; 3; ; 191}sao cho

P(n)≡191 (mod 191)⇔P(n) 191

Ví dụ 6.5 Cho p số nguyên tố Chứng minh với sốm

nguyên không âm bất kì, ln tồn đa thứcQ(x) có hệ số nguyên chopm ước chung lớn sốan= (p+ 1)n+Q(n); n=

(119)

6.2 Hệ thặng dư 111

Lời giải Ta có bổ đề sau:

Bổ đề 6.2– ∀k∈N, k < m tồn bk∈Z chobkpm+pk k! Chứng minh Giả sửk! =pαkM

k với(Mk;p) = Khi e chạy tập {0; 1; ;Mk−1} số

epm−k lập thành HĐĐ modMk, tồn bk ∈ Z cho bkpm−k ≡ −1 (modMk)

⇔ (bkpm−k+ 1) Mk ⇔ (bkpm+pk) pk.Mk Mặt khác αk ∞ X i=1 k pi < ∞ X i=1 k pi < k

Vậy(bkpm+pk) pαk.Mk=k! Bổ đề chứng minh Trở toán

Đặtfi(x) =

x(x−1) (x−i+ 1)

i! fi(n) =

Cni n≥i

0 n < i

ĐặtR(x) =−m

−1

P

i=0

fi(x)(bipm+pi)thì theo Bổ đề 6.2,R(x) đa thức có hệ số nguyên

Ta có:

un= (p+ 1)n+R(n) = n

X

i=0

Cnipi− m−1

X

i=1

fi(n)pi−pm m−1

X

i=0

fi(n)bi

P

i=0

fi(n)pi− m−1

P

i=1

fi(n)pi (mod pm) ≡

P

i=0

fi(n)pi ≡0 (modpm) ∀n= 1,2,3 Đặc biệtu1 = (p+ 1) +R(1) =epm

(120)

112 6.2 Hệ thặng dư

Ta chứng minh đa thứcQ(x) =R(x)+pm(1−e)là đa thức cần tìm.Thật vậy,

an= (p+ 1)n+Q(n) = (p+ 1)n+R(n) +pm(1−e)

=un+pm(1−e) pm, ∀n= 1,2,3 (6.6) Mặt khác

a1 = (p+ 1) +Q(1) =p+ +R(1) +pm(1−e) =epm+pm(1−e) pm Do pm là ƯCLN của a

n với mọin= 1,2,3

Ví dụ 6.6 Cho p≥3là số nguyên tố vàa1, a2, , ap−2 dãy số nguyên dương cho p không ước số ak akk−1 với mọik= 1,2,3, , p−2 Chứng minh tồn số phần tử dãya1, a2, , ap−2 có tích đồng dư với module p Lời giải Ta có bổ đề sau:

Bổ đề 6.3– Với số nguyênk= 1,2, , p−1tồn tập số nguyên{bk,1, bk,2, , bk,k} thỏa mãn hai điều kiện sau:

1 Mỗibk,j bằng1, tích số phần tử dãy

a1, a2, , ap−2,

2 bk,i,bk,j (mod p) với1≤i6=j≤k Chứng minh Với k=2 chọnb21 = 1;b22 =a1 ,1 (modp) (doa11−1 không chia hết chop)

Giả sử với2 ≤k≤p−2 ta chọn tập {bk,1, bk,2, , bk,k} thỏa mãn hai tính chất

Vìak6 p nên hai phần tử khác tập {akbk,1, akbk,2, , akbk,k} phân biệt theo modp

(121)

6.2 Hệ thặng dư 113

Từ hai điều suy tồn số j(1 ≤ j ≤k) cho akbk,j ∈/ {bk,1, bk,2, , bk,k}

Xét tập{bk,1, bk,2, , bk,k, akbk,j}

Sau đánh số lại phần tử ta thu tập {bk+1,1, bk+1,2, , bk+1,k, bk+1,k+1}

Ta thấy tập có k+ phần tử thỏa mãn hai tính chất nên theo nguyên lí quy nạp, bổ đề chứng minh

Quay lại toán, áp dụng bổ đề6.3, xét tập{bp−1,1, bp−1,2, , bp−1,p−1}, ta thấy tập HTG mod p nên chứa phần tử đồng dư với2 mod p Vì phần tử khác1 nên phải đồng dư với

tích sốak Suy đpcm

Trong tập tập số nguyên dương, tốn số học chia hết Ví dụ 6.7 Cho p >3 số nguyên tố có dạng 3k+

a Chứng minh tập A =

23−1; 33−1; 43−1; ;p3−1 HTG modp

b Chứng minh p

Q

i=1

(i2+i+ 3)≡3(modp) Lời giải a Ta chứng minh tập A thỏa mãn điều kiện nêu

ở Định nghĩa 6.2

• Hiển nhiên phần tử A không chia hết cho p (thỏa mãn điều kiện(i))

• Giả sử tồn tại1≤i < j ≤p−1sao cho

i3−1≡j3−1 (mod p) ⇒ i3 ≡j3 (modp)

⇒ i3k ≡j3k (modp)

Mặt khác, theo định lí Ferma, ta có:i3k+1≡j3k+1 (modp) Từ suy i≡ j (modp) ⇒ i= j (mâu thuẫn) Vậy A

thỏa mãn điều kiện(ii)

(122)

114 6.2 Hệ thặng dư

• VìΦ(p) =p−1 =|A|nên điều kiện(iii) thỏa mãn

VậyA HTG modp

b VìB={1; 2; 3; ;p−1}là HTG modp MàA HTG modp (theo phầna.) nên ta có:

p

Q

i=2

(i3−1)≡(p−1)! (modp) ⇔

p

Q

i=2

(i2+i+ 1)≡1 (modp) ⇔

p

Q

i=1

(i2+i+ 1)≡3 (modp) Nhận xét Ta mở rộng Ví dụ 6.7như sau:

Ví dụ 6.8 Cho p số nguyên tố lẻ có dạng mk+ (m, k số nguyên dương, m >2) Tìm số dư phép chia

T = p

Y

t=1

(tm−1+tm−2+ +t+ 1)

cho p

Ví dụ 6.9 Chứng minh với số nguyên dương n, tồn số tự nhiên n gồm nchữ số lẻ chia hết cho 5n Lời giải Xét số xn = a1a2 an = 5n.a thỏa mãn (với ∈ Z+ lẻ với

mọi i= 1,2, , nvàa∈Z+)

Ta chứng minh toán phương pháp quy nạp toán học Vớin= 1⇒ ∃a1 = 51 Vậy mệnh đề vớin=

Giả sử mệnh đề với n⇔xn=a1a2 an = 5n.a, cần chứng minh mệnh đề vớin+

Xét số sau đây:

a1 = 1a1a2 an= 5n(1.2n+a)

a2 = 3a1a2 an= 5n(3.2n+a)

a3 = 5a1a2 an= 5n(5.2n+a)

a4 = 7a1a2 an= 5n(7.2n+a)

(123)

6.2 Hệ thặng dư 115

DoB ={1,3,5,7,9} HĐĐ mod5

B∗ ={1.2n+ 1; 3.2n+a; 5.2n+a; 7.2n+a; 9.2n+a}

cũng HĐĐ mod nên tồn số B∗ chia hết cho5

⇒Trong sốa1;a2;a3;a4;a5 có số chia hết cho5(n+ 1) mà số gồmn+ 1chữ số lẻ Vậy mệnh đề với n+

Theo nguyên lí quy nạp, mệnh đề với mọin nguyên dương Vậy với số nguyên dươngn, tồn số tự nhiên gồmnchữ số

đều lẻ chia hết cho5n

Trong số dạng tốn Số học khác

Ngồi ứng dụng nêu trên, hệ thặng dư dùng nhiều dạng toán số học khác, đơn biểu tốn liên quan tới tính tổng, giải phương trình nghiệm nguyên (phương trình Diophant bậc nhất) Sau xin nêu số ví dụ

Ví dụ 6.10 Với cặp số nguyên tố (p,q), đặt

S = q p + 2q p + +

(p−1)q p

a Chứng minh rằng: S= (p−1)(q−1)

b Xác định giá trị p, q để S số nguyên tố Lời giải a Ta có

kq p

= rk

q , rk số dư phép chia q

chop (0≤rk≤p−1) Ta có:

S= q

p +

2q

p + +

(p−1)q p −

r1

p + r2

p + + rp−1

p

Vì(p, q) = 1⇒rk 6= 0∀k= 1,2, , p−1, từ ta thấy tậpA= {r1;r2; ;rp−1}chính hốn vị tậpA={1; 2; ;p−1}

(124)

116 6.2 Hệ thặng dư

Thật vậy, ngược lại, giả sử ∃ i, j ∈ {1; 2; ;p−1}, i < j mà

ri =rj

(

1≤j−i≤p−2 (j−i)q p ⇔

(

≤j−i≤p−2

j−i p (vô lý)

Ta có:

r1

p + r2

p + + rp−1

p =

1 + + +p−1

p = p−1

2

⇒S= (p−1)(q−1)

2 (6.7)

b Từ (6.7) suy để S số nguyên tố cần có p, q >1 hai sốp, q lẻ

• Trường hợp 1: p, q lẻ ⇒ p, q ≥ 3, p6= q (do (p,q)=1), kết hợp với (6.7) ⇒S số chẵn lớn 2⇒S khơng phải số ngun tố

• Trường hợp2:plà số chẵn, q số lẻ

S∈P⇔

               

(p, q) =

p−1 =

q−1 ∈P

  

 

(p, q) =

p−1∈P q−1

2 =

⇔    

p=

q = 2h+ (h∈P)

q =

p=t+ (t∈P, t,2 (mod3))

(125)

6.3 Định lí thặng dư Trung Hoa 117

• Trường hợp3:q số chẵn,p số lẻ Tương tự trường hợp 2, ta có:

  

p= 2m+ 1(m∈P)

q=

p=

q=n+ 1(n∈P, n,2 (mod 3))

(6.9)

Từ (6.8) (6.9) ta có cặp sốp, q cần tìm

Ví dụ 6.11 Cho a, b, c số nguyên dương thỏa mãn a ≤ b ≤ c

và (a, b, c) = Chứng minh n > ac+b phương trình

n=ax+by+cz có nghiệm ngun dương Lời giải Gọi(a, c) =d⇒(b, d) = 1⇒A={bi}di=1 HĐĐ mod d

⇒ ∃y∈ {1,2, , d} cho by≡n (modd)⇔(n−by) d

Do (a, c) =d⇒ a= a1d; c =c1d(a1, c1 ∈ Z+; (a1, c2) = 1)⇒ B = {a1j}cj1=1 HĐĐ mod c1

⇒ ∃x∈ {1,2, , c1} choa1x≡

n−by

d (modc1)⇒ ∃z∈Zsao cho n−by

d =a1x+c1z

Mặt khác, ta có:

n−by d >

ac+b−by

d = (d−1)

ca1−b

d +a1c1 ≥a1c1 ≥a1x⇒z∈Z

+

Từ suy n−by=ax+cz⇔n=ax+by+cz

Vậy nếun > ac+bthì phương trìnhn=ax+by+czcó nghiệm ngun

dương

6.3

Định lí thặng dư Trung Hoa

6.3.1 Kiến thức

Định lý 6.1– Cho k số nguyên dương n1, n2, , nk đôi nguên tố vàksố nguyên bất kìa1, a2, , ak Khi tồn số ngun

athỏa mãn a≡ai (modni),∀i= 1, k

(126)

118 6.3 Định lí thặng dư Trung Hoa

Số nguyên b thỏa mãn b≡ai (modni),∀i= 1, k b ≡a

(modn) với n=n1n2 nk

Lời giải • Đặt n=n1n2 nk đặtNi=

n ni

Do(ni, nj) = 1,∀i6=j nên suy (Ni, ni) = ∀i= 1;k

Do(Ni, ni) = 1,∀i= 1;knên với i(1≤i≤k) tồn bi cho

Nibi≡1 (modni) (6.10) Như ta có b1, b2, , bk DoNj ≡0 (mod ni) i6=j, từ dĩ nhiên suy

Njbj ≡0 (modni) (6.11)

Đặta= k

P

j=1

Njbjaj Với i(1≤i≤k)ta có

a=Nibiai+ k

X

j=1;j6=i

Njbjaj (6.12)

Từ (6.10),(6.11),(6.12) suy a≡ai (modni),∀i= 1, k

• Dễ thấy, vìn1, n2, , nk đơi ngun tố nên ta có kêt luận sau: Số nguyên b thỏa mãn b ≡ (modni),∀i = 1, k khib≡a (modn) vớin=n1n2 nk Nhận xét Ngoài cách chứng minh trên, ta cịn sử dụng

phép quy nạp để chứng minh định lí thặng dư Trung Hoa Định lí Thặng dư Trung Hoa khẳng định tồn

(127)

6.3 Định lí thặng dư Trung Hoa 119

điều kiện quan hệ, chia hết, , hay đếm số nghiệm phương trình đồng dư Việc sử dụng hợp lý (trong định lý) cho ta nhiều kết thú vị từ đưa nhiều tốn hay khó

Ví dụ 6.12 Cho m1, m2, , mn số nguyên dương, r1, r2, , rn số nguyên Chứng minh điều kiện cần đủ để hệ phương trình đồng dư

x ≡ r1 (modm1)

x ≡ r2 (modm2)

x ≡ rn (modmn)

có nghiệm ri ≡rj (mod GCD(mi, mj)); ∀1≤i < j ≤n

Nếu x0 x1 hai nghiệm thỏa mãn hệ phương trình thìx0 ≡x1 (modm) với m = LCM(m1, m2, , mn) Tức hệ phương trình cho có nghiệm theo module m Lời giải Trước hết ta giả sử hệ phương trình cho có nghiệmx0 Đặt

GCD (mi, mj) =d, ta có:

xo−ri≡0 (modmi)

xo−rj ≡0 (modmj)

Suy ri ≡ rj mod (GCD (mi, mj)) Do i, j tùy chọn nên ri ≡ rj (modGCD(mi, mj)),∀1 ≤ i < j ≤ n Đây điều kiện cần để hệ phương trình có nghiệm

Ngược lại, ta chứng minh quy nạp theon điều kiện thỏa mãn hệ phương trình ln có nghiệm theo modulem với m=LCM(m1, m2, , mn)

Với trường hợpn= 2, đặt GCD (m1, m2) =d⇒m1=dd1; m2 =dd2 vớiGCD (d1, d2) =

Suy rari ≡rj ≡r (modd) Đặt r1 =r+k1d; r2 =r+k2d

(128)

120 6.3 Định lí thặng dư Trung Hoa

Ta có:

x≡r1 (modm1)

x≡r2 (modm2) ⇔

 

(x−r)−k1d dd1 (x−r)−k2d dd2

 

x−r

d ≡k2 (mod d1) x−r

d ≡k2 (mod d2)

(6.13)

Do(d1, d2) = nên theo định lí Thặng dư Trung Hoa, tồn số dương x cho x ≡ k1 (modd1); x ≡ k2 (modd2) Vì x

x−r d

là hai nghiệm phương trình

x≡k1 (mod d1)

x≡k2 (mod d2) nên

x−r d ≡ x

(modd1d2) hay x≡xd+r (moddd1d2)

Dom=LCM(m1, m2) =dd1d2nên theo định lí Thặng dư Trung Hoa, hệ có nghiệm module m

Giả sử định lí đếnn−1 Ta chứng minh định lí đến n Đặt m01 = LCM(m1, m2, , mn−1) ; m02 = mn; r02 = rn Vì ri ≡

rj(modGCD (mi, mj))với ≤i < j ≤ n nên theo giả thiết quy nạp, hệ phương trình

x≡ri (modmi)

i= 1, n−1 có nghiệmx≡r

0

1 (modm01)

Mặt khác từri ≡rj( mod GCD(mi, mj))với mọi1≤i < j ≤nsuy

r10 ≡r20 (modGCD(m01, m02))

Theo chứng minh cho trường hợp n = ta có hệ phương trình

x≡r10 (mod m01)

x≡r20 (mod m02) có nghiệm theo module

m=LCM m01, m02

=LCM(m1, m2, , mn)

Theo ngun lí quy nạp ta có điều phải chứng minh

(129)

6.3 Định lí thặng dư Trung Hoa 121

6.3.2 Ứng dụng Trong Lý thuyết số

Ví dụ 6.13 Chứng minh với số tự nhiên n, tồn n số tự nhiên liên tiếp mà số trongn số hợp số Lời giải Ý tưởng: ta tạo hệ phương trình đồng dư gồm n

phương trình đồng dư Dựa vào định lí thặng dư Trung Hoa, ta kết luận tồn nghiệm hệ

Giả sửp1, p2, , pn lànsố nguyên tố khác đôi Xét hệ phương trình đồng dư x≡ −k (modp2k)(k= 1,2, , n) Theo định lí thặng dư Trung Hoa, tồn x0 ∈ N∗ cho x0 ≡ −k (modp2k),∀k= 1,2, , n

Khi số x0+ 1;x0+ 2, ;x0+nđều hợp số.(đpcm) Ví dụ 6.14 Chứng minh với số tự nhiên n, tồn n số tự nhiên liên tiếp cho số số khơng phải lũy thừa (với số mũ nguyên dương) số nguyên tố Nhận xét Bài gần tương tự với ý tưởng tốn ví dụ củng cố Tuy nhiên viếc tìm hệ phương trình đồng dư khó chút

Lời giải Với số tự nhiênn, xétnsố nguyên tố khác đôi mộtp1, p2, , pn

Theo định lí Thặng dư Trung Hoa, tồn tạia∈N∗ cho a≡pk−k (modp2k) (k= 1,2, , n)

Khi dễ thấy số a+ 1, a+ 2, , a+n lũy thừa với số mũ nguyên dương số nguyên tố (đpcm)

Ví dụ 6.15 Cho trước số nguyên dươngn, s Chứng minh tồn n số nguyên dương liên tiếp mà số có ước lũy thừa bậc

scủa số nguyên dương lớn Lời giải Xét dãyFn= 22

n

+ 1,(n= 0,1,2, ) Dễ chứng minh bổ đề sau:

Bổ đề 6.4– Nếu n6=m (Fn, Fm) =

(130)

122 6.3 Định lí thặng dư Trung Hoa

Áp dụng định lí Thặng dư Trung Hoa chon số nguyên tố

F1s, F2s, , Fns n số ri = −i(i = 1,2, , n) ta có tồn số nguyên c cho c+i Fis

Vậy dãy{c+i}ni=1 n số nguyên dương liên tiếp, số hạng thứi chia

hết choFis

Ví dụ 6.16 Chứng minh tồn đa thức P(x)∈Z[x], khơng có nghiệm ngun cho với số nguyên dươngn, tồn số nguyên

x choP(x) chia hết chon

Lời giải Ta xét đa thứcP(x) = (3x+ 1)(2x+ 1)

Với số nguyên dươngn, ta biểu diễnn dạngn= 2k(2m+ 1) VìGCD(2k,3) = 1nên tồn asao cho 3a≡1 (mod 2k) Từ

3x≡ −1 (mod 2k)⇔x≡ −a (mod 2k)

Tương tựGCD(2,2m+1) = 1nên tồn tạibsao cho2b≡1 (mod (2m+ 1)) Từ

2x≡ −1 (mod (2m+ 1))⇔x≡ −b (mod (2m+ 1))

Cuối cùng, doGCD(2k,2m+ 1) = 1nên theo định lý Thặng dư Trung Hoa, tồn số nguyênx nghiệm hệ:

x ≡ −a (mod 2k)

x ≡ −b (mod (2m+ 1))

Và theo lý luận trên,P(x) = (3x+ 1)(2x+ 1) n

Ví dụ 6.17 Trong lưới điểm nguyên mặt phẳng tọa độ Oxy, điểm A với tọa độ (x0, y0) ∈ Z2 gọi nhìn thấy từ O đoạn

(131)

6.3 Định lí thặng dư Trung Hoa 123

Lời giải Dễ thấy điều kiện cần đủ để điểmA(x0, y0)nhìn thấy từ O làgcd(x0, y0) =

Để giải toán, ta xây dựng hình vng n×n với n

ngun dương lớn tùy ý cho với điểm nguyên(x, y)nằm hình vng khơng thể nhìn thấy từO

Thật vậy, chọnpij số nguyên tố đôi khác với0≤i, j≤

n Xét hai hệ đồng dư sau:

          

x ≡ (modp01p02 p0n)

x+ ≡ (modp11p12 p1n)

x+ ≡ (modp21p22 p2n)

x+n ≡ (modpn1pn2 pnn)

và           

y ≡ (modp01p02 p0n)

y+ ≡ (modp11p12 p1n)

y+ ≡ (modp21p22 p2n)

y+n ≡ (modpn1pn2 pnn)

Theo định lý Thặng dư Trung Hoa tồn tại(x0, y0)thỏa mãn hai hệ đồng dư

Khi đó, rõ rànggcd(x0+i, y0+i)>1, ∀i, j = 0,1,2, , n

Điều có nghĩa điểm nằm bên biên hình vng

n×n xác định điểm phía bên trái (x0, y0) khơng thể nhìn thấy từO Bài tốn chứng minh

Trong tìm số lượng nghiệm nguyên phương trình nghiệm nguyên

Ví dụ 6.18 Cho số nguyên dươngn=pα1

1 p α2

2 p αk

k , đóp1, p2, , pk số nguyên tố đơi khác Tìm số nghiệm phương trình:

x2+x≡0 (modn)

(132)

124 6.3 Định lí thặng dư Trung Hoa

Lời giải Ta có:

x2+x≡0 (modn)⇔

x(x+ 1)≡0 (mod pαi

i )

i= 1, k

  

 

"

x ≡ (modpαii)

x ≡ −1 (modpαii)

i= 1, k

(6.14)

Theo định lí Thặng dư Trung Hoa, hệ phương trình x2+x ≡ (modn)⇔

 

x≡ai (modpαii)

ai ∈ {−1; 0}

i= 1, k

có nghiệm ta có2k hệ (bằng số (a1, a2, , ak),ai ∈ {−1; 0}), nghiệm hệ khác Suy phương trình cho có 2k nghiệm

Ví dụ 6.19 Cho m = 20072008 Hỏi có tất số tự nhiên n<m cho m|n(2n+ 1)(5n+ 2) Lời giải Dễ thấyGCD (m; 10) = Do đó:

n(2n+ 1)(5n+ 2) ≡ (modm)

⇔10n(10n+ 5)(10n+ 4) ≡ (modm) (6.15) Ta có: m = 34016.2232008 Để cho thuận tiện, đặt 10n = x; 34016 =

q1; 2232008 =q2

Khi đóGCD (q1, q2) = nên (6.15) tương đương với:

x(x+ 5)(x+ 4)≡0 (modq1) (6.16)

x(x+ 5)(x+ 4)≡0 (modq2) (6.17) Dễ thấy:

• (6.16) xảy khix≡0 (modq1)hoặcx≡ −5 (mod q1) hoặcx≡ −4 (modq1)

(133)

6.4 Bài tập đề nghị & gợi ý – đáp số 125

Do từ (6.16) (6.17), với lưu ý x≡0 (mod 10), suy ran số tự nhiên thỏa mãn điều kiện đề khin= x

10, vớix số nguyên thỏa mãn hệ điều kiện sau:

          

x≡0 (mod 10)

x≡1 (modq1)

x≡r2 (modq2) 0≤x <10q1q2

r1, r2 ∈ {0;−4;−5}

(6.18)

Vì10; q1; q2 đôi nguyên tố nên theo định lí Thặng dư Trung Hoa, hệ (6.18) có nghiệm

Dễ thấy có 9sốxlà nghiệm của9 hệ (6.18) tương ứng Vì sốx

cho ta sốnvà hai sốx cho hai sốnkhác nên có9 sốnthỏa

mãn điều kiện đề

Nhận xét Ví dụ 6.19 trường hợp đặc biệt tốn tổng qt sau:

Ví dụ 6.20 Cho số ngun dương n có phân tích tiêu chuẩn n =

pα1

1 p α2

2 p αk

k Xét đa thứcP(x)có hệ số nguyên Nghiệmx0 phương trình đồng dưP(x)≡0 (mod n) lớp đồng dưx0 ∈

0,1,2, , n−1 thỏa mãnP(x0)≡0 (mod n) Khi đó, điều kiện cần đủ để phương trình P(x) ≡ (mod n) có nghiệm với i = 1,2, , s, phương trình P(x) ≡ (modpαi

i ) có nghiệm Hơn nữa, với i = 1,2, , s, phương trình P(x) ≡ (mod pαi

i ) có ri nghiệm module p αi

i phương trình có r=r1r2 rs nghiệm modulen

6.4

Bài tập đề nghị & gợi ý – đáp số

Bài tập đề nghị

Bài1 a Chứng minh rằng: Nếu (a, m) = x chạy qua hệ thặng dư đầy đủ modulo m ax+b, với b số nguyên tùy ý, chạy qua hệ thặng dư đầy đủ modulem

(134)

126 6.4 Bài tập đề nghị & gợi ý – đáp số

b Chứng minh rằng: Nếu(a, m) = x chạy qua hệ thặng dư thu gọn modulom thìaxcũng chạy qua hệ thặng dư thu gọn modulem

Bài2 Mỗi số nguyên dươngT gọi số tam giác có dạng

T = k(k+ 1)

2 , đóklà số nguyên dương Chứng minh tồn HĐĐ modulengồmn số tam giác

Bài3 a Chom1, m2 hai số nguyên dương nguyên tố Chứng minh rằng:

Φ(m1m2) = Φ(m1).Φ(m2)

b Giả sử số nguyên dương m có phân tích tắc thành tích thừa số nguyên tốm=pα1

1 p α2

2 p αk

k Chứng minh rằng:

Φ(m) =pα1−1

1 p α2−1

2 p αk−1

k (p1−1)(p2−2) (pk−1) Bài4 Tính tổng sau:

S= 2012 X k=6 17k 11

Bài5 Cho số nguyên dươngnvà số nguyên tốplớn hơnn+ Chứng minh đa thức P(x) = + x

n+ +

x2

2n+ 1+ +

xp pn+ khơng có nghiệm nguyên

Bài6 Cho p số nguyên tố có dạng 3k+ (k ngun dương) Tìm số dư chiaS =

p

P

k=1

(k2+k+ 1)cho p

(135)

6.4 Bài tập đề nghị & gợi ý – đáp số 127

Bài8 Tìm số ngun dương nhỏ có tính chất: chia dư5, chia 11 dư7, chia13 dư

Bài9 Chứng minh tồn dãy tăng{an}∞n=1các số tự nhiên cho với số tự nhiên k, dãy{k+an} chứa hữu hạn số nguyên tố

Bài10 Số ngun dươngn gọi có tính chấtP với số nguyên dương a, b mà a3b+ n a3+b n Chứng minh số số ngun dương có tính chấtP khơng vượt q24 Bài11 Tìm tất số tự nhiênnthỏa mãn2n−1chia hết cho3

có số nguyênm mà n−1

3 |4m 2+ 1.

Bài12 Chứng minh tồn số tự nhiên k cho tất số

k.2n+ (n= 1,2, ) đều hợp số. Gợi ý – đáp số

Bài1 Chứng minh trực tiếp dựa vào định nghĩa

Bài2 Ta chứng minh n phải có dạng n = 2k Phản chứng, giả sử

n= 2k.m với m lẻ m > Sử dụng tính chất hệ thặng dư đầy đủ

Bài3 Ta chứng minh dựa vào kiến thức hệ thặng dư đầy đủ, chứng minh dựa vào định lí Thặng dư trung Hoa

Bài4 Sử dụng HTG

Bài5 Biểu diễnP(x)dưới dạngP(x) =apxp+ap−1xp−1+ +a2x2+

a1x+a0 Phản chứng, giả sử P(x) có nghiệm nguyên x = u Suy mâu thuẫn

Bài6 Tiến hành tương tự Ví dụ6.7 Bài7 Sử dụng kiến thức HĐĐ

(136)

128 6.4 Bài tập đề nghị & gợi ý – đáp số

Bài8 Đáp số: 887

Bài9 Gọi pk số nguyên tố thứ k, k > Theo định lí Thặng dư Trung Hoa, tồn dãy số {an}∞n=1 thỏa mãn a1 = 2;an = −k(modpk+1),∀k≤n

Bài10 Định lý Thặng dư Trung Hoa

Bài11 Chứng minh n có dạng 2k Sử dụng tính chất số Fecma (xem lại Ví dụ6.15)

(137)

Chương

7

Một số toán số học

hay VMF

7.1 m3+ 17 3n 129

7.2 c(ac+ 1)2= (5c+ 2)(2c+b) 136

Phần gồm số toán hay thảo luận nhiều Diễn đàn Tốn học Bạn đọc vào trực tiếp topic tốn Diễn đàn Tốn học, cách click vào tiêu đề tốn

7.1

m

3

+ 17

3

n

Bài toán 7.1 Chứng minh với số nguyên dương n, tồn số tự nhiên m cho

m3+ 17 3n

Đầu tiên, đến với chứng minh đề xuất cho toán đầu Chứng minh Ta chứng minh toán quy nạp

Vớin= 1, ta chọnm= Vớin= 2, ta chọnm=

Giả sử toán đếnn=k, hay ∃m∈N:m3+ 17 3k

Ta chứng minh trường hợpn=k+ 1cũng tức tồn sốm0 cho m03+ 17 3k+1

Đặtm3+ 17 = 3k.n⇒n6

(138)

130 7.1 m3+ 17 3n

n≡2

n≡1 (mod3)⇒

m3+ 17≡2.3k

m3+ 17≡3k

mod3k+1

• Trường hợp 1: m3+ 17≡2.3k (mod 3k+1) Xét:

(m+ 3k−1)3 =m3+m23k+m32k−1+ 33k−3≡m3+m23k (mod 3k+1) (Dok≥2⇒32k−1 3k+1 33k−3 3k+1)

Suy ra:

m+ 3k−1

3

+ 17≡m3+m2.3k+ 17≡2.3k+m2.3k≡0 (mod 3k+1) (vìm6 3⇒m2≡1 (mod 3)⇒2 +m2 3⇒(2 +m2).3k 3k+1)

Như vậy, trường hợp 1, ta có: m+ 3k−13

+ 17 3k+1. • Trường hợp 2: m3+ 17≡3k (mod 3k+1)

Xét:

m−3k−1

3

=m3−m23k+m32k−1−33k−3 ≡m3−m23k (mod 3k+1) (Dok≥2⇒32k−1 3k+1 33k−3 3k+1)

Suy ra:

m−3k−1

3

+ 17≡m3−m23k+ 17≡3k−m23k≡0 (mod 3k+1) (vìm6 3⇒m2≡1 (mod 3)⇒1−m2 3⇒ 1−m2.3k 3k+1) Như vậy, trường hợp ta có: m−3k−13+ 17 3k+1

Tóm lại, ta tìm số nguyên t6 màt3+ 17 3k+1

Ta chứng minh vấn đề trường hợpn=k+ Theo nguyên lý quy nạp, ta có đpcm

(139)

7.1 m3+ 17 3n 131

Bổ đề 7.1– Cho a, b, qlà số nguyên thỏa(a;q) = q >0 Khi ấy, tồn tạik∈Z choak±b q

Chứng minh Ta chứng minh đại diện cho trường hợpak+b q Trường hợp lại tương tự

XétA={1; 2; 3; ;q}là hệ đầy đủ HĐĐ mod q

Theo tính chất Hệ thặng dư, ta có tậpB={a; 2a; 3a; ;qa} HĐĐ mod q

⇒C={a+b; 2a+b; 3a+b; ;qa+b} HĐĐ mod q

Do đó, tồn tạik∈[1;q]sao cho ak+b q

Nhận xét Bài toán cho thực chất yêu cầu tìm sốx nguyên chox+ 17 3n vàx lập phương số nguyên Bổ đề cho thấy tồn x nguyên để x+ 17 3n Cịn việc tìm x để x lập phương số nguyên ta dùng phương pháp quy nạp Đối với người u tốn, ta phải khơng ngừng sáng tạo Ta thử tổng qt tốn cho:

• thay m3, ta thử thaymk vớik số nguyên dương cố định • thay 3n, ta thử thaypn vớip số nguyên tố

• thay số17 y∈Nvới y cố định

Kết hợp thay đổi trên, ta có tốn "tổng qt" Dự đoán 7.1– Cho p số nguyên tố y, k∈Nvà y, k cố định

Khẳng định phủ định mệnh đề sau

∀n∈N,∃x∈N:xk+y pn (7.1) Ta thử thay vài giá trịp, k, yvào để thử xem (7.1) có khơng Khi thayk= 2, y= 1, p= mệnh đề (7.1) trở thành

∀n∈N,∃x∈N:x2+ 3n (7.2)

(140)

132 7.1 m3+ 17 3n

Rất tiếc, này, (7.2) lại sai!!! Ta chứng minh (7.2) sai khin≥1 Thật vậy, để chứng minh dự đốn7.1 sai, ta cần có bổ đề sau

Bổ đề 7.2– Cho plà số nguyên tố dạng 4k+ 3vàa, b∈Z Khi

a2+b2 p⇔(a p)∧(b p)

Từ (7.2), suy x2+ Áp dụng bổ đề 7.2với p= 3, ta suy 3: vô lý

Vậy khin≥1thì ∃x∈Z:x2+ 3n

Khơng nản lòng, ta thử thêm vài điều kiện để (7.1) trở nên chặt Nếu bạn đọc có ý kiến hay, xin gửi vào topicnày

để thảo luận Sau thêm số điều kiện, ta có tốn hẹp ln

Định lý 7.1– Cho p nguyên tố lẻ y, k∈Nvà y, k cố định Biết rằnggcd(k, p) =gcd(k, p−1) =gcd(y, p) =

Chứng minh rằng:

∀n∈N,∃x∈N:xk+y pn (7.3) Chứng minh Trước hết, để chứng minh (7.3), ta cần có bổ đề sau Bổ đề 7.3– Cho plà số nguyên tố lẻ.k nguyên dương thỏa

(k;p) = (k−1;p) =

Khi đó, {1k; 2k; ; (p−1)k}là HTG modp Chứng minh Gọig nguyên thủy củap tức làordp(g) =p−1 Khi thìg1, g2, , gp−1 lập thành HTG modp rõ ràng

ga1, ga2, , gap−1 là HTG modp⇔a

1, a2, , ap−1 HĐĐ p−1 Với ≤ i ≤ p−1 tồn i ≡ gai (mod p) rõ ràng

ai lập thành HTG modp nên hệ 1k,2k, ,(p−1)k viết lại

gk, g2k, , g(p−1)k, HTG modpkhi khik,2k, ,(p−1)k hệ thặng dư đầy đủ củap−1, tức knguyên tố với p−1

(141)

7.1 m3+ 17 3n 133

Quay lại toán Ta chứng minh (7.3) phương pháp quy nạp Vớin= 1, theo bổ đề7.3thì

∃x0 ∈ {1; 2; ;p−1}:x0k≡ −y (modp)⇒xk0+y p Giả sử toán đếnnhay tồn tạixk+y pn

Ta chứng minh n+ 1cũng hay tồn xk0+y pn+1

Thật vậy, từ giả thiết quy nạp suy raxk+y=pn.q

• Trường hợp 1: q p⇒ đpcm • Trường hợp 2:

gcd(q, p) = (7.4)

Khi ta chọnx0 =v.pn+x Do

xk0+y= (v.pn+x)k+y

=vk.pnk+

1

k

.vk−1.pn(k−1).x+ +

k−1

k

.v.pn.xk−1+ (xk+y) (7.5) Dễ dàng chứng minh

pn+1|vk.pnk+

1

k

.vk−1.pn(k−1).x+

k−2

k

.v2.p2n.xk−2

Do ta xét

k−1

k

.v.pn.xk−1+ (xk+y) =k.v.pn.xk−1+pn.q=pn(k.v.xk−1+q) Nhận thấy giả sửk.xk−1 ≡t (modp) mà gcd(k, p) = xk+y p⇒

gcd(x, p) = (dogcd(y, p) = 1) suy ragcd(t, p) =

Do đó(k.v.xk−1+q)≡tv+q (modp)mà từ (7.4) ta cógcd(q, p) = Cho nên ln tồn vthỏa mãn tv+q p Do toán khẳng đinh vớin+

Theo nguyên lý quy nạp, toán chứng minh

(142)

134 7.1 m3+ 17 3n

Chưa dừng lại đây, (7.3), ta thaykbởix, ta toán khác:

Định lý 7.2– Cho p nguyên tố lẻ y ∈ N y cố định Biết

gcd(y, p) = Khi đó:

∀n∈N,∃x∈N:xx+y pn (7.6)

Chứng minh Ta chứng minh toán phương pháp quy nạp Ta coi định lý 7.1 bổ đề Dễ thấy x thỏa (7.6)

gcd(x;p) =

Khi đó, vớin= 1, ta xét hệ đồng dư (I)

x≡k (mod (p−1))

x≡x0 (modp) đó, x0;k∈N thỏaxk0 +y p

Dogcd(p−1;p) = nên theo định lý Thặng dư Trung Hoa hệ (I) ln có nghiệmx0

Chọnx=x0, ta chứng minh xthỏa (7.6) n= Thật

gcd(x;p) = 1⇒xp−1≡1 (modp)⇒xk≡xx (modp) ⇒xx+y≡xk+y ≡xk

0+y≡0 (modp) Vậy∃x∈N:xx+y p

Giả sử (7.6) đến n−1, tức tồn tạix0 đểx0x0 +y pn−1 Theo cách chứng minh quy nạp (7.6), ta chọn xn = apn+x0 thỏaxx0

n +y pn

Khi đó, dễ nhận thấyxn≡x0 (modpn−1) Ta xét hệ đồng dư (II)

X≡x0 (mod (pn−1(p−1)))

X≡xn (modpn)

Dogcd(pn−1(p−1);pn) = 1nên theo định lý Thặ ng dư Trung hoa, hệ (II) có nghiệmX Ta chứng minh x=X thỏa (7.6) Thật

(143)

7.1 m3+ 17 3n 135

Mặt khácXx0 ≡x

nx0 (mod pn) (do cách chọn hệ (II)) ⇒XX+y≡xnx0 +y≡0 (modpn)

Theo nguyên lý quy nạp, toán chứng minh

Mở rộng toán đầu đề nhiều, tăng thêm điều kiện để chặn như(m3+ 17 3n)∧(m3+ 176 3n+1), v.v Rất mong nhận ý kiến đóng góp cho việc mở rộng

Lời cảm ơn

Rất cảm ơnNguyen Lam Thinh,Karl Heinrich Marx,nguyenta98,The Gunnerđã đóng góp ý kiến mở rộng cho viết

(144)

136 7.2 c(ac+ 1)2 = (5c+ 2)(2c+b)

7.2

c(ac

+ 1)

2

= (5c

+ 2)(2c

+

b)

Bài toán 7.2 Cho số nguyên dương a;b;c thoả mãn đẳng thức:

c(ac+ 1)2 = (5c+ 2b)(2c+b) (7.7) Chứng minh : c số phương lẻ Nhận xét Thoạt nhìn vào tốn, thật khó để tìm phương pháp cho loại Nhận xét giả thiết VP (7.7), b xuất với bậc là2 Thế ta có hướng nghĩ dùng tam thức bậc cho tốn Ta khơng nên chọn c bậc c 3, khơng chọna phương trình theoahiển nhiên trở lại (7.7)

Chứng minh (Chứng minh 1)

c(ac+ 1)2 = (5c+ 2b) (2c+b) ⇔2b2+ 9bc+ 10c2−c(ac+ 1)2 = ∆b = 81c2−4.2

10c2−c(ac+ 1)2=c2+ 8c(ac+ 1)2 ⇒∆b =c

h

c+ (ac+ 1)2

i

=x2,(x∈N∗)

Đặt d=GCD(c;c+ 8(ac+ 1)

2)⇒d|8 (ac+ 1)2

d|c⇒(ac+ 1)2;d=

)

⇒d|8 • Trường hợp 1: d=8 ⇒c

8;

c

8+ (ac+ 1) 2= 1

c

h

c+ (ac+ 1)2

i

=x2(x∈N)⇔ c

8

c

8+ (ac+ 1)

=

x

8

2

⇒8|x⇒x= 8x2(x2 ∈N∗)⇒ c

8

c

8 + (ac+ 1)

=x22

 

c

8 =t

c

8 + (ac+ 1) 2=p2

t;p∈N∗

(t;p) =

c= 8t2

t2+ 8t2a+ 12 =p2

Mà dễ chứng minh

(145)

7.2 c(ac+ 1)2 = (5c+ 2)(2c+b) 137

Do đó,d= bị loại

• Trường hợp 2: d=4 ⇒c 4;

c

4+ (ac+ 1) 2= 1 ⇒ c

4;

c

4+ (ac+ 1)

2 là số phương (*)

Nếu c

4 số chẵn ⇒

c

4+ (ac+ 1) .2 ⇒c

4;

c

4 + (ac+ 1)

= 2: mâu thuẫn Do đó, c

4 số lẻ Mà

c

4 số phương ⇒

c

4 ≡1 (mod 4) Mặt khác, cchẵn nên ac+ 1là số lẻ ⇒(ac+ 1)2 ≡1 (mod 4) ⇒ c

4 + (ac+ 1)

2 ≡1 + 2.1≡3 (mod 4): vô lý (*). Do đó,d= bị loại

• Trường hợp 3: d=2

Tương tự tự Trường hợp 2, ta có c lẻ ⇒

c

2 ≡1 (mod 8)

cchẵn nên ac+ 1lẻ⇒(ac+ 1)2 ≡1 (mod 8)

⇒ c

2+ 4(ac+ 1)

2 ≡1 + 4.1≡5 (mod 8) : vô lý

Do đó,d= bị loại • Trường hợp 4: d=1

Tương tự trường hơp 2, ta có c lẻ (c;c+ 8(ac+ 1)2) = nên clà số phương

Vậy ta có đpcm

Nhận xét Ta thấy này, b c có mối liên quan chặt chẽ với nên ta thử giải theob, c sử dụng kĩ thuậtGCD tức đặt

d=GCD(b;c)ta có cách chứng minh thứ Chứng minh (Chứng minh 2) Đặtd= (b;c)⇒

c=dm b=dn

m;n∈N∗

(m;n) =

(146)

138 7.2 c(ac+ 1)2 = (5c+ 2)(2c+b)

Khi

(7.7)⇔m(dam+ 1)2 =d(5m+ 2n) (2m+n) ⇒d|m(dam+ 1)2

(d;dam+ 1) =

⇒d|m⇒m=dp⇒(p;n) = (d;n) = (7.7)⇔p d2ap+ 12

= (5dp+ 2n) (2dp+n) ⇒p|(5dp+ 2n) (2dp+n)

(p; 2dp+n) =

⇒p|5dp+ 2n⇒p|2n

(p;n) = 1⇒p|2⇒p∈ {1; 2}

• Trường hợp 1: p=2 , 2ad2+ 12 = (10d+ 2n) (4d+n), suy 2ad2+ 12 = (5d+n) (4d+n) Nhưng (5d+n; 4d+n) = (d; 4d+n) = (d;n) = 1Cho nên ta phải có

5d+n=x2

4d+n=y2 (x;y∈N ∗

,(x;y) = 1) Suy rad=x2−y2 Mặt khác

2ad2+ =xy⇔a= xy−1 2d2 =

xy−1 (x2−y2)2 Ta chứng minh2 x2−y22

>(x+y)2 > xy−1 Thật

(x+y)2 ≥4xy > xy−1 x2−y22

−(x+y)2= (x+y)22 (x−y)2−1>0 ⇒2 x2−y22> xy−1⇒a <1 : Trái gt

Vậyp= bị loại • Trường hợp 2: p=1

⇒d=m⇒

c=d2,(i)

b=dn

(7.7)⇔d2 ad2+ 12 = 5d2+ 2dn 2d2+dn

(147)

7.2 c(ac+ 1)2 = (5c+ 2)(2c+b) 139

(5d+ 2n; 2d+n) = (d; 2d+n) = (d;n) = ⇒

5d+ 2n=x2

2d+n=y2

x;y∈N∗

(x;y) =

d=x2−2y2 n= 5y2−2x2

Nếux= 2z vớiz∈N∗⇒

d= 4z2−2y2 n= 5y2−8z2 (7.8)⇔ ad2+ 12

= 4z2y2 ⇔a 4z2−2y22

+ = 2zy

Phương trình cuối vơ nghiệm ngun vế khác tính chẵn lẻ Suy ra, xlẻ ⇒dlẻ⇒clẻ (ii)

Kết luận: (i),(ii)⇒clà số phương lẻ

Khơng ngừng tìm kiếm, ta tìm lời giải khác súc tích Nếu ta biết đến cơng cụvp(n) thấy hiệu cho tốn này, ta có cách chứng minh thú vị sau

Chứng minh (Chứng minh 3) Giả sửc chẵn ta có:

v2(c) =v2(5c+ 2b) +v2(2c+b)

Nếub lẻ ta cóv2(c) =v2(5c+ 2b) =v2(5c)⇒v2(5c)< v2(2b) = Điều vơ lí!

Do clẻ Xétp|c ước nguyên tố củac Ta cóvp(c) =vp(5c+ 2b) +vp(2c+b)

Ta thấy rằngvp(c)> vp(5c+ 2b), vp(2c+b)>0 Do vp(5c+ 2b) =min[vp(c);vp(4c+ 2b)] ⇒vp(5c+ 2b) =vp(4c+ 2b) =vp(2c+b)

⇒vp(c) = 2vp(5c+ 2b): số chẵn nên suy rac số phương Và hi vọng lời giải khác hay hơn, sáng tạo từ bạn Mong bạn đọc thảo luận thêm đóng góp ý kiến cho tốn

Lời cảm ơn

Rất cảm ơn Karl Heinrich Marx,nguyenta98, Vương Nguyễn Thùy Dương perfectstrongđã đóng góp ý kiến cho viết

(148)(149)

Tài liệu tham khảo

[1] Vũ Hữu Bình,Phương trình nghiệm nguyên kinh nghiệm giải [2] Phan Huy Khải,Các chuyên đề bồi dưỡng học sinh giỏi toán trung

học Chuyên đề 5: Phương trình nghiệm nguyên

[3] Phạm Minh Phương nhóm tác giả chun tốn Đại học Sư phạm Hà Nội, Các chuyên đề Số học bồi dưỡng học sinh giỏi Trung học sở

[4] Titu Andreescu, Dorin Andrica, Number Theory: Structures, Ex-amples and Problems

[5] Tạp chí Tốn Tuổi Thơ, Tốn học Tuổi trẻ, Mathematical Re-flections, v.v

[6] Các đề thi học sinh giỏi, tuyển sinh vào THPT, TST, IMO,v.v

[7] Tài nguyên Internet, đặc biệt:

http://diendantoanhoc.net/forum/,

http://www.artofproblemsolving.com/,

http://boxmath.vn

[8] Gv THPT chuyên ĐHKHTN Hà Nội,Bài giảng Số học [9] Đặng Hùng Thắng, Đồng dư phương trình đồng dư [10] Phan Huy Khải,Các toán Số học

[11] Hà Huy Khoái,Chuyên đề bồi dưỡng HSG THPT Số Học

[12] Kỷ yếu hội thảo Tốn học, Tạp chí Tốn học Tuổi trẻ, tạp chí Crux,v.v

(150)

142 Tài liệu tham khảo

142Tài liệu tham khảo [perfectstrong] [Ispectorgadget] [Phạm Quang Toàn] [Nesbit] [T*genie*] Diễn đàn Toán học Diễn đàn Toán học (duongld (yeutoan11 (nguyentrunghieua (Phạm Quang (L Lawliet ( Is-pectorgadget (tungc3sp Nguyen Lam Thinh, nguyenta98, Karl Heinrich Marx, TheGunner perfectstrong (Phạm Quang Toàn khanh3570883 3n này Nguyen Lam Thinh, Karl Heinrich Marx, nguyenta98, TheGunner c( http://diendantoanhoc.net/forum/, http://www.artofproblemsolving.com/, http://boxmath.vn

Ngày đăng: 15/04/2021, 14:42

TỪ KHÓA LIÊN QUAN

TÀI LIỆU CÙNG NGƯỜI DÙNG

TÀI LIỆU LIÊN QUAN

w